Top Banner
GRE ARG ESSAYS Arg-2 Response: 1.The argument has no explanation to believe that there exists any relationship between the characteristics of monkeys and humans. 2.The study is based on only 18 rhesus monkeys which is too small and insignificant a number to be taken as a “clue” for any scientific phenomenon to be researched for an entire species. 3.There is a possibility that the sample selected has given some aberrant result which if believed, will only lead to false interpretations. 4.The argument says “firstborn infant monkeys produce up to twice as much of the hormone cortisol, …, as do their younger siblings”, but doesn’t talk about the number of monkeys that exhibit this characteristic. Is it all? Is it most? Is it few? 5.No explanation about the habitat/ eating habits of the sample studied (the 18 rhesus monkeys) has been given. It is quite possible that all the 18 monkeys belonged to a particular kind of environment/ shared the same food habits that would result in the production of the similar amount of cortisol in them. 6.The study seems to have overlooked the possibility of the presence of some other hormone/ physiological factor in the body of the “firstborns”, that might have had a balancing effect for cortisol. 7.As about firstborn humans also producing relatively high levels of cortisol in stimulating situations, the author doesn’t back the statement by any statistics whatsoever about firstborn “humans”.
289
Welcome message from author
This document is posted to help you gain knowledge. Please leave a comment to let me know what you think about it! Share it to your friends and learn new things together.
Transcript
Page 1: ARG-FINAL

GRE ARG ESSAYS

Arg-2

Response: 1. The argument has no explanation to believe that there exists any relationship

between the characteristics of monkeys and humans.2. The study is based on only 18 rhesus monkeys which is too small and

insignificant a number to be taken as a “clue” for any scientific phenomenon to be researched for an entire species.

3. There is a possibility that the sample selected has given some aberrant result which if believed, will only lead to false interpretations.

4. The argument says “firstborn infant monkeys produce up to twice as much of the hormone cortisol, …, as do their younger siblings”, but doesn’t talk about the number of monkeys that exhibit this characteristic. Is it all? Is it most? Is it few?

5. No explanation about the habitat/ eating habits of the sample studied (the 18 rhesus monkeys) has been given. It is quite possible that all the 18 monkeys belonged to a particular kind of environment/ shared the same food habits that would result in the production of the similar amount of cortisol in them.

6. The study seems to have overlooked the possibility of the presence of some other hormone/ physiological factor in the body of the “firstborns”, that might have had a balancing effect for cortisol.

7. As about firstborn humans also producing relatively high levels of cortisol in stimulating situations, the author doesn’t back the statement by any statistics whatsoever about firstborn “humans”.

8. The situation to study the stimulation level in the monkeys refers to meeting someone “unfamiliar” while that in case of the humans is stark opposite i.e. to meet someone as “familiar” as their parents. These opposite situations may be but also may NOT be taken to have the same effect.

Based on the study conducted, eighteen rhesus monkeys are a very small size to measure the situation of cortisol stimulation. The stimulation depends upon individual’s order. The rhesus monkeys are only one type of monkey. There are also so many other type of monkeys which belong to different groups and have

Page 2: ARG-FINAL

different order of stimulation. Same as in human, as they belong to different area they have different characteristics. Also here author did not define the habits of their living styles and food. There are flaws and lack of information in the argument that arguer doesn’t give us any scientific information about stimulation order. Without scientific information we can’t reach to any conclusion.

Other argument shows that, when monkeys meet with other unfamiliar monkeys then this stimulation situation happens. But in human the stimulation happens when they meet with a familiar person after a time. Let us take an example, suppose parents have only one child, then all the concentration and attention goes to one child. But if his parents go to an office or somewhere else. In such situation first-born child shows high level of stimulation when they return home after an absence. But when younger siblings are born. They don’t take only their parents attention but also their elder siblings. This will decrease the feeling of loss when parents leave. This shows that younger siblings have less cortisol and elder one. The younger siblings also get excited for their parents’ return but not as the elder one. Because the older one feel more loneliness than their young siblings.

Author states that first-time mother monkeys have a high level of cortisol in the first-born child than other offspring. He didn’t mention why high level of cortisol lead to the firstborn child. The case of stimulation in monkeys and humans is different.

To summing it up, author must provide concise, detailed and scientific proof about cortisol stimulation. Author gave no information about living styles and food habits. Without proper information we can reach to a conclusion.

Many studies have been done on the effect of birth order on children. It is an interesting subject to us that simply the order in which we are born could affect our life. The scientific study mentioned in this letter to the editor shows that birth order does effect cortisol levels. What this does not account for however is the multitude of environmental factors that come with being a first-born child. The conditioning received from these environmental situations could also influence

Page 3: ARG-FINAL

first-born children in different ways than their siblings, thus producing higher levels of cortisol.

In the study of the rhesus monkeys, first born infant monkeys produced higher levels of cortisol when meeting other monkeys for the first time. This should not simply be amounted to chemistry and hormone production. Other factors that could influence this are that the first-borns spend more time alone and thus are more excited to meet new monkeys. Any siblings would have the advantage of a brother or sister to entertain them; the first-born however spends time mostly with the parents until meeting a new monkey. There is not as much socialization for the first-born in its own family, making it a more exciting experience to meet new monkeys.

This study is also correlated to humans in mentioning that first-born children produce higher levels of cortisol in such situations as a parent returning after an absence. Again, environmental factors could account for this. First-born children receive all the attention of their parents until their younger siblings are born. When these younger siblings are born, they receive not only the attention of their parents, but also of their older siblings. They have that attention of their older siblings to partially negate the feeling of loss when their parents leave. This could account for their lower levels of cortisol. While they may be excited at a parent’s return, they did not feel the same sense of loneliness that their older sibling might have experienced.

These explanations are by no means definitive in explaining why there are higher levels of cortisol in first-born monkeys as compared to their younger siblings. They are missing too much crucial information about living conditions and environmental factors to be anything other than hypotheses as to why this happens. They are meant to highlight the fact that they study says the cortisol levels are higher, but gives no explanation as to why this might be. The reader is left to suppose that it is because of genetics when there are so many other factors that could contribute to these higher levels. To definitively put forth this argument, all factors must be considered, especially the environmental.

ARG-3

In the period of two years, the number of shoppers in Central Plaza have been diminishing. According to the arguer, this is the result of the increase number of

Page 4: ARG-FINAL

skateboard users. The arguer has also stated that there has also been a dramatic increase in the amount of litter and vandalism. The arguer predicts that if the city prohibits skateboarding in Central Plaza, business would return. This recommendation seems like plausible, but fallacious at the same time.

First and foremost, what supportive reasons do the arguer have to conclude that the increase in number of skateboarders have caused the decrease in business? He/she only stated that the numbers of shoppers have decreased in the past 2 years. Two years isn’t a long enough time to predict that it is the cause of decline in business. Also, we have to take into consideration the economic status of the country. In an economic crisis such as the one that we experienced in 2008; consumers cutback on spending. It could be that coincidentally, skateboarders have started to come to Central Plaza to skateboard. With the extra available spaces from the diminished number of shoppers, skateboarders began to gather there more often.

Secondly, the issue of increase of littering; what evidence does the arguer have to show that they are done by skateboarders? There is no concrete proof aside of the assumption that is conveniently made because of a small trend of decline in business in the past two years. It seems that the arguer is using the skateboarders as a scapegoat for a bigger problem that the city is facing. As stated before, there could be an economic downturn that could’ve stifle consumers spending habits. Also, with economic downturns, there are a lot of government and business cutbacks. Janitorial duties could have been cut down so Central Plaza could save money. Instead of cleaning up the plaza every day, it now could be every other day. And the garbage gets piled up.

Thirdly, the rise of vandalism has also played part in the declined of business, the arguer stated. It is true that the aesthetic of an area can make a place of business a success or a fail; however, has the arguer considered that gangs may have defaced the plaza instead of skateboarders? Parts of gang activities are known to vandalize public places with graffiti. In the past two years, has there been new residents moving into the neighborhood around Central Plaza? There could be new families with delinquents lurking in the area looking to cause trouble.

Page 5: ARG-FINAL

Without much evidence, the arguer cannot falsely condemn skateboarders for the decline in business. There are many external factors to the misfortunate of businesses. Instead of recommending to prohibit skateboarding in the plaza, how about asking the city to build a designated area in the plaza for skateboarders? This way they have their own place to skate and does bother shoppers by swooping through them on skateboards

It is very easy to assume that skateboarders, who are usually teenagers, would be responsible for the increase in litter and vandalism. Most people associate the youth as being irresponsible and careless and there seems to be an assumption that skateboards are even more so reckless. Similarly, it would be safe to assume that their presence may keep other customers away, which would result in a decrease in sales for the surrounding business. However, this strong correlation does not necessarily prove that they skateboarder cause these events to occur. In order to prove the skateboarders are to blame the author most prove that the increase in litter and vandalism and decrease in sales is only due to the skateboards and not any other outside factors.One such outside factor is the economy. Research reports have shown that when the economy struggles there is a decrease in sales and an increase in theft and violence. The committee making the decision would have to prove that the recent downfall of the economy would was not responsible for the decline in Central Plaza sales. A possible suggestion would be to gather data from similar geographical areas that were hit by the economy and prove that overall business sales did not take financial hits. The purpose of this data would show that the economy was not the main factor in decrease in business sales.Another component that needs to be addressed is business appeal. If central plaza contains businesses that are out of touch with current styles, then a lack of buyers interest may be the cause and not the skateboarders. In addition, the committee would have to investigate if all business where affected, or just particular stores. If only particular stores, for example expensive furniture stores were affected, then a lack of interest may be a possible factor. The committee would have to compare if profits in all stores took a fall. Furthermore the number of shoppers may have decreased but this does not mean that sales in all business

Page 6: ARG-FINAL

decreased. Shoppers may be only purchasing items from certain stores, again business appeal and profit numbers would answer theses questions.In addition to business appeal, another factor to consider is local competition. The committee would have to scan the local area to see if new businesses outside the Central Plaza have opened. The local competition may be a driving factor that is keeping shoppers away and business sales downIn regards to increase vandalism and littering, the committee would have confirm that in the absence of the skateboarders there is a decrease in vandalism and littering. Since most skaterborders are young teenagers, then a solutuion is too look at the amount of littering and vandalism that takes place during school hours and after school hours. Another suggestion would be to take data during the summer and compare it to the school year. If the results show there is a dramatic increase in vandalism and littering when the skateboarders are most likely to be there this would strengthen the case against skateboarders.While it may sound easy and logical to point the finger at the skateboards, the author must prove that economy, local competition and consumer appeal do not have an influence in the sale of the Central Plaza. If the author were to provide such evidence then it would strengthen the request against the skateboarders.

It is very easy to assume that skateboarders, who are usually teenagers, would be responsible for the increase in litter and vandalism. Most people associate the youth as being irresponsible and careless and there seems to be an assumption that skateboards are even more so reckless. Similarly, it would be safe to assume that their presence may keep other customers away, which would result in a decrease in sales for the surrounding business. However, this strong correlation does not necessarily prove that they skateboarder cause these events to occur. In order to prove the skateboarders are to blame the author most prove that the increase in litter and vandalism and decrease in sales is only due to the skateboards and not any other outside factors.

One such outside factor is the economy. Research reports have shown that when the economy struggles there is a decrease in sales and an increase in theft and

Page 7: ARG-FINAL

violence. The committee making the decision would have to prove that the recent downfall of the economy would was not responsible for the decline in Central Plaza sales. A possible suggestion would be to gather data from similar geographical areas that were hit by the economy and prove that overall business sales did not take financial hits. The purpose of this data would show that the economy was not the main factor in decrease in business sales.

Another component that needs to be addressed is business appeal. If central plaza contains businesses that are out of touch with current styles, then a lack of buyers interest may be the cause and not the skateboarders. In addition, the committee would have to investigate if all business where affected, or just particular stores. If only particular stores, for example expensive furniture stores were affected, then a lack of interest may be a possible factor. The committee would have to compare if profits in all stores took a fall. Furthermore the number of shoppers may have decreased but this does not mean that sales in all business decreased. Shoppers may be only purchasing items from certain stores, again business appeal and profit numbers would answer theses questions.

In addition to business appeal, another factor to consider is local competition. The committee would have to scan the local area to see if new businesses outside the Central Plaza have opened. The local competition may be a driving factor that is keeping shoppers away and business sales down.

In regards to increase vandalism and littering, the committee would have confirm that in the absence of the skateboarders there is a decrease in vandalism and littering. Since most skater borders are young teenagers, then a solution is to look at the amount of littering and vandalism that takes place during school hours and after school hours. Another suggestion would be to take data during the summer and compare it to the school year. If the results show there is a dramatic increase in vandalism and littering when the skateboarders are most likely to be there this would strengthen the case against skateboarders.

Page 8: ARG-FINAL

While it may sound easy and logical to point the finger at the skateboards, the author must prove that economy, local competition and consumer appeal do not have an influence in the sale of the Central Plaza. If the author were to provide such evidence then it would strengthen the request against the skateboarders.

ARG-4

In the letter, the homeowner argues that a estate firm---Adams,is superior to Fitch, which is another leading firm in his town. To bolster it, the homeowner cites various evidence to support it. However, none of these is convincing because the claims relies on a series of unsubstantiated assumptionswhich render it unconvincing on it stands.First, the argument unfairly rests on the assumption that the agents from these two firms have equal professional skills, thus, the author argues that more agents indicates greater efficiency. However, absent evidences to support it. So, it is as likely that agents in Fitch have much better abilities to deal with matters like promoting the sales of houses though the number of agents in there is relatively less than Adams'.Besides, many of the agents in Fitch work only part-job indicates that they have wider circle of people in other fields. Thus, they will gain more clients when they work as agents, which is a beneficial factor for the development and economical efficiency of the firm.

Second,the homeowner also unfairly assumes that these two estate firms sold the same amount of houses last year. According to it, he claim that higher sale price equate to much more income. However, the letter contain no convincing evidence to support it. Lacking the evidences that this is the case,it is entirely possible that in last years, Adams sold 5 houses, for each of them is $168000,, whereas Fitch sold 15 houses,for each of them is $144000. In this case,it's clear to see that Fitch contains more economic benefits compared with Adams. Thence,

Page 9: ARG-FINAL

the argument of the homeowner that Adams is superior to Fitch will be unconvincing.

Thirdly, the homeowner assumes that the conditions of estate market of recent are similar to ten years ago. So , the author judges Adams is better for it took relatively shorter time to sale a houses last year while ten years ago, Fitch spent 4 month to sale one. However, the assumption is unsound regarding of everything is changing .Beside, no convincing evidences in the letter are used to support the assumption. Due to it ,it is as likely that with the improvement of people's lives, the richer become more and it's just a cake for them to buy a house while ten year ago,a large part of people could not afford it. Moreover, rather than buying houses for living, they prefer to invest on them so as to earn more money.

In sum , the argument is unpersuasive. The author need to offer more evidences to substantiate these following assumptions:1)the agents in the two firms have the same professional abilities.2) The two firm sold the same amount of houses last year.3)the conditions of estate market are constant. Without these evidence, the homeowner cannot convince me.

This letter is concludes that Adams estate firm is superior than Fitch reality estate firm. Homeowner's experience states that his home listed with Adams sell faster than the Fitch real estate firm.

Firstly,Adams has 40 real estate agents; fitch has 25, many of them work only part-time.However, the author provides no information about qualification of a real estate firm; also, how much hours per week these agents worked. Lacking such information, it is possible that estate agent work part-time is more efficient than a full-time agent, also ,a smaller firm is more efficient than the larger firm. Moreover, author didn't provide how many Fitch's agents work part-time.

Secondly, the author states that it took Fitch's firm longer to sell one of his house than Adams's firm to sell another one of his home ten years back. However,

Page 10: ARG-FINAL

there are many possible factors to sell the home such as change in economic condition beween last year and 10 years earlier, location of...

ARG-5

The conclusion of the passage auther states is that the reduction in the moped rentals shall reduce the number of accidents occuring on Balmer island.Auther fails to support his assertion on some of the following assumptions.At first, auther has stated that there has been increase in the population of Balmer Island and so the number of mopeds rental should be reduced which does not make sense as the more population will go against the govt decision and so the pedestrian amount is also going to increase if the number of modes of transportation present are less than the population.

Secondly, if the population does not rent the moped who says they can't just buy it?They will definitely have their own vehicle may be borrowin money loan,etc. as the mode of transport is necessary to achieve the living of society.So the claim for reducing rentals reduces accidents is totally absurd.Again,comparing the town near Seaville with Balmer also does not hold good support for the auther's assertion as the regional conditions of roads may be different and increase in the population is the factor with Balmer Island only.Also by reducing the rentals through companies the govt is making the companies angry for reducing their profits and consequently reducing the employment in the

regional area for the requirement fall.So in colclusion to the auther's response we must say the major response should be to take care of roads and look at the other modes of transportation as well as making the current transportation better instead of cutting the rentals of mopeds speed limits imposition and traffic regulations would serve the purpose better.

The major question which the town council of Balmer Island faces is how to reduce the number of accidents involving mopeds and pedestrians on Balmer

Page 11: ARG-FINAL

Island. The recommendation given by the town council of Balmer Island to reduce the number of accident might not have the predicted result.

As mopeds are the major sources of communication in town and also most of the accident of pedestrian is because of mopeds according to the report thus, town has to reduce the use of mopeds. To overcome this problem the town council of Balmer Island recommended to limit the number of mopeds rented by the island's moped rental companies from 50 per day to 25 per day during the summer season. This recommendation is based on the results which have been seen on Seaville Island. The town council of Seaville Island by limiting the number of rentals reduced the 50 percent annual reduction in mopeds. By seeing a positive result at one place one should not take decision because it might be wrong sometimes.The recommendation to limit the number of mopeds might not be useful and also has some drawback. It would not be useful if a person would buy a moped instead of rent it. Here, council could not ask a mopeds companies to not to sell the mopeds. Disadvantage by doing so is for the rental companies which will go in loss. Also author said that mopeds is the major mode of communication in town so by limiting the number of mopeds rented in another way we are creating a problem for the peoples who live in Balmer Island.We can solve this problem by introducing a strict traffic rule, by constructing a proper street for pedestrians and a proper signal for pedestrians on road crossing and on busy roads.In summary, by a seeing a positive result on Seaville Island, council should not make decision. Instead of limiting the number of mopeds rented council have to recommend a proper traffic rule to reduce the number of accident.

that summer in Balmer island sees an increase of 100000 people. Even if the figures stated are true, there cannot be an exponential increase in the number of people over one summer week or so. Hence it can be safely stated that the increase in foot traffic is gradual and not abrupt. So abruptly dropping the rentals of mopeds by half would mean a drastic fall in sales by moped rental companies, yet there may not be any marked or significant change in the frequency of accidentfrom mopeds.It has been indicated here that there is a direct correlation between the number of paedestrian accidents and the number of mopeds rented. In this extrac,t

Page 12: ARG-FINAL

mentions of the ratio of the number of mopeds owned by people to the number of mopeds rented hasn’t been brought forth. If the ratio is quite high then it would indicate that the significant amount of mopeds are owned and not rented, hence no considerable change or decrease in the number of road accidents can be brought about with reduction in the...

ARG-6

While it may be true that the declining rates of deer may be partly due to the global warming which hinders the deer's migration pattern. But from the evidence in the argument, it is not strong enough to conclude that the deer declining rate is only due to global warming.

The above argument states that declining rate of deer is reported by local hunters, it could be pointed out that hunters, their increased rate of hunting for food could also be one of the reasons why numbers of deer are reducing. Due to global warming, the deer will be restricted to one part of the island hence increasing the chance for the hunters to poach. 

The numbers of deer population is reported by the local hunters and not by any wildlife authorities, so it could also mean that the numbers of deer available for hunting is reduced or deer reduced due to their cruel hunting and also these reports cannot be fully accounted as true. 

Page 13: ARG-FINAL

Global warming makes the island warmer and hence it could also lead to the fact that plants can survive in the warmer climate and hence deer do not have to migrate to other islands; as it is already stated in the argument that deer migrate mostly for food. Hence weakening the argument that migration pattern could lead to decline deer population.

Building upon the implication that major reason for reduction in deer population is due to global warming, , as global warming trends cause the sea ice to melt, hence making it unable for deer to migrate to other islands for food, hence it starve to death. This can be reduced if more awareness is created for global warming and possible steps taken for reduction of global warming. In spite of these measures if the hunters continue to hunt down in more tremendous rate, without the concern of extinction of the deer population, control of global warming cannot provide a solution for the decrease in the deer population.

From the above evidence stated author cannot conclude that the decreasing rate of deer population is due to global warming. It is clear that hunters are the main cause for decline in deer population and to hide their cruelty towards these animals, blaming global warming as a reason for decline of deer. 

You need to explicitly identify the type of evidence

Page 14: ARG-FINAL

that would be needed to evaluate the original argument. That is, you need to write sentences like the following: “To evaluate the argument, we would need to know if the hunters’ reports were based on a comprehensive survey of the deer population in the islands, or else we would need such a survey to confirm that deer populations are declining.” 

The reading stated that there is a strong direct relationship between

global warming and decline of deer population. The author is assuming

causal relationships between two pieces of evidence that aren’t

necessarily true. There are alternative explanations that may also be

true given the material presented.

First, global warming refers to increase in average temperature of

atmosphere and oceans, caused by rising concentrations of

greenhouse gases generated by human activities. Climate change

produces an increase in sea level and species extinction, but it doesn´t

reduce the plants which feed to deer populations. It is natural to think

that a higher temperature favors the growth of the plants;

consequently, the deer would have more food.

Second, global warming has been occurred during one hundred years.

If there are an association between climate change and decline of deer

population, this problem had to occur several years ago. But one can

logically infer based on the information provided in the passage that

this problems is happening in the present.

Third, we need more information about the reports from local hunters. I

would like to know the percentage decrease of deer populations, the

number of years in which this change has been happening or the way

in which the local hunters’ reports are realized.

Page 15: ARG-FINAL

Fourth, the author argues that the decline in deer populations are the

result of changes in migration patterns across the frozen sea. But deer

have food; consequently, they can feed, grow and reproduce. Author´s

theory cannot explain the main reason for decline in deer populations.

On the other hand, it is natural to think that decline in deer populations

are associated with global warming, because the author gives no facts

or proofs to consider other causes of this problem.

In conclusion, there isn´t sufficient information based on the argument

to safely state a relationship between climate change and decline in

deer populations. The writer needs to present more evidence to

support his opinion.

This argument is not based on reports from any authentic sources and is thereby very unconvincing. It

rests on the feedback given by local hunters who themselves can be misguided as they are not the

experts to notice the changing patterns. There is every possibility that the arctic deer have migrated to

other regions leaving the hunters to make guesses. It is equally possible that in spite of the climatic

changes, it is actually the local hunters who are responsible for the migration or decline of the arctic deer.

Hence, the inability of deer to cope up with the changing climate is probably only a word spread by

hunters to cover up the cruel pace at which they went on hunting the deer.

The author has quickly jumped to the conclusion that the population of arctic deer is waning because of

global warming hampering their age-old patterns of migration across the frozen sea. Though it is true that

global warming has caused sea ice in Polar Regions to melt, but it may not be the cause of decline of the

arctic deer. They need a habitat cold enough for at least some part of the year and this condition is likely

to have been met in the arctic region. Moreover, if global warming was the cause, then it should have

affected other animals in the same region as well. No such issue is covered in the editorial.

The author was not only quick to assume that there is a decline in the arctic deer population, but also

overlooked any causes of this, other than the global warming trends, when there can be different reasons

for the decline of deer, if at all it is true. It could be due to their natural habitats being disturbed by

increasing human interference, or due to increase in the number of their predators. The fall in number

may also be associated with some sort of pollution of water or due to shortage of the plants they feed on.

There may even be lack of suitable environment for their mating. Before arriving at any conclusion, the

adaption trends of different animals, especially arctic deer in this case should have been studied. Since it

is quite likely that just as birds change their timing and countries of migration, adapting to the changing

climates, arctic deer have also adapted to the rise in temperature due to global warming and migrated to

a different location.

Conclusively, the argument given by the author in the editorial of wild life journal is denounced due to lack

of evidence. The author should have supported it with more reliable statistical data.

Page 16: ARG-FINAL

The above argument is relatively sound; however, close scrutiny of the evidence reveals that it

accomplishes little toward supporting the author's claim, as discussed below.

Firstly, the argument is based on the assumption that arctic deer, in search of food, meanders from island

to island. However, no evidence has been cited to substantiate this assumption. Lacking this, it is entirely

possible that the arctic deer look for its food in a single island. And if this holds true, the editorial becomes

unwarranted.

Secondly, even if deer move from one island to another island, the author claims that there is a decline in

the deer population on the basis of reports documented by hunters. However, in the journal no evidence

has been provided about the accuracy of local hunters report. Perhaps the hunters came to such a

conclusion on the basis of their observation only. Without considering the tools used by local hunters for

making the report, it cannot be inferred that there is an actual decrease in Arctic deer species. 

Even if it is assumed that the deer population is dwindling, the argument further relies on an additional

assumption that this decline is owing to global warming. Perhaps there is a scarcity of vegetation on

which the deer survives. Or perhaps the spree hunting by local hunters is the actual cause behind this

decline. Without ruling out these possible explanations for the decrease of deer species, the author

cannot justify the conclusion that global warming is the factor responsible for this.

Finally, the author asserts that deer fails in following their age-old migration patterns across the frozen

sea because of the melting of sea. Yet, no evidence has been provided to justify the author's assertion.

For that case, perhaps the deer develop a liking for the new environment and they don't basically migrate

back.

In sum, to make this argument persuasive, the author needs to provide clear evidence that the migration

of deer take place. The author must also show that local hunter report is accurate, and the other possible

factor responsible for the decline in Arctic deer population. 

ARG-7

The recommendation maintains that by opening a cafe, where the children's books place, Monarch would better compete with another bookstore called Regal, on the basis of its wide selection of books and a nationwide census indicating that the percentage of the population under age 10 will decline significantly. Given these facts, we, however, are unable to reach the conclusion that a cafe should be set by discontinuing the children's book section. Here are the questions we should proposed, pointing to the flaws made in the recommendation.First, it is said in the recommendation that for the wide selection of books, attracting a great number of customers, Monarch already has, by opening a cafe would bring more customers while children's books have to be deserted. This is implausible, in that the majority of the customer in Monarch might be children and without keeping serving children's books, the number of customers would decrease significantly, in spite of the opening of the cafe. We recommend that the bookstore make clear the comprise of its customers, especially the proportion the children customer possess. Second, mentioned in the recommendation, the opening of the cafe would be able to attract more

Page 17: ARG-FINAL

customers and better compete with Regal Books. It is assumed that even though the cafe seems to add an extra group of customers to the original customers, the bookstore would gain more profit than before the setup of the cafe. The assumption is not correct because of the uncertainty that whether the profits the newly-opened cafe is about to gain are able to compensate the lose resulted from the canceling of the children's books. Accordingly, it is difficult to predict whether Monarch would compete with Regal. Plus, whether the newly-opened cafe in Monarch is about to be profitable remains unknown, in spite of the lucrative result of the cafe opened in Regal. Thus we recommend that the recommendation should include the reason why the cafe opened in Regal is profitable.Third, the national census quoted in the recommendation indicate a decline in the percentage of the population of the population under age ten is useless in terms of the Monarch's decision on supplanting the children's books with a cafe for the following reasons. Firstly, the percentage of the population does not refer to the absolute quantity of the people. So, a decline in percentage is not equivalent to a decline in quantity of children under ten. Secondly, the nationwide census is unlikely to present the situation in the town where Monarch is located, which means the number of children under ten is not bound to decrease; it is reasonable to surmise even an increase in the population of children under ten. At last, it is not likely that only children under ten will go to the area where children's books are placed. Children above ten, or even adolescents and adults read children's book probably, or even exclusively. As a result, the nationwide census is meaningless to Monarch.In conclusion, we have presented the flaws in the conclusion made in the recommendation and questioned the the assumptions. It is more rational to make a decision on the cafe thing by investigating the reason why the cafe in Regal is lucrative and refer to a more adequate local census.The recommendation made by the Board of Directors of Monarch Books of opening a cafe within the book store may be perceived as a good business opportunity, but more precise facts should be mentioned and scrutinized before arriving at the ultimate conclusion. There are many unsubstantial facts in the recommendation which cannot insinuate the conclusion without the use of assumptions. The board of director fails to mention any data about the structure and size of the bookstore, if the book store is small and over occupied by books, then accommodating some space for the cafe might not be good idea. As, this well reflect a bad reputation to the customers. The book store must be spacious first, so that the customers feel comfortable while sifting through books. The Director recommendation states a fact that according to a national census the population of children under the age of ten has decreased. This data is a matter of significance at country level, it cannot be inferred that the population residing in the locality of the book store will reflect a similar trend. Instead of national census, if the recommendation had included a city census, then it would have been clearer, whether the children section needs to be totally ousted from the book store. So that, a cafe can use the freed space. Recommendation should contain facts about the age groups of the book buyers and the number of books sold in different sections. Such statistics will show the actual picture about the customers and most sought book sections. These data are essential to decide, whether the children section or another section should be removed for accommodating a cafe, while considering the profitability of the

Page 18: ARG-FINAL

book store. The director also needs to mention the statistical number of genuine book buyers compared with the actual customers who show up in the book store. It may be possible that opening of cafe, increases the number of customers who don't really want to buy books, rather than, they show up at the store for a sip of coffee. Recommendations states that Regal Books has also opened a cafe inside the book store vicinity, but no facts are mentioned about the effect of the cafe in his book business. It's plausible that there was no change in his revenue after the opening of the cafe. Conclusively, to portend the effectiveness of the recommendation, the board of director should collect more statistical information about the books sold and their customer base. In absence of concrete and precise facts, the recommendation fails to prove that opening of cafe will have a positive effect on the book store business of Monarch Books.

In the recommendation, the author argues to open a Cafe in Monarch Books so as to attract more customers. He cites various evidences to justify it. However, a meticulous analysis will show otherwise. To bolster the argument, the author needs to answer some questions about the argument. First, does the large number of customers who prefer to go to Monarch Books like to drink coffee there? Second, whether the percentage of the population under age ten in the area can reflect that of the national's or not. Third,how does the competition abilities of Regal Books?Firstly, whether or not do the Monarch Book's customer like to drink coffee in there will decide the number of the customers if Monarch Book open a Cafe in there. Possibly, in past, there are many people prefer to read and select books in Monarch Book indeed due to its peaceful atmosphere. If opening a Cafe, it will be noisy,which the people don't like. In contrast, if people like to read book while drinking coffee. It will bolster the argument.Second, about the the percentage of the population under age ten. If the area can not reflect the national's trend of these percentage, it will weaken the argument. Thus, discontinuing the children's book section may lead to the decreasing of some part of customers.Third, it's indispensable to know the competition abilities of the Regal Books, though it open its own cafe recently but it's unavoidable that it may also has a large customer base regarding it may do a good advertising before it built. It may bring fresh air for the people in the area. Thus, whether or not Monarch Book can compete with Regal Books is a riddle.In sum, the authors of the argument needs to cites more convincing evidences to support his claim. For example, whether the customers who prefer to go to Monarch Books like to drink coffee there and whether the percentage of the population under age ten in the area can reflect that of the national's. In addition, it's important to know the competition abilities of Regal Books.One, the author overlooks such a question: does the data of the recent nation census accords with the data in this place? Would the sale of children's book has a decline either? It is quite likely that the population under age ten here would have a significant increase, contrary to the national census. What's more, even if there would be a decrease in the percentage of the population under ten here, the author provides no evidence of the sale of children's books. Perhaps parents would pay an increasing more attention to children's education, so they would buy more books for their children. Thus, there would be an increase in the sale of children's books despite of a decline of population of children. Therefore, without detailed and accurate prediction of children's population and sales of children's books in

Page 19: ARG-FINAL

this place, the conclusion is unpersuasive.Two, even if both the population under age ten and sales of children's book would increase, running a cafe needs large amounts of investment, if the manager would have a profit ?If the manager has enough experience to run the cafe successfully?  It is entirely possible that most customers here do not like to have coffee, then the revenue is less than costs. And running a cafe need a great deal of experience and novel ideas, the manager might be incompetent, and this scenario is quite possible. Therefore, without ruling these possibilities, the author cannot conceive me that the café will attract more customers and run success.

And finally, even if the cafe would run success, if Monarch Books is comparable with Regal Books? And if the cafe in Regal Books would attract more customers and run success? The author lacks sufficient information about the state of operation of Regal Books . It is very likely that the  cafe in Regal Books is struggling to survive. Even if it is operating very well, perhaps the situations between two book shops, such as the purchasing power of consumers, have a big difference. So without accounting for the details of two book shops, the author cannot reasonably prove the proposed method will help Monarch Books

Summed up, the recommendation is not on sound reasoning and therefore unconvincing as it stand. To bolster it the author must provide clear and persuasive evidence that the population under age ten and the sales of children's book would increase. That the café will attract more customers and run success, in addition must prove that Monarch Books and Regal Books are comparable.

ARG-8

Grounding on the reason that the dormitories will be inadequate in the future, the author suggests to build more dormitories to meet the housing needs of students, after which he/she concludes that sufficient dormitories would increase students' enrollment at Buckingham. However, after scrutinizing the evidence the author provides, some logical fallacies can be found.In the first place, the author indicates that new dormitories would be more likely to attract students to enroll. But it is worth a second considering whether the condition of dormitories is an important factor to the students who are choosing their university, where they may want better education more than an environment to live in. There may be other determinant facets rather than the condition of dormitories, such as the academic atmosphere of the college, the concrete major the students are interested in, the location and history of the college, so on and so forth. To make the argument more convincing, the author has to provide more information about the indexes students care about while choosing colleges.Though admitting that the condition of dormitories is a determinant to the students, the evidence that the dormitories will be lacking in the next 50 years in this argument is also untenable. The

Page 20: ARG-FINAL

current trends may be a suitable basis of analyzing the next year or the next two years’ situation, yet it is unwise to hastily predict how the situation would be in the long run of time as many as 50 years. Even it may be possible that 50 years later there would be no this college at all. The author should be more objective about the prediction of the dormitory needs in the future or offer more evidence for supporting this argument.It is also unwise to conclude that the college would need more dormitory merely because the rent of apartments has risen in recent years. For one thing, everything changes over time. There may be a crush on the rent of the apartments and the students can afford themselves again. Even if the rent keeps rising still, other ways of tackling this problem is also accessible. For instance, several students can share the rent and they can afford a small part of the sum of the rent. For another thing, the rent rising up off-campus may be less expensive than on-campus. Without detailed evidence about the comparison with both of the rent on-campus and off-campus in the future, no conclusion of the students’ choice could be made.

In sum, more prerequisites should be considered about whether to build new dormitories. It’s precipitous to make such rash decisions without finding out the real factors the students are concerning and the valid evidence that the need of dormitories will indeed rise up. More detailed information should be provided and the author should scrutinize the argument thoughtfully.

In the above statement, the author states that Buckingham college should increase their number

of dormitories. But, the author is stating based on the future trends and not stating about the

current needs. Hence, the above statement requires more reasons to persuade the concerned

person to make a decision about the given memo.

The author states that in 50 years the rate of enrollment will increase. But, he has not mentioned

how many dorms are available for the current students. If students enroll then students will also

graduate and go out of the college making room for new students. The author has not provided

proper statistics about the current enrollment details and number of students graduating each

year. Building new dormitories ahead of 50 years its waste of time and resources, because during

that course of time it is quite possible for other colleges to perform better than Buckingham

college.

The author talks about the average rent but does not give a specific , number of how much it has

risen. If the rent has increased by only a small number than it might not effect students as the

increase will be small. The author also does not give enough details as to how many students

leave on-campus and off-campus; The author does not give any details about the on-campus

leaving expenses so that we can have a comparison and have a clear idea of whether the off-

campus expenses are more than on-campus.

Page 21: ARG-FINAL

Lastly, the author assumes that having new dormitories in the campus attracts more students. The

main priority for any student who wants to attend college is finding a program in which he/she is

interested, checks the tuition fee, city and maybe or may not be the dormitories. But, surely no

student will want to have his education in a college which has new dormitories but old syllabus

and not good faculty. Hence, dormitories maybe the last thing he/she might look up. So,

assuming that having new dormitories will increase the number of students admission is certainly

a misconception and not a valid reason to make a decision about building new dormitories.

Hence, in order to consider the proposal, the director of the student housing should give enough

details to come to conclusion of whether to build a new dormitory or not. Hence, because of the

above reasons, the memo is specious and needs more research and valid reasons.

The Director has stated that a number of new dormitories has to be built as enrollment is growing at Buckingham college and it will be double over the next 50 years. The Director could have given a report of increase in number of students enrolling each year and the total strength of college students that the dormitory accomodates. Even if the college strength doubles, it may not be necessary that all those students would be staying in college dormitories. Students staying with their parents would be coming from their houses. So if the Director had provided a report of how many students out of total college strength stay in college dormitory, it would have helped strengthen the argument.

Secondly, the director has mentioned that the rent of apartments in town has increased recently and that the students would find it difficult to afford to stay there. But director has provided no figures regarding how much the rent has increased and how much a studednt pays per year for staying in college dormitories. If there is only a little difference, two or three students can stay together in an apartment and share the rent. Hence this evidence lacks, which would have again helped in strengthening the argument.

Additionally, Director has mentioned that attractive new dormitories would make prospective students enroll at this college. To strengthen this statement, director could have approached the new students individually and prepared a survey of how many students enroll in a college based on how big and confortable the dormitory is.

Finally, providing a survey of how many studetns are currently stating in the college dormitories and how many new students can be accommodated in the following year could have illustrated the need for building new dormitories. Hence lack of all these evidences weakens the argument and does not make it look like a serious issue to be considered immediately.

ARG-9

Page 22: ARG-FINAL

In this argument the author comes to the conclusion that the next opening of Nature's Way in Plainsville should be very successful. To justify the claim, the author points out that Nature's Way's profit success are most significant in areas where residents lead healthy lives and that Plainsville is such an area due to its local health club popularity. The author also cites Plainsville's fitness-for-life program at schools, which represents a new generation of potential customers. Close scrutiny of this argument, however, reveals that the argument is unconvincing as it relies on several suspicious assumptions.First and foremost, the author unfairly assumes that Nature's Way's profit success is the result of the healthy lives lead by an area's residents rather than some other phenomenon. The arguer ignores a host of other possible reasons for the success of Nature's Way. Perhaps the residents in those areas are financially capable of purchasing more expensive and healthful food and products; or perhaps there are fewer competition stores similar as Nature's Way in those areas. Without ruling out all other possible explanations for the business success of Nature's Way, the author can not convince me that by opening a franchise in another town where people lead healthy lives would guarantee the success of Nature's Way.Secondly, the author's poor assumption that good sales report of running shoes and exercise clothing at Plainsville is the indication of the Plainsville resident's healthy lives is unsupported. The author fails to consider other possible reasons for the good sales of sport apparels. Perhaps they purchase those types of clothes simply because of the refreshing and cool style which they believe would make them look younger and more energetic; or perhaps they order those clothes because of the apparel's comfort material and reasonable price. Similarly, the popularity of Plainsville's local health club might be explained by other reasons as well. It is possible that people go there for fun and friends making rather than health building. And the reason why those weight training and aerobics courses are always full might be the lack of enough training teachers or enough health clubs in Plainsville. Moreover, the author does not provide any information about the geographical background of the customers for the good sale and the health club participants for the club popularity: whether the majority of the customers and participants come from Plainsville or from other places. In addition, even if the residents are fond of exercising, it does not guarantee that they are actually healthy; they might take those weighting classes just becuase they have health problems. Since the author does to account for alternative explanations for the good sale report and health club popularity and fails to provide the geographical background and exact health status of the buyers and health club class-takers, the author can not assume Plainsville's residents lead healthy lives. Finally, the author's assumption on the link between potential customers and the market success of Nature's Way is totally groundless. It is true that the schoolchildren are required to participate the fitness-for-life program, but how much percent of them are going to implement it? Even if these schoolchildren do have exercise at their early age, it does not necessarily imply that they would keep doing this after they grow up, nor does it imply that the same group of schoolchildren would necessarily stay in Plainsville after they graduate. Moreover, the gap between the opening of the next store of Nature's Way in Plainsville and the maturity of the potential customers would very likely leads to the economical failure of Nature's Way's business

Page 23: ARG-FINAL

in Plainsville as there might not be enough customers for Nature's Way before those schoolchildren grow up. Since a store's financial success is a function of both profit and expense, it is also entirely possible that Nature's Way store's cost of obtaining high-quality, health food and other products, or of promoting the new store in PLainsville, might render it unprofitable despite its popularity. Without more information about supply, demand, production costs and revenue, and whether there are enough current customers before the potential customers become available, the author's expectance of potential customers' contribution to Nature's Way store's immediate success by its next opening in Plainsville is highly doubtable.

In sum, the conclusion reached in this argument is invalid and misleading. To make it logically acceptable, the arguer would have to substantiate that an area's residents' health lives is the only reason for Nature's Way store's financial success, that Plainsville indeed has the majority of its residents leading healthy lives and that Nature's Way store does have enough customers before the potential customers become available. Moreover, I would suspend my judgement about the credibility of this argument until the author can provide further information about why Nature's Way stores have their most success in areas with healthy residents, whether Plainsville is indeed such a place and whether the Nature's Way can deal with issue of the gap before potential customers become available.

The school children of Painsville are described as a new generation of potential customers. It is said that they are required to participate in fitness for life programme, but no knowledge is thrown on whether this programme is willingly participated by the students. So the statemen prooves it self skeptic about schoolchildren being healthy in the locality.

The author of this argument states that opening Nature's Way store in Plainsville would be very successful because Nature's Way franchises tend to be most profitable in areas where residents lead healthy lives. The author has made three assumptions and used them as strong points in concluding that opening Nature's Way's next franchise in Plainsville would be successful.

First of all, the author states that Nature's Way franchises are successful in the areas where residents lead healthy lives. The success of the store in those areas may also be because of the absence of other competitive stores that sell health-related products in those areas or because of discounts are promotions that the store proves on every product. The author should have provided some more evidence like surveys of people on why they purchase from Nature's Way in order to prove his assumption true. If, from a survey report, we can conclude that the success of Nature's Way franchises is not because it is in a area where healthy people live, then the argument will lose its basis.

The second assumption is that the residents of Plainsville town lead healthy lives. The author seems to have concluded this from the information collected from Plainsville merchants, the local health club and the weight training and aerobics classes. The local merchants have reported

Page 24: ARG-FINAL

that running shoes and exercise clothing are sold the most always. This may be because wearing sporting cloths may be the latest trend of wearing clothes in the town of Plainsville and hence people buy such clothing. Next, the local health club report states that it has more members than ever and the weight training and aerobics classes are always full. This does are necessarily mean that the people in Plainsville are health conscious. This may also be because most of the people in Plainsville are not healthy and they join in health club and aerobics classes because of their doctor's advice. The author has collected information from various parts of the town, but has not put in any effort in taking a survey from people on why they buy sporting cloths, join health club and physical exercise classes. If the survey report states that people do not take up these activities to stay healthy, as assumed by the author, then the argument cannot conclude that new Nature's Way franchise would be successful in Plainsville.

Finally, the author assumes that school children are new generation of potential customers. The author has supported this assumption with the fact that school children are required to participate in a fitness-for-life program, which emphasizes the benefits of regular exercise at an early age. Just because the school children participate in a fitness-for-life program, one cannot come to the conclusion that they would practice regular exercise. Students participate in fitness program because they are required to do so. The students may not even like taking part in these programs. The author has not provided any stong evidence to prove that the students participate in fitness program to practice exercises regularly and not because their parents or teachers ask them to.

If the above assumptions are proved wrong, the argument does not stand a chance and opening a new Nature's Way franchise in Plainsville may not be successful.

ARG-10

Dr. Karp who is an anthropologist and his student used interview-centered research method to understand child-rearing style of island of Tertia and culture of that. Besides, he and his students’ research outcome is that children in Tertia island talked about their biological parents more than other adults while these children were interviewed by Dr. Karp research team. Therefore, Dr. Karp advocated that the former research Dr. Field’s research result which is children in island of Tertia were raised not by their own biological parents but by an entire village is invalid and Dr. Field’s research method called observation-centered approach is obsolete. In contrast, his research outcome is valid and his research method called interview-centered approach is able to understanding child-rearing traditions of island of Tertia and cultures of that accurately. Dr. Karp’s argument seems sound apparently. However, after think over deliberately, I think Dr. Karp’s argument is not cogent enough. In the following paragraphs, I will point out several fallacies of this argument as follows.

To begin with, Dr. Karp didn’t show us his interview-questions and questionnaires; therefore, we also don’t evaluate his research outcome and prove it valid or invalid. It is possible that children

Page 25: ARG-FINAL

are reared by an entire village and children are curious at their biological parents so that their talked about their biological parents more. Nevertheless, these children regard all adults in Tertia village are ordinary so that they talk those less. In addition, Dr. Karp didn’t tell us how much time children in Tertia Village spent on talking about their biological parent and other adults. Perhaps children in Tertia Village spent more time on talking about their biological parent; likewise they also spent a lot of time on talking about other adults. That is to say, if children in Tertia village didn’t spend much more time on talking about their biological parents rather than other adults, Dr. Karp’s inference that children were reared by their biological parents is totally not convincing enough.

Moreover, according to American psychologist Elton Mayo’s “Hawthorne Effect” theory, people under the observation or interview will follow observers or interviewers’ anticipation to do something or answer the questions which questioner want. As so many academic researchers know, interview research method is more incapable of understand the real situation than observation approach because people always don’t know they are under observation but they already know they are tested and observed under interview conservation. In other words, it is possible that children in Tertia village just wanted to satisfy Dr. Karp and his students and answered the question which Dr. Karp wanted in order to follow Dr. Karp’s anticipation. Thus, Dr. Karp and his research team perhaps got an invalid research outcome and unable to prove their research method which is called interview approach is more appropriate and better than observation approach that was used by former notable anthropologist, Dr. Field.

Furthermore, even though children talked much more about their biological parent under the interview conservation, these interview conservation results are unable to represent the reality that children in Tertia village are nursed by their biological parents because there is no direct causal relationship between interview conservation outcome and child-rearing tradition in island of Tertia. To illuminate the above statement, due to accidental causes like children in Tertia village regarded talking their biological parents as funny affair so that they talked much more in one interview conservation. Besides, it is possible that one child talked a little more about his or her biological parents occasionally and leaded so many children talking about their biological parents because of “bandwagon effect” which are called by political scholars or “conformity” which are called by psychologists. Therefore, if Dr. Karp tried to convince us, he should provide more concrete evidences to us rather than vague inferences.

To sum up, the research method “interview approach” seems inappropriate in investigating child-raising tradition in island of Tertia and probably got a meaningless research outcome that is incapable of proving and inferring anything. In addition, Dr. Karp’s argument over-inferred because his argument was short of direct relationship between his research outcome and child-raising tradition of Tertia village. Lethally, Dr. Karp made a hasty and bold conclusion that interview approach is better than observation approach. On the basis of these fatal fallacies which I presented above, Dr. Karp’s argument is entirely not compelling.

Page 26: ARG-FINAL

Here in this article author is talking about the study of anthropologist in the island Tertia. Author

is considering the study of two anthropologist who visited the Tertia and given their conclusion.

Analysis of one anthropologist says that children of the Tertia reared by entire village rather then

only parent. While the study of another anthropologist says that children of Tertia reared by their

parent rather then village people.

First observer Dr. Field analyze that the children in Tertia were reared by an entire village rather

than by their own biological parents. His this result was observation centered. According to his

observation in the Tertia he concluded this. But we know that mentality and the thinking power

of the every person is different and we can not trust the result came from and single person.

Because the matter which one may ignore would be very important for someone else. More there

is nothing about the way of observation done by Dr. Field is given so we can not trust the result

of his study.

Second time Dr. Karp another Anthropologist visited the Tertia and his analysis was Interview

centered. He concluded from his study that childred in the Terpia reared by their biological

parent only rather then people of village. His this result contradicts the result of the Dr. Field.

But here in the observation of the Dr. Karp there is nothing about the method and way of

analysis is given. For interview centered analysis one has to conduct interview of all people in

exact number and manner in very particular manner. If one take interview of 100 people from

one region of the city and then he conclude his result from that then it’s not going to give the

correct result.

To conclude the result about the Tertian Children, one has to study the tradition in which the

children in the Tertia reared from last few century. In addition the interview of children from all

around the area with interview of village people required to make the conclusion about how

children of Tertia reared. Some other anthropologists say that it required some more research and

interview centered method to analyze how children of Tertia reared. But they did not say

anything about how they should be interviewed and what parameters should be considered in

order to make the study.

So as it stands, authors argument are not compelling. In order to conclude how the children of

Tertia reared one has to make detailed study and research based on both personal observation as

well interview and has to focus on some other related parameters

ARG-11

Page 27: ARG-FINAL

In this argument, the council predicts that the proposed measure, if passed, will result in a significant increase in housing prices in Maple Country. To support this conclusion, the proponents point out that Chestnut Country adopt a similar measure ten years ago ,and its housing prices increased slightly, on the contrary, Pine Country has its housing prices double since they take the restrictions on the development of new residential housing fifteen years ago. This argument is well-presented but far-fetched, for several reasons, this argument fails to provide adequate support for the prediction.In the first place, either the proponents or the opponents commit the fallacy of “false analogy”. The most apparent deficiency is that the argument rests on the assumption that Maple City is analogous to Chestnut Country and Pine Country in all aspects, which is absolutely weak.  To start with, the situations cited in the argument, that is, the housing prices of Pine Country increased slightly as well as of Chestnut Country doubled, was happened in different time period. The variation of prices occurred ten years ago in Chestnut while fifteen years ago in Pine. As time going by, there will be differences in many aspects. There is possibility, which might exist rationally, that the housing price of these two counties are all decrease after adopting the measures mentioned above.  Secondly, comparison among Maple , Pine and Chestnut is not valid, because we all know that two countries could differ from a large amount of aspects such as locations, weather, natural resources and so on, which may definitely effect not only the economy but the culture here. So taking Chestnut and Pine as references to predict the housing price in Maple is not tenable. Thirdly, the council fails to establish the causal relationship between the prices-increase and measure-adopted. The housing prices in Pine and Chestnut might be low initially, there are many factors that contribute to the price increasing, such as the large amount of external population or some other politic measures leads to the increase. Pursuing this line of reasoning, it seems to be the author’s responsibility to list more evidence to elucidate the similarity of Maple and the two countries mentioned above. If not, no basis exist for the comparison. In the second place, even if the flaws cited above is worked out by some ensuing evidence, a critical problem still remains that the assumption that limitation on house supply could definitely results in the significant increase in housing prices is unreasonable. The society is a whole system, a phenomenon cannot appears only because one factor, well, this is the case with housing price. Local economy development as well as the growth of population can also effect the level of prices. Thus supply is just part of those factors, so we cannot take the relationship between supply and price as causal and result for granted. Other than the author provides some evidence to substantiate that it is the limitation of housing supply that leads to the increasing of housing prices, or we cannot predict that the proposed measure, if passed, will result in a significant increase in housing prices in Maple County.

 In a nutshell, it seems precipitous for the author to jump to the conclusion based on a series of problematic premise. This prediction is based on a false analogy, it fails to elucidate that the Maple with Chestnut and Pine is analogous to make comparison. What’s more it only take a little part of effective factors into account and overlooks other crux elements in the variation of

Page 28: ARG-FINAL

housing prices. To make this prediction more plausible, the author should come to grips with the questions mentioned above. Only by enriching the evidence of bolstering the argument could the author put a persuasive prediction

In the maple country, due to over development council has proposed to prevent the development

of existing farmland in the country. In the further section the effects of this restriction on prices

of housing is stated. Proponents of the proposal, on the basis of Chestnut Country example are in

favor of the proposal. Opponents are considering the example of Pine Country and are opposing

the proposal. At the end the current prediction of council is given, according to which proposed

measure will lead to increase in housing prices.

Council of Maple country are concerned with the over development of the country and proposed

to limit the supply of new housing. Firstly is this the only way to prevent the over development

and what are the main reasons for the over development, these questions would have been

answered in order to justify the proposed restriction. Council is assuming that by limiting the

supply of new housing, will affect the price of housing and prices will increase. The facts and

reasons behind the assumptions are not clearly presented in the passage.

Proponents and opponents both are considering the situations which has happened ten and fifteen

years ago in the past and there are possibilities that at present the circumstances are different. So

examples for their assumption are not in the correct time frame. In Chestnut and Pine countries

the population density, geographical structure, demand of new housing, can be different from

Maple country. So these conditions would have been clearly stated to make their assumption

reasonable. The causes of restrictions on the development of new residential housing can also

differs, it might have possible for the protection of harvesting land and to save the land in its

natural state, Chestnut and Pine countries had applied the restrictions.

At the end council has predicted that by applying the proposed restriction will increase in

housing prices. The reasons and evidence behind the conclusion are not mentioned and the stated

examples are proved to be weaken in order to justify the conclusion.Questions like Time frame,

present situations of the Chestnut and Pine countries, causes behind the restrictions should be

answered to provide strong support to assumptions and conclusion

The council of Maple County has predicted that establishing a measure that would limit the supply of new housing will result in a significant increase in housing prices. This prediction was made based on the fact that Pine County adopted restrictions on the development of new residential housing fifteen years ago and its housing prices have doubled. The proponents of this

Page 29: ARG-FINAL

measure have stated that ten years ago, Chestnut County established a similar measure and there was only a little increse in its housing prices. However, there are few questions that need to be answered before we can conclude that the proposed measure will result in increase in housing prises in Mapple County.

The oponents of the measure have cited the fact about increase in housing prices in Pine County since adopting a similar measure. Was there a high demand for houses in Pine County, soon after this measure was adopted, fifteen years ago? We need to know if new industries were opened in and around Pine County after this measure and if this was the reason for high demand of houses there. People would want to stay in places near their office for known advantages like reduced travelling time from office to home and back home. If there were no such industries developed near Pine County, this would confirm that the increase in housing prices is only due to restrictions on the development of new residential housing.

The argument states that restricting new residential housing development resulted in housing prices increasing to double the previous price in Pine County and only a small increase in housing prices in Chestnut County. To go or not go with the conclusion that adopting the measure will result in increased housing prices, we need the answer to two questions: How many houses are there in Maple County, Chestnut County and Pine County? How much demand does each of these places have for houses? Here is why: the housing prices in Pine County may have doubled because there were less number of houses compared to the increasing demand when the measure was adopted, and the housing prices in the Chestnut County may not have increased much on adopting the measure because there was no high demand for houses in in that area and there were more than enough houses for that much demand. Similarly we need to know the number of houses in Maple County at present and the demand from people for the houses in Maple County in order to conclude if adopting this measure would increase housing prices.

Hence, knowing answers to these questions would help us in deciding whether the proposed

measure, if passed, would result in a significant increase in housing prices in Maple County.

ARG-12

In this memo, the dean of Omega university explain about a new procedure that evaluates the teaching effectiveness of the professors. He points out that due to this procedure, there has been an inflation in the students grades. He is of the opinion that this is the reason why Omega graduates have not been successful in getting jobs. However, there are many logical errors in this argument.

Page 30: ARG-FINAL

The author is convinced that the implementation of procedure that evaluates the teaching effectiveness of the professors. However, he has failed to tell us the number of students who participate in this procedure. If the number of students who actively take part in this procedure is very less, this reasoning will be ineffective. It just means that removal of this procedure will have no effect on the grades of students. Also it can be concluded that students earn these grades through their hard work.

The author has unfairly assumed that the students grades have increased due to implementation of procedure. However, he has failed to ignore a host of possible explanations of the same.The quality of students entering Omega university may have improved over the coming years. Also higher admission stds, higher teaching facilities, etc. Without taking these points into consideration, the author cannot convince me that the evaluating procedure can lead to grade inflation.

In addition to this, the author has unreasonably compared the placements of Alpha and Omega University and concluded that Alpha graduates have been more successful in getting jobs. However, he has not taken certain points into consideration. It could be that the number of students may be less and the quality entering Alpha University may be very good. On the other hand,it is possible that Omega University has lot of students . Hence even if many students have been placed, the percentage of students unplaced will be more than Alpha University. The author should also know the quality of companies that Alpha graduates have been placed in. It could be that even though quantity wise Alpha graduates have more placements qualitywise they may be poor.Hence, without much details it is not right to compare Omega and Alpha university.

Summing up, one can conclude that without substantial evidence it is incorrect to conclude that the evaluating procedure is the reason of inflation of students grades. The author should find out how many number of students actively take part in this procedure. Also he must take into other considerations that could be possible for rise in students grades like admission standards and quality of education. Last but not the least, instead of comparing with Alpha University, the

author should find out the quality of placement services in Omega university.

At the first glance, this statement is very convincing that the reputation of Omega University's graduates has been tarnished from a process that encouraged students to evaluate all of their professors. However, after reconsideration, some parts of statement should be substantiated in order to analyze correctly what the real reason of the low employed rate of Omega University's students is.

Page 31: ARG-FINAL

First, author stated that nowadays companies disbelieve in academic performance of students who graduated from Omega University and hence most students have not been successful to get jobs after graduation. However, to make this statement more ponderous, author should also specify the number of unemployed students who just graduated from Omega University before the evaluation proocess have been used. By this information, readers con easily conclude that the low rate of employment in this university results from the evaluation process or not.

Moreover, statement did not give any information about providing majors at Omega University. Probably, all diciplines instructed at Omega Univesity can not meet company's need, thus most students failed in getting jobs. Therefore, to consider under the same basis, statistic data of the number of students at the same faculty in Omega University and Alpha University should be considered.

Finally, author did not provide the information whether evaluation process has also been used at Alpha Univesity or not. Without this major basic background, it is difficult to conclude that graduates at Alpha Univesity have been successful to be hired simply because this university does not use the evaluation procedure.

In short, to make this statement stronger, author should take more information into account. For example, the comparison of the number of unemployed graduates before and after evalution process has been adopted should be shown. Moreover, the same major students at both Omega and Alpha Universities should be used to consider in order to avoid factor about the difference of company's need in both university. Futhermore, author should also confirm that the higher rate of employed graduates at Alpha University resulted from the refutation to use evaluation procedure.

In the Omega University a process of evaluation of teachers by the students was implemented.

The outcomes of that implementation is presented in the further passage. This procedure lead to

proliferate the students grades but these grades were inflated and unable to present the

achievement of the student. The consequences of this is further demonstrated as unsuccess in

securing good jobs. Finally conclusion is given as to terminate the procedure.

'Evaluation of teachers by the students' policy may have been applied to improve the teaching

quality and skills. But here the adverse effect of the implementation is presented. To secure

better evaluation points from the students, teachers might have been started to give the grades to

please the students not to evaluate their achievements properly. This results in significant

increase in grade averages at Omega but that is unjustified.

Here there could have been another reasons behind the increase of average grades, like teaching

quality would have been improved and learning would have been more in that period. So the

Page 32: ARG-FINAL

assumption of grade inflation and reason behind the inflation seems to be weakened. Further it is

noticed that Omega graduates have not been as successful at getting jobs as compare to Alpha

University. There might have been many reasons for the unsuccess of the students like lack of

communication skills, propriety etc. . So here the assumption that grades were unable to reflect

the achievement can not be completely true and it might be one of the reasons for unsuccess of

students in getting the jobs.

For enable the better jobs, prohibition of 'student evaluation of professors' policy is proposed,

which seems to be an immature conclusion. The reasons causing the unsuccess of the students

should be evaluated properly and policies should be finalized accordingly. The implementation

of 'student evaluation of professors' process would have been in favor to improve the teaching

quality and skills of the students

Thirdly, another flaw that undermine the logic of this argument is that the the author assumes

that the Omega’s grades have not been as successful at getting jobs as Alpha’s is the result of

their inflated grade-average is not accept by the market. Nevertheless, no evidence is provide by

the author can justify this assumption. The author fails to take into account other possible

alternatives, such alternative may include some contingencies such as supply and demand or

changes in market. it is entirely possible that the market in this fifteen years is highly insufficient

for industry grades which the Alpha is good at and Omega is not. It is equally possible that

Alpha is a outstanding university in the worldwide and it’s grades are subsequently outstanding.

If it is the case, terminating the procedure alone is far from being enough to make sure that the

Omega’s graduates can be as successful in securing jobs as Alpha’s. Since the author fails to

account for these and other alternative explanation, it is impossible to assess the feasibility of the

recommondation

ARG-13

The newspaper editorial states that the 55 mph speed limit should be restored because the 45 mph speed limit seems ineffective. It uses the argument thate ven with the 45 mph speed limi,t drivers do exceed the speed limit and the accident rate has decreased only slightly. The editorial states that Prunty County should use the methods followed by Buter county since the number of accidents reported there in this past year are 25 percent lower than five years ago. Though, this argument may seem plausible, it has a few flaws that can make its validity debatable.

The reason given for the failed safety effort is that drivers exceed the speed limit. First of all, it is not necessary that an accident has to occur whenever the speed limit is exceeded. For instance, if

Page 33: ARG-FINAL

a person is driving on an empty straight highway at a speed of 150 mph, the chances of an accident occurring are minimal. Accidents on the highway can be due to a number of reasons other than high speed driving. For instance, if the high way has heavy traffic, hair pin turns and steep slopes, then the chances of accidents occurring increase manifold. Fog and low visibility can also cause accidents. Thus, it is unfair to put the blame on drivers exceeding the speed limit.

A comparison with Butler county is warranted only if it has the same topographical layout as Prunty county. If there are geographical differences, then such a comparison would be useless. For instance, if Prunty county is located in a mountainous area and if Butler county is located in a delta, then the structure, type of construction and safety parameters for the highways would be vastly different. Thus, what works for Butler county definitely would not work for Prunty county. Thus, before comapring the two counties, it is important to ensure that they have similar topography.

The example of Butler city is cited where accidents for the past year have decreased by 25 percent as compared to five years ago. But, is this statistic strong enough to decide that the road resurfacing and lane width increasing operations were a success? For instance, if the number of accidents 5 yeas ago was 20, then due to the 25 percent decrease, the number of accidents for the past year would be 15. This points to a decrease in the number of accidents by 5 over a period of 5 years. Thus, per year the number of accidents reduces by one. Suddenly, the 25 percent statistic does not seem very formidable. There could be other reasons for the decrease in accidents. For instance, increased highway patrolling may have lead to vehicular offenders being apprehended and charged a hefty fine. Thus, it is necessary to isolate the exact reasons for the 25 percent decrease. Hence, the 25 percent statistic is not enough to consider as successful the steps taken to improve highways in Butler county.Exceeding the speed limit cannot be the sole reason for highway accidents. The two counties should be compared only after comparing them topogrphically. Also, a 25 percent decrease is simply not enough. The argument would have been strengthened if more plausible reaons were isolated to determine why Butler county had fewer accidents and if a convincingly higher statistic was available.In the passage author has described the attempts taken in Prunty Country to improve the highway safety. The restriction on speed limit was unsuccessful to curtail the number of accidents in the Prunty country. Further Butler Country road improvement techniques are presented along with its consequences on number of road accidents. Arguer asserts that the speed limit in Butler County is 55 mph and the number of accidents reported in the past year were 25 percent lesser than what were reported five years ago. The accident rates on the highway in Prunty county can be reduced by implementing the same kind of road improvement project that Butler county implemented few years ago seems to be the conclusion made by the author at the first glance.

Page 34: ARG-FINAL

Prunty country has applied the speed limit of 45 miles per hour to ensure safety and according to

report it has failed to reduce the number of accidents. Firstly here the time of taking the

observation of number accidents is not mentioned. It would have been possible that report have

been made just after applying the restriction on limit and by that time the policy was not

completely effective. It was observed that drivers were not following the speed limits, so

enforcing vast limitations on the speeding drivers can reduce accident rates by making them pay

high fine for violating such laws could help in reducing the accident rate.

There are another reasons like road conditions, driver driving skills, geographical situation of the

roads etc. which can be major cause of road accidents. So author assumption to claim only speed

factor as the cause of greater road accidents are seems to be weaken here.

Further the Butler country attempts for road improvement project like increasing lane widths,

resurfacing rough highways, and improving visibility at dangerous intersections are mentioned.

Here author is suggesting to implement same kind of improvement project to be applied in

Prunty country also. The argument would hold strong only if the author provides evidence by

considering all the elements by comparing and contrasting between both counties performance in

reducing the accident rates on highways. Implementation and success of this type of projects

strongly depends on the country's geographical location and to emulate the project policies of

Butler country, Prunty country should have same geographical conditions.

So, author should present more evidence and facts to make his assumption and conclusion to

more stronger. Reasons behind the accidents in Prunty country, locations and other measures of

both the countries should be clearly mentioned to decide the road improvement policy for the

Prunty country.

Also, Butler county could have enforced strict rules on wearing helmets and wearing seat belts and following speed limits and heavy fines for people who don't follow these rules. If the decrease in number of accidents is due to one of the above reasons, then this would weaken the argument.

ARG-14

In the passage author has try to propound a relation between sleeping hour of executives and

their success profile . Author tries to conclude the fact from a recent survey and propose that

Page 35: ARG-FINAL

company should hire those people who sleep less. Though his claim may well have merit, the

author presents a poorly reasoned argument, based on several questionable premises and

assumptions, and based solely on the evidence the author offers, we cannot accept his argument

as valid.

Primary issue with the author’s reasoning lies in his unsubstantiated premises. The arguer

provides no assurances that those 300 people on which the argument depends is statistically

reliable. Lacking information about the number of people surveyed by a single surveyor it is

impossible to assess the validity of the result. For example, it may be possible the survey result is

only applicable for Mentian advertising executives only and it is not possible to extrapolate those

result to other companies.In the second place, the argument is based on a false analogy. The

arguer simply conclude that less than six hour sleep will result profit margin in company .

However, the author provides no evidence to support that this is indeed comparable other than a

single survey . May be one company is doing field business and other is making money by

simply selling software via internet . The author`s premises, the basis for his argument, lack any

legitimate evidentiary support and render his conclusion unacceptable.While the author does

have several key issues in his argument’s premises and assumptions, that is not to say that the

entire argument is without base. Like the devotion to company is indeed necessary for the

prosperity of company . But that has a little connection with less than six hour sleep.

In sum, the author’s illogical argument is based on unsupported premises and assumptions that

render his conclusion invalid. If the author hopes to change his reader’s minds on the issue, he

would have to largely restructure his argument, fix the flaws in his logic, clearly explicate his

assumptions, and provide evidentiary support. Without these things, his poorly reasoned

argument will likely convince few people.

Imp:

The argument is trying to correlate the numbers of hours a person sleeps to the efficiency of his or her work using some facts, which might seem probable and logical at the beginning. But if the argument is scrutinized then, one can find that the base of the claim is very misleading. Firstly, the author has pointed about a survey which he claims that is recent and he also states that the survey has rated 300 female and male executives of Mentian Advertising Company. The text hasn't mentioned anything about the time of the survey because recent might include 2 months or even a year. And the author hasn't mentioned anything about the age or the experience of the executives who were rated in the survey. It is possible that the survey included only young people and since they are young, they might have the energy to work even with lesser hours of

Page 36: ARG-FINAL

sleep. Sleep is a necessary for every human and many scientists have deduced that 6 hours of sleep is nominal for every human being because it rejuvenates the human body and it makes the person fresh and zestful.

Secondly, the author has claimed that the executives who have slept for less than six hours have contributed to the success for the firm. But the author has failed to take into account neither the number of years of experience of the company nor the projects that were at their hands of the company. If the company is well established then they would have lots of experience and the executives would also know how to handle crunch situations. The company might also have a bigger project in their hands and this single project might have led to increase in their success of their firms.

Thirdly, the author hasn't mentioned about the number of working hours of these executives during the survey period. If the executives were asked to work for more than 12 hours per day then, it is possible that they would complete their works swiftly leading to higher success rate. The author has also mentioned about the higher profit margin of their company has increased during the survey period and he claims that these 300 executives are responsible for it. But the profit margin is not set by these executives as the quotation is sent only by the higher officials. It is possible that these 300 executives include only low grade workers and these workers would only have completed their tasks on time or even much earlier. But correlating the profit margin to the work of the executive is not logical.

Finally, the author has strongly inferred that for a company to be successful they should hire only people who sleep for less than 6 hours a day. So, if the company keeps this as one of the selection criterion then many of talented people might end up losing the job. This would in turn affect the company's growth because if they are rejecting a person solely based only on the criterion that he sleeps more than 6 hours a day, then they are making a foolish mistake. The author doesn't think about the people who are lethargic in their work but they might be very much efficient in their own ways. As Bill Gates has once said, "I would hire a lazy person because he would find an easy way to solve a problem". So sleep is not the only factor that a company should take in consideration while hiring a person. So, the text is flawed and these points would undermine the text completely.

ARG-15

Author has mentioned a memorandum from the business manager of Happy Pancake House restaurant. It is talking about the replacement of the butter with margarine. There are main two points are discussed here. One is effect of replacing the butter with margarine to the customer

Page 37: ARG-FINAL

and another one is customers power of distinguishing the butter and margarine.Here it is stated that two percent of the customers have complained and other 98 percent of the customer are satisfied. But one thing is that it is not necessary that all the customer who are not satisfied with the change make the complain. Some customer may be unhappy with the change but they avoided making complain.Some of the customer coming first time may think that the quality of the butter in Happy Pancake House restaurant is of such a bad only so deciding not to visit the restaurant again they left the restaurant. So such people may be out of count causing the result wrong that 2 percent customer unsatisfied.Then author is adding the comment from servers that a number of customer who ask for butter do not complain when they are given margarine instead butter. But it doesn’t mean that customer can not distinguish the different between butter and margarine. As people are using butter very often so at least some of the customer would surely distinguish the difference between both. It might that customer don’t like arguing and making issue so they might not have complained.In the last paragraph author is criticizing the distinguishing power of customer. Author us saying that either they could not distinguish both or they refer margarine and butter as the same think. To decide in better way one has taken review from each of the customer and analyzing the reason could have stated that whether they are satisfied with the change or not.Reporting from the server may be false and they might have said such a comment for their personal benefit or due to some other issue. So as it states, author arguments are not compelling. To strengthen the argument manager needs to take the review and can also put feedback form for their experience in the restaurant. Such a result can let manager to decide whether customer are satisfied or not. So based on the arguments given in the main paragraph one cannot say any thing about customer distinguishing power or their satisfaction regarding the replacement of butter with margarine

In this memorandum, the manager points outthat the change of replacing butter with the margarine in the southwesternUnited States has little impact on our customers. To corroborate this claim,the manager cites that only 2 percents of the customers have complained thechange. In addition, the manager cites that the customers do not distinguishbutter from margarine or they use the term ‘butter’ to refer to the either butteror margarine. At first glance, the memorandum appears to be somehow plausible,close scrutiny of these evidences, however, reveals that it tends littlecredible support for the manager’s claim.

To begin with, citing the surveys of thecustomers, the manager concludes that the change has little impact on ourcustomers. It is not clear, however, the scope and validity of that survey. Maybethe samples are not so representative to whole people, or the capacity ofsamples is too small to represent. If they just had 50 people investigated,only one of them complained about the placement, the conclusion that an averageof 98 people out of 100 is happy with the change is not cogent. The managerjust generalizes the survey without enough rationality. More persuasive,practical and professional information need to be support the conclusion of

Page 38: ARG-FINAL

thesurvey. 

Apart from that, evidence that a number ofcustomers who ask for the butter do not complain when they are given margarineinstead. Yet this might not be the truth for a variety of possible reasons.Perhaps the servers did not report the truth. If the requests of every customerare satisfied, the servers had to afford additional work. Thus they reportedthat customers hold indifferent attitude towards butter and margarine. Or it islikely that the customers did not express their real feelings. That they didnot voice it does not mean they did not hate it.

A third problem with the argument is thatit is unfair to assume that either these customers do not distinguish butterfrom margarine or they use the term ‘butter’ to refer either butter ormargarine. It is entirely possible that the customers know the differencesbetween butter and margarine, but they do not want to lose temper just withoutthe butter. Or perhaps the customers decide not to go Happy Pancake HouseRestaurant once more, thus they do not have incentives to make a complain. 

To sum up, the manager fails to validatethe conclusion that the change of replacing butter with the margarine in thesouthwestern United States has little impact on our customers. To solidify theargument, the manager should provide more concrete evidence to demonstrate theconclusion. In addition, the manager would have to rule out the above mentionedpossibilities that would undermine the claim.

ARG-16

The argument that the city government should invest in riverside recreational facilities due to a sure increase in the use of the river for water sports is not entirely logically convincing since it ignores certain crucial assumptions.

First, the argument assumes that the residents of Mason City don’t use extensively the river for recreational activities due to the poor quality and terrible smell of the river’s water, yet it does nothing to explain such assumptions. Could it be that the river is inadequate in terms of shape and depth for boats to safely navigate or simply that the river is home to several dangerous animal species such as alligators, bull sharks and snakes making it unsafe for swimmers? The argument might have been straightened by showing that the main reason why the river is not used for water sports is that it is in squalid conditions.

Second, the argument assumes that the plans to clean up the river are more than a mere political promise that might never be carried out if the government in power changes meanwhile. The argument never addresses the extent to which the river will be cleaned and more importantly when it is scheduled to be cleaned. The argument could have been straightened by mentioning the exact procedure that is intended to be carried out and a guarantee that the river would be

Page 39: ARG-FINAL

cleaned by a specific date.

Finally, even if the river safe and adequate for water sports and that it is guaranteed that it will be cleaned appropriately by a specific date, the argument still depends on the assumption that the river will remain clean and odourless in the future. What if the river is cleaned but the source of pollution remains? It would be a question of time before it would become dirty again and its usage for watersports will wane slowly back to its original level. The argument could be strengthened by saying that the river’s water quality would be regularly verified to assured that it is clean and odourless.

Thus, the argument is not completely sound. The conclusion that it would be appropriate to invest in river side facilities because plans to clean the river will unquestionably result in an increase of its usage for watersports in is supported by very little evidence and relies on many unaddressed assumptions.

The author assumes that more money in the budget will increase use of riverside recreational facilities by Mason city residents. This may not be true. It would be necessary to consider what other recreational activities outranked the water sports and if these results were statistically significant. Issues of the age groups that were surveyed, the different wealth groups and even education level and even marital status as all these factors may affect the use of any recreational facilities including water sports.

For example it may be that the Mason City residents may not engage in these activities because they lack skills to swim or row boats. Though they desire to do these activities they are not able to engage in water sports and thus engage in other activities. 

It also may be that water sports are expensive. One must own a boat or hire a boat in order to boat or raft. One will need to pay an entrance fee to go swimming, buy a costume and floatation device to swim. If these weighed against sports that are not costly e.g. running or cycling. A bicycle is much cheaper than a boat and so one may choose to cycle than boat and they may choose to run because there is no entrance fee

The smell that the residents complain about may not be due to dirt but by the rivers water composition. High concentrations of chemicals such as sulphur may give the water a smell and are also toxic to the human population and the water animals. 

In this argument, as per surveys Mason city residents rank water sports among their favourite

recreational activities, but deciding based on the surveys alone is wrong because surveys are

usually done on few thousands of people, comparing this with lakhs of people residing in Mason

city, it is very less and one more point that has to be considered is, in any city different cultured

Page 40: ARG-FINAL

people reside in different places, if so this survey is based on a persons from a particular locality

then it is completely wrong to assume that entire city residents favourite is water sports.

In this argument, author has stated that, for years there have been complaints from residents

regarding the river's water quality, hence state has recently announced plans to clean up the river.

It will definitely be appreciated by the city residents but it is wrong to claim that use of river for

water sports will increase because people may have complained, since the water was dirty and

they could not use the water for any household purpose or even enjoy the beauty of river.

Another mistake the author has made is, he has said that city government should devote more

money in this year's budget to riverside for recreational facilities. Cleaning the river is the

important but devoting more money for recreational facilities, without knowing whether, it will

be useful to people or not is wrong. Government has to conduct a survey on a larger scale and

include different cultured people and then based on the results of survey they can decide whether

to alot more money for it or not.

Summing it up, trusting the surveys is to be appreciated but without blindly deciding based on

the surveys, they have to first study the survey like how many people has taken part in the

survey, has it involved different cultured people from different environments and based on all

this, if the survey has involved reasonable amount of people then they can decide based on the

survey.

The nexus made between the Mason City surveys about recreational activities and need of allotment of funds for cleaning the river is specious. It is a general tendency that the local people will vouch for clean river. The argument alone is not sufficient to conclude as there are various assumptions made as the argument progresses.

It's never mentioned that the surveys made within the Mason City covers the people living near the river or away from it. It neither mentions the age groups who participated in the surveys nor the idea that those people who like water sports participated in the surveys. Also, the fact that the surveys show that water sports are among the favorite recreational activities is inconsistent with the fact that the river is rarely used for these pursuits and a little of city's budget is employed in maintaining riverside recreational facilities.

The details about the complains are also not revealed, it may be possible that complains are logged by same people, numerous times who may be living near the river or away from it. The people may be complaining due to basic tendency to have a cleaner and pollution free river. There is no fact mentioned in the argument which states that people want the river cleaned for recreational activities.

Page 41: ARG-FINAL

The source of river pollution is not known, if the river is polluted by artificial wastes from the city then it can be cleaned. The water pollution may be caused due to some geographical features as underground mines or leakage of sulphur gas from underground deposits which may be the reason of fowl smell, in such cases the water pollution problem cannot be solved.

Also, it's very uncertain that whether the funding program can be carried out successfully because it depends on political factors such as agreement between parties and associations.

The proclivity of the whole mass of the population towards water sports is unknown. Even after the river is cleaned, there is no certainty that people will devote their time in water sports.

An apt reason for the cleaning of the river and investing the funds on recreational facilities can be attracting tourists and thus improving the overall living standard of the population. It may also answer any health concerns of the people. The argument made by the author is supported by weak premises. The fund invested on the recreational facilities can be of no use if people do not turn up for recreational activities.

ARG-17

In the passage author has proposed strategies to increase listener numbers of WWAC radio

station. Author has claimed the increase in population in the prescribed area, which is mainly

from people moving here after their retirement. So author has emphasized to switch the music

format according to resident's likings. Prediction is made by the author that cause of decrease in

local sales of recorded music is due to lack of interest in music. So as a conclusion

implementation of news and talk format is suggested to increase the sales.

It is stated that WWAC radio station is using rock-music format. An observation is made that

there is decrease in listener numbers. But reasons behind this declination is not presented clearly.

There can be several causes like presence of other multimedia entertainment sources like

internet, television, computer. It may be possible that people are attracting towards these new

sources and that's why there is a decrease in number of listeners in WWAC radio station.

Author has presented the evidence of increase in population of the reported area and most of

them are retired people. Assumption is made that may be these senior citizens do not like the

current rock-music format , so there is a decrease in number of listeners. Here the assumption

seems to weaken and unreasonable. The ratio of these retired people to young people is not

mentioned over here. It might have been possible that listeners number is decreasing due to low

quality of sound or due to improper network.

Page 42: ARG-FINAL

Due to observed declination in sales of recorded music author has predicted the reason that

people have limited interest in music. The prediction has proved to be unsatisfactory . There is

possibility that people would have been using other sources of music or people would have been

facing problems with recording quality and versatility. Author has suggested to change the music

format to news and talk format which is not justifiable.

So in order to provide strong support to argument, author should mention the details of WWAC

radio stations schemes and their music quality. The population ratio of adults to senior citizen

and their likings should be examined to increase the number of listeners. The owners of WWAC

should mingle both the style: the rock-music and news and talk format to provide versatile

formats.

In the argument above, the author concludes that the WWACradio station should change it current rock-music format to a news and talk format so that to reverse a decline in listener numbers.To support the conclusion, the author points out that although the population in the listening area has grown,that growth mainly relates to the retired people who immigratesfrom other place.If the radio station want to make their lisreners of these new residents,they should follow their tastes to choose a music format.Based on the declining of the local sales of recorded music ,the author illuminates that the new residents lack of interest in music.However,these all do not constitute a logical argument.The evidences fails to prove the conclusion ina logical perspective.

The threshold problem is the increasing population lacks of evidence to indicate that they has resulted mainly from the retired people without a pinpoint statistic.The growth number of a city should be calculated by the number of the death and birht,not only the different between immigration and emmigration.The new residents who immigrate cannot decide whether the population is up or down.Even if the population has increased for the maily reason of the retired new arrival, the passage does not insure when the retired people made the immigration.If they had come to the area for many years , but the decliningof the listeners is occurred recently, how can we find out the population be increased by this group of people?

Another problem is if the growth of the population is really caused by the resident who newly be there, that is hardly to decide that they will be the mainly listeners of the WWAC radio station in a new music form.For instance, if a large number of the new residents lack of interest in listening radio but perfer watching TV to listen music or get the daily news, the change of WWAC will has no help to reverse the declining listener.However,even the new residents are crazied to listen radio music or other things ,the radio station can not find out the reason of the decline is about the retired people's age which may be an age perfer news than music.Although many people in a retired age like news rather than rock-music,the author should have proved whether THIS group prefer.

Page 43: ARG-FINAL

The author also hastily assumes that music is being lossing popular because of the local sales of recorded music.Firstly the recorded music is only a kind of music store. With the development of the technologythe lover of music can collect their favors in their computer or mp3,moble phone,I Pad,I Pod and so on . Secondly,in the perspective of eviromental protection,recording may be some kind of wasting materia things.Fruthor more,the recorded music will be heavier than an I Phone. 

Finally,the conclusion about the changing of radio program format which from rock-music format to news and talk format also in an absence of some strongly evidence.If the population is increased by the retired people who will be the mainly listeners of WWAC radio station, what is the relationship between this group of people to the customers of recorded music?The author fails to interpretate that may be the mainly customers of recorded music are that group of residents.Even all the above problems can be proved, there is still a puzzle floating on the argument. How can the author be sure that the listeners of WWAC will like the new format which includes news and talk? We have to predict the cause before any dicision being accomplished.If the listeners decrease in a repidly speed which more serious than before,the radio station makesolution plans better.

To sum up,if the author want to prove the decision of the radio format changing, the absence evidence shoud be added to make the passage be logically acceptable.

ARG-18

The argument states that the late-night news program of a television channel devoted more times

towards national news. As a result those audiences having interest in local and weather channel

loose interest on that television channel. The advertising agencies also have canceled their

contract with the television channel in their late night shows. So to prevent such revenue loss and

to retain interest of audiences in their late night shows author has suggested some measures .The

argument, however, relies on assumptions for which there is no clear evidence. It is infested with

poor reasoning and thus, it is rather specious.

Firstly, author did not mention any reason behind their migration from local and weather news to

national news. It may be an experiment to increase their business. Another serious question does

arise when the complaints were received from viewers and there were no urge to change the

program pattern then. It may be possible that the television channel may be in transition from

local to national television channel. So to getting more audiences at national level the channel

may be devoting more time to national news rather than local news. It may be one of the reasons

why the television channel neglect those complain received from their local viewers. Thus we

need to know the proper reason to understand the argument.

Page 44: ARG-FINAL

Secondly, the author shows that the local business canceled their contracts with the television

channel. Author did not mention the exact reason for that action of advertising company. There

may be other possible reason like dispute with those advertisement companies or many more

reason possible. Thus the assumption provides weak support.

Thirdly, author did not mention about the decrease in the number of the viewers. They may

complain but do they adapt the change later? Author assumed that there are decrement in the

audience number but does not provide any information to support that assumption. Without these

details, the assumption is seriously weakened.

Finally , reverting again to the local and weather news does not guarantee that the television

channel will get their old audiences back . It may be possible that in between these time those

audiences found another suitable channel accordingly their choices. So the assumption of regain

the revenue again may be unreasonable.

Thus, the arguments fails to consider the above mentioned facts and therefore, it is rendered

unconvincing to the reader. Had these details been mentioned along with supporting evidence, it

would have appeared much more compelling.

The business manager of the TV station have stated that the amount of weather and local news programs in the late-night news program should be restored to former level in order to improve the amount of program viewer and enhance the advertising income losted through. However, throughout his arguing process, it is not logical enough for him to carry out such conclusion.  

First of all, the manager lacks the clarification on the proportion of programs viewers making a complaint. He only mentioned the complaints are concentrated on teh coverage of weather and local news, but he did not mentioned how many viewers have made such complaints. It is possible that merely 1% of the total program viewers have made such complaints while the rest 99% were satisfied with the coverage or may even thinking the coverage was too much. Therefore, to make the evidence convincible, the total number of complaints amongst the total number of program viewers should be stated in order to prove that the majority of viewers do concern about the low coverage of weather and local news. 

Additionally, the manager did not stated that the cancelling of advertising contracts is related to the low coverage of weather and local news. It is not logical to claim that A is the cause of B merely because B happened right after A, which is exactly the same arguing method that the manager has used. It is possible that there are other issues that led to the cancellation of advertising contracts, such as the lacking of funds of the companies which were advertising and they are not able to advertise anymore, or the station has increased their fee of advertising that

Page 45: ARG-FINAL

those companies are not able to afford anymore. To link the cancellation of advertising contracts with the low coverage of weather and local news programs, the simplist way would be to organise a meeting with the companies who wish to cancel their contracts. If they do claim that they are to leave because the number of program viewers have decreased due to the low coverage of weather and local news and they find their advertising ineffective, it would prove that the manager's decision is reasonable.

Lastly, the manager did not mention the cost of making such variation and whether it is financially feasible. Although it may seem easy on the television to make a change of programs from broadcasting national news to weather and local news, the efforts require to make such switch would be enormous. To begin with, extra journalists camera men and their equipments are required for local news. Meanwhile, those staff distributed on various places in the country for distributing national news may require to relocate. All of these actions will cause expenditures to the station. Therefore, assuming it is already proven that the low coverage of weather and national news is the cause of viewer complaints and cancellation of contracts, the manager should still execute a detailed financial report in order to prove that it is a feasible and profitable decision.

To sum up, the manager have made a inappropriate conclusion that the restoration of local news and weather programs could attract more viewer and improve the advertisment revenue because his/her argument is not very logical. With the supplement evidences suggested in the paragraph, the manager could make a more reasonable approach.

ARG-19The passage given here contains some of well-supported and well-presented argument, but not well-reasoned. In the argument, the station manager of KICK in Medway has recommended for inclusion of more call-in advice programs in an attempt to gain a larger audience share in its listening area. This recommendation is inspired by the success of another radio station WCQP in Rockville, which adopted verisimilar regimes for the station and reported inclination in their audience. Manager believes that such programs are appreciated by many radio listeners and hence if they include more call-in advice programs, they will able to allure many audiences towards their radio station. Semblance of the argument is presented in such manner that it seems very reasonable and veracious at first glance, but delve yourself a bit more and we realize that argument is based on very scanty and insufficient information.

Firstly, argument has failed to provide quantified and exact data for the inclination noticed in the WCQP radio station in Rockville. Though, it says that its share of radio audience has significantly increased in listening area, judging anything from such uninformative statement is not sensible enough to make any decision. The WCQP station had started call-in advise programs two years ago, and since then it is possible that population growth in listening area has been increased. Apparently, number of listeners will increase if the area population is increasing.

Page 46: ARG-FINAL

Such insufficient data does not lead to any concrete conclusion. 

Secondly, writer has stated that a nationwide survey indicates that listeners are actively showing interest in call-in advise programs. But how many people were interviewed? How many states were surveyed? and what is the ratio of those people who showed interest in such programs to those who did not show any positive response for such shows? Moreover, what are the odds for success and failure if call-in advice programs are included in radio station services? Unfortunately, none of the questions above are answered in the passage presented here, hence it is not viable to rely upon any superficial conclusion.

Scrutinizing the whole paragraph keeping every minor factor in your mind, argument contains some inadequate and sleazy information. It fails to convince readers to accede with its content due to following reasons.a) Insufficient information about WCQP radio station and ratio of audience with respect to population growth in listening area.b) Groundless reports and lack of quantified data of the survey made.c) Supposititious conclusion made upon incomplete and scanty analysis.

First of all, the evidence of radio station WCQP is outdated. The author assumes that past performance is a guarantee of future results, without considering that listeners may get bored with call-in advice programs after a while. Now, listeners may like other programs. In addition, listeners in Rockville and Medway areas are two different populations and they may have different preferences. The author needs to conduct another survey in Medway to make sure that current listeners in Meday area like call-in advice programs. Even though, the author mentions that many radio listeners are interested in call-in programs based on nationwide survey, the author does not specify when the national survey was conducted. The result of the survey would be less convincing if it was conducted years ago. Nor does the survey reveal under what circumstances listeners said they prefer call-in advice programs. It is possible that the survey was conducted when call-in advice programs was just introduced, and people are interested in these programs because they are curious. 

Further, the increase in the audience may be attributed to the possibility that radio station WCQP was one of few radio stations at that time having call-in advice programs. Thus, it would have less competitors and find it easier to attract more listeners. Therefore, in order to make the decision whether KICK should include more call-in advice programs or not, KICK should conduct a research to figure out how many radio stations have call-in advice programs in Medway area, and whether these radion stations are popular or not. If there are too many radio stations having call-in advice programs in Meday area already, how KICK could compete with these existing radio stations and stand out if KICK decide to join the competition. 

Overall, in order to bolster the conclusion, the author should provide more recent statitistics to

Page 47: ARG-FINAL

show that current listeners in Meday area like call-in advice programs. In addition, KICK should also conduct a research to find out how many radio stations in Meday area have call-in advice programs, and then re-evaluate the chance of success.

ARG-22

In the given passage a scenario of Groveton College is presented for the noticeable reduction of

cheating cases. Groveton College has implemented an honor code and it is assumed by the author

that due to this there is a significant decrease in cheating cases. The policy under the honor code

is explained and further its consequences are presented. Groveton students opinion for the honor

code is mentioned following by author's conclusion that is to implement similar code in all

colleges and universities.

According to honor code student is tempted to agree not to cheat in their academic exams and if

they find any student to do so, it is their responsibility to inform this to faculty member. But here

this policy seems to be not so strict and details of the codes are inadequate to understand it

properly. If a student has caught in cheating case what punishment will be there ?, how much

strict the rules and regulations are, these type of questionable premises should be answered

properly to strengthen the argument.

In an honor code implemented system, teachers are assigned to closely monitor the students and

their activities. But it is difficult to monitor the class with an enormous number of students by a

singe teacher. So technologies like CCD cameras can be implemented for this purpose. Whether

Groveton College is using such type of policies or not, is not described.

The faculties of Groveton College has reported that cases of cheating have dropped from twenty

one to fourteen in recent five years. Here author is assuming that this honor code adaption is

responsible for this. This assumption seems to be unreasonable and weaken. There can be several

causes which would have been responsible for this like fear of hidden cameras, castigation for

cheating cases and many more. It is reported that majority of students said that due to honor code

they are less prone to cheating, which can be said by students under pressure of teachers and

seems to be unjustifiable.

Page 48: ARG-FINAL

At the end author has concluded with the suggestion for the other colleges to implement the

honor code. More explaination and technical details of the code should be mentioned in order to

make the conclusion strong and justifiable.

1. Most institutions tell their students not to cheat, but still have problems with cheating. Even if the statistics were accurate, there could be other factors that reduced the cheating. More lenient professors or more interesting subject material, more opportunities for student-teacher contact or perhaps even smaller class size could be the primary factor which can account for less reported cheating under the new honor code system.2. Author's primary fallacy however is that he or she confuses reported cases of cheating with total instances of cheating by ignoring the fact that cheating may have even increased overall, but may just be harder to detect. Less teacher supervision certainly accounts for this fact.3. Many students may not report cheating even if they see it. Many students feel that witnessing other students cheating does not really affect them. Furthermore, students may not feel that they have support even if they were to report the cheating. If a student reports cheating, the cheater could simply deny that he or she was cheating. Now the burden of proof is on the student who is reporting the cheating. It seems unlikely that most students would go to the extreme measures of acquiring sufficient evidence necessary to convince scholastic officials that cheating has taken place. 4. Another reason that students may not report cheating is that it can be seen as socially disfavorable. Many students care about how they are perceived by others. No one wants to be seen as the tattle-tale or teacher's pet.5. Furthermore, even if we were to assume that the cases reported represent entirely all cases of cheating, it remains to be proven that the rate of cheating has decreased. If the school's population is cut in half due, then it follows that the rate of cheating may actually decrease as well given that there are fewer students to engage in cheating. Likewise, the author does not indicate how the cases of cheating were reported. Realistically, all cases of cheating may still be reported by teachers alone, having no students report cases of cheating that they observe. 6. Finally, the authors last statement of support needs to be substantiated. Many students may claim that they would be less likely to cheat with the honor code in place than without, but the students may simply be lying. What guarantees that the students are being honest? Perhaps the surveyor was a recognizable school official and the students were just telling the official what he or she wanted to hear. The survey does not even indicate that the students surveyed went to Groveton College, many may not attend the school. Also, the survey only gave two alternatives, a more reliable survey would question only Groveton College and give students the option of choosing between the new honor code system vs. the traditional system.

Page 49: ARG-FINAL

To begin with, by citing this comparison, it is obviously that the author(or the author hopes the reader) takes for granted the following assumption: The decrease in number of cheating cases reported is due to the honor codes. But the author fails to account a number of alternative explanations. The college might underwent a lot of changes over the course of five years. For instance, the admission standard might be enhanced and the new students, due to their relatively good record before, would volunteerly reject to cheat academically. Moreover, fewer cases reported does not necessarily lead to fewer cases occured. The reporting procedure might become ineffective during these years. Without considering and excluding these possible scenarios, the author cannot convince me of his reasoning.

Another problem with this argument is the cited survey. The argument provides no assurances to the reliability of the survey. In other words, can the respondents represented all the students in Groveton College? How many students participated in this survey? Without providing detailed information concerning the above questions, it might well be that the students who never cheated in exam were more inclined to response to the survey, or perhaps only 1 percent of the students in GC participated in this survey. Any of the events, if true, would serve to undermine the validity of this survey.

Even assuming that the author can substantiate all the foregoing assumptions, he cannot conclude the same approach would work for other institutions. In order to substantiate this, he should provide clues to show that all these institutions are comparable or similar in every aspects. Otherwise, he just cannot make his recommendation convincible.

In sum, I have no doubt that the author is in his/her best intention to help reduce the cheating rate in those institutions by proposing this "honor codes" approach, but unfortuantely, I have to say, his evidence has failed this intention.

ARG-24

The author predicts that the number of headaches are sure to decline by basing his research on a twenty year old study which found a correlation between consumption of salicylates and decline in headache reports.

The author however fails to mention the possible effects of salicylates that results in decline of headaches. The decline of headaches could have been due to various factors. It could be possible that people under study experienced frequent headaches in the nascent stages of study due to work pressure which declined over a period of twenty years as they steadied into their jobs. The decline in headaches could also be due to increased use of other medicines such as aspirin.

Similarly, the increase in salicylates could be due to a number of other reasons such as the

Page 50: ARG-FINAL

quality of the product which was being consumed. Another reason for rise in consumption could be an increased dosage of salicylates by the food processing industries in order to preserve their products for a longer period of time. The author does not mention that everyone under study consumed the same quality and quantity of salicylates over the entire period of study.

To justify the findings, author must first deduce the effect of salicylates on human body when consumed in mild quantities, large quantities and excessive quantities. The author should also carry out a survey whereby random people are selected and provided with identical diet and fixed quantity of salicylates and physical regime. The author must also provide valid proof that increasing the consumption of salicylates does help in reducing the headaches. Unless the author backs his/her prediction with some logical reasoning and facts, the prediction cannot be held true

In this argument, the author predicts that the residents of Mentia will suffer fewer headaches, for

more salicylates will be used as additives for food. To illustrate this, the author argues that

salicylates is of the same kind as aspirin and cites a recent study showing the correlation between

commercial use of salicylates and a steady decline in the average number of headaches reported

by study participants. However, the author's prediction is not as grounded as it seems, he or she

fails to provide enough information for the argument.

To begin with, the evidence provided that salicylates belongs to the same chemical family as

aspirin doesn't lend scant support to the effectiveness in treating headaches, particularly the

residents of Mentia. Although salicylates may be similar to aspirin in some chemical properties,

these two medicines can still differ in medical use, just as graphite and diamond, which

subordinate to the carbon family but have different physical properties and are applied to

distinctive use. Moreover, there are many reasons accounting for the headaches Mentia residents

suffer, like the polluted air or unhealthy way of living, for example always staying up late at

night. Without ruling all these possibilities, the author cannot convince me that the salicylates are

the right treatment for headaches.

Additionally, there are some vital shortcomings in the study mentioned by author, which

seriously compromise its validity. Firstly, since many food have already contained salicylates,

the researcher should conduct the experiment on two groups of people, one group have

salicylate-contained food while the other have food containing no salicylates at all. In this way,

one can clearly observe whether salicylates really treat headaches or not. Besides, the author

didn't provide sufficient information about the decline in reported number of headaches. It is

Page 51: ARG-FINAL

quite possible that not all the participants report their headaches. In order to reach reliable

conclusion, the researchers should take measures to refine the experiment.

Finally, even if salicylates really can cure headache, it doesn't necessarily require more

salicylates added into food. For one thing, many foods are naturally rich in salicylates as the

author mentioned; for the other thing, it is quite possible that a slight amount of salicylates may

work properly on headaches, while the overdose of salicylates may create undesirable side-effect

on human body. Without showing enough evidence that how much precisely salicylates can

work on headache, one cannot make hasty prediction that the more salicylates, the fewer

headaches.

From the reasons discussed above, the author may not see the results as predicted, unless more

information about salicylate’s chemical properties, effectiveness as a medicine and improvement

in the experiment can be made. In order to evaluate the predicted results, all these questions

should be answered.

ARG-25

The facts given by the developers seem to support the recommendation that a local jazz club be built in Monroe. But by strictly speculation can we find some defects about the ratiocination given by the developers. In following we will present these specific evidences to determine whether the club behoove to be invested.

First, mentioned in the application, the fact that over 100,000 people have attended the annual jazz club hold in Monroe is not enough to support that Monroe residents are fond of jazz music, for the following reasons: not all 100,000 people are Monroe residents, and there may be a majority of them accumulating from different places to attend the jazz club, in which case we cannot claim that many people living in Monroe enjoy jazz music. Besides, it was last summer that 100,000 people, whatever it is a large number, went to the festival, which suggests it is not always like last summer that over 100,000 people attended the festival, or a large group in last summer is a special example. However, whether 100,000 is a large number is not for sure, given that the tend of the population attending the annual festival and the total population in Monroe remain unknown to us. In these case, we suggest that the developers present more specific data addressing the tend and the population of Monroe and thus make a more deep analyse. 

Second, the application includes a fact that several well-known jazz musicians live in Monroe. The writer assume them being the evidence of the love of jazz from residents in Monroe. It may make sense because those musicians might choose a place where fan group

Page 52: ARG-FINAL

are in large population as their home. However, that they live there for the other reason, for example, the beautiful view, or little pollution, is always reasonable. It follows that jazz musician's living there should not become the evidence to argue that most residents of Monroe like jazz music. The write should investigate the relevance between these two events before regarding it as a support of building a jazz club in Monroe.

Still, the radio on weekday, mentioned in the application, might not be the evidence of the affection of jazz music among residents of Monroe. People who like this channel may not be in a large number, and 7 P.M on weekends is a period when most people finish their work back to home. If, coincidentally, there is a lack of channels airing at that time, large amount of residents may have to pick this one airing jazz music. In addition, the nationwide study indicating a worth of $1,000 per year spending on jazz entertainment, which is listed at the last part of the application, does not reflect on a specific place like Monroe, where people may not pay that much for their jazz entertainment. Moreover, that does not lead to a profit for the considered jazz club in that we are still not clear whether the payment worth the cost of building a club. In a nutshell, we recommend that developers take every situation into account, considerably; to make a rational decision is based on an exhaustive analyse regarding all kinds of issues.

All in all, as recommended above, the evidence on the cost of building a club, whether it will call local residents' attention, and whether a majority of people living in Monroe love jazz should be given, to determine whether the plan to open a jazz club is worthy

The argument states that the city of Monroe requires a jazz club because jazz is very popular in

the city and that the proposed club, the C-Note, would be a tremendously profitable enterprise

and would have the local market all to itself. Moreover, several renowned jazz musicians live in

Monroe and over 100,000 people attended Monroe's annual jazz festival last summer. Hence, the

developers have decided to establish the club for which they require a small-business loan. The

agrument, however, relies on assumptions for which there is no clear evidence. It is infested with

poor reasoning and thus, it is rather specious.

Firstly, the argument states that jazz is popular in Monroe and that the nearest jazz club is 65

miles away. Had jazz been really popular in Monroe, there would be at least a couple of

renowned jazz clubs within the vicinity of each other established in the city already. However,

this is not the case and thus, the statement seems unconvincing. Secondly, it makes a really broad

assumption that it would be a tremendously profitable enterprise and would have the local

market all to itself without providing the necessary substantive details - How many people, on an

average, attend the nearest jazz club that is 65 miles away? How many people among them hail

Page 53: ARG-FINAL

from Monroe? How many musicians perform on a daily basis? What is the annual turnover?

Without these details, the assumption is seriously weakened.

Secondly, it states that over 100,000 people attended Monroe's annual jazz festival last summer.

Well, there is no proper evidence to support the statement. It is highly possible that out of these

100,000 people, only a fraction of them were from Monroe and a majority of attendees were

from the other parts of the nation or from the various parts of the world, for that matter. It also

states that several established musicians live in Monroe and that the highest-rated radio program

in Monroe is 'Jazz Nightly'. How many of these musicians actually perform on the show? It is

very much likely that the majority of performers are from the other parts of the country.

Thirdly, it states that a nationwide study indicates that the typical jazz fan spends close to $1,000

per year on jazz entertainment. The statement makes an unconvincing generalization. Just

because a typical jazz fan in the country spends $1,000 on jazz entertainment does not mean that

they belong to Monroe. Additionally, there is no mention of the number of "typical" jazz fans in

the country.

Thus, the arguments fails to consider the above mentioned facts and therefore, it is rendered

unconvincing to the reader. Had these details been mentioned along with a supporting evidence,

it would have appeared much more compelling.

A national study shows that typical jazz fan spends almost 1000 on jazz entertainment, which again, is a piece of weak evidence. First, a national study does not precisely reflect the situation of Monroe, where people might virtually spend none on jazz. Plus, we do not know where this average of 1000 goes. It might be spent on jazz CDs or other instruments, but according to the argument, it is more likely to be used for an annual trip to Monroe for that jazz festival, say, money for the road trip, food, accommodation and tickets to the festival, or maybe souvenir for friends and family.

The developers try to substantiate the argument that jazz is considerably popular in Monroe, however, with groundless evidence, unfortunately. They argue that over 100,000 people attended the annual jazz festival held in Monroe last year which gives readers an illusion that Monroe has a huge jazz fan base. However, it is not necessarily the case. These people might have come from other cities in the country; or that last year’s jazz festival was sponsored by a certain charity group which aimed for raising fund for AIDS research, which attracted people who were committed to charity and humanity work, not necessarily jazz. We just do not know more concrete details of the festival. 

Several well known jazz musicians living in Monroe is not sufficient enough to demonstrate

Page 54: ARG-FINAL

jazz is popular here. Some think it might be the “jazz atmosphere” that attracts them over; however, this phenomenon can also be interpreted in another way: for pronounced stars, they may want to avoid being chased by fans, and that they want an ordinary life as anyone else; if Monroe is not a place where fans congregate, it might be a perfect city for famous jazz musicians to live in, which in a way counteracts the developer’s argument.

ARG-27

The argument put forward by the editorial of the newspaper plans to introduce a bicycle lane in

order to ease the traffic flow and reduce the rush-hour traffic. But the argument is not supported

by valid evidence and the information presented in the argument leads to lots of questions that

must be answered in order to validate the sound reasoning of the argument. The recommendation

to add a bicycle lane can be implemented only if it's efforts are proven to yield the beneficial

results without an iota of doubt.

First of all, the editorial quotes that the commuters complain of the increased rush-hour traffic

between suburbs and the City center, solely attributing them to the increase in commuting time.

What about other conditions necessary to increase in commuting time? Are the roads in proper

condition so as to ease the traffic flow? Do the motorists travel in the required speed to reach the

destination in time and do they start early in time so as to make up their trips successful? If the

motorists start late and if they happen to put the blame on traffic, then the reason for introducing

a new lane becomes invalid. Hence a thorough analysis for the increase in commuting time must

be made at the first instant.

Secondly, the author assumes that the widening of the highway can be facilitated by an

additional lane of traffic? But, one has to consider if the widening of the highway will ever be

economically feasible? Even if the highway is widened, will it serve the purpose of reducing the

rush-hour traffic between the suburbs and the city center, which is the main reason for which the

expansion work is performed? These questions if unanswered will severely mar the efficacy of

the argument. Additionally, the argument seems to indulge in a faulty analogy wherein the author

compares the effects on Blue highway with that of the Green highway. Do both highways

ecnounter the same number of motorists? Were the type of vehicles that passed through both of

the highways of same nature comparable to each other? Furthermore, it must be considered to

Page 55: ARG-FINAL

what extent did the addition of lane to Green highway worsen the traffic? If the worsening factor

is negligible, then it is least likely that it will lead to severe consequences in the current

implementation with Blue highway.

The opponents of the motive of widening the highway are quoting an alternative proposal to add

a bicycle lane. But is adding of bicycle lane, the only possible alternative? If there exists several

alternatives, they ought to be weighed against each other before reaching a conclusion. Also, the

author seems to commit the mistake of "Post hoc Ergo Propter Hoc" fallacy wherein he comes to

a conclusion that bicycle lane is beneficial because many area residents are keen bicyclists. The

interests of the people to bicycle has less to do with the traffic encountered in the highway.

Because, some may prefer to use bicycles only in the evening hours, the case is that the newly

constructed lane will be vacant most of the time. Hence, how interested are the bicyclists to use

tha lane during peak hours must be given due attention before planning to implement the

suggestion.

Above all, the important factor to be considered is the rate of growth of population in the cities

using Blue highway. If the population grows at a tremendous rate, then it is least likely that the

addition of a bicycle lane will serve to relieve the problems encountered during rush-hour traffic.

The age-groups of the residents using the Blue highway must also be taken into consideration.

This information will help the evaluators of the new policy to determine how many people will

exactly be using the bicycle lane and the Blue Highway. Hence a variety of other factors must be

listed out to decide whether adding a bicycle land is necessary to ease the traffic flow or if any

other alternatives can be explored.

Thus, the proposal of constructing a bicycle lane has to answer a variety of questions like: the

rate of increase in rush-hour traffic, opinion of the citizens concerning the expansion of highway,

feasibility of constructing an additional highway, comparison between Blue highwat and Green

highway, other alternatives for constructing the highway, the extent to which the bicycle lane

will be put to use. Only if these questions are answered, can we come to a conclusion regarding

the efficiency of the newly proposed recommendation.

First of all, the author implies that addition of a motor lane to the current highway is not a good solution, because this proposal resulted in a worsening of traffic jams on Green Highway. However, it is fundamentally true that the result of one case does not always predict the result of another case. In this case, we need to know what caused the worsening of traffic jams after widening Green highway, and whether Blue Highway has the same

Page 56: ARG-FINAL

problems as Green Highway does or not. If the worsening of traffic on Green highway is because drivers tend to drive faster after addition of a lane, then, governments could still consider adding a motor lane with a more stringent speed limit. If the worsening of traffic jams on Green Highway is coincidental, or completely irrelevant to adding a new lane, for instance most of these accidents are due to drunk driving, then, Blue Highway can still consider adding a motor lane, because the worsening of traffic has nothing to do with adding a new motor lane.

ARG-30

The author contends plausibly in this argument that they will have sufficient money to fund city

facilities and programs used mainly by adults, due to a prediction of increasing population of the

adults in the city. However, this argument remains flawed based on a lack of evidence of the

author's omission of validity of the reasons he quotes after a close scrutiny.

First off, the threshold problem is that the fact that last year's birth-rate was only one-half that of

five years ago could expound on nothing related with the trend of birth-rate. For example, it's

possible that the birth-rate last year was higher than those of two years ago to four years ago.

That is to say, perhaps the birth-rate of five years ago was highest during the five years. Thus the

author should pay a close heed to the evidence of the all five years' birth rate indicating a gradual

decrease in the city, otherwise it's not possible to predict the decrease of the birth rate of next

year, let alone that of the next decade.

Moreover, even if the birth rate could keep sliding through ensuing evidence, this argument still

stays suspicious owing to the casually concluded relationship between the number of students

enrolled in their public schools and the birth rate in the next decade. As is universally known, a

school may never confine itself into enrolling merely local students. In this case, it's hard to

determine the number of students enrolled in their public schools could be predicted based on the

statistic of the birth rate in this city. Therefore, reduction of budget for education could result in

serious problems pertaining to all students attending schools, which ought to be a basic right for

every citizen in this city as well as in any other location. Unless we are provided with evidence

concerning reassurance of controls of students outside the city, this argument scarcely convinces

anyone that the number of students could decrease along with the birth rate.

Page 57: ARG-FINAL

Besides, even though the foregoing problems may be solved within short time, the author falsely

assumes that the athletic playing fields as well as other recreational facilities are solely devised

for students. As a matter of fact, a great many adults all over the world are sports fans, and it

might prove to be necessary for them to exercise within the schools 'borders in addition to the

facilities and programs used primarily by adults, due to the athletic playing fields and

recreational facilities within the schools' grounds. This assumption could only be true in a

situation where the adults in this city are so different from ones outside the city that either they

almost don't value the athletic playing fields for daily use, or they are too lazy to take exercise.

The problem presented above bear no relative evidence in this argument, thereby casting doubts

about the soundness of the final prediction.

In retrospect, the author omits to present a series of evidence directly related with the ability of

convincing of this argument. A more sound argument should include evidence as the statistics of

every year's birth rate, the control over the number of students outside the city, and the adults in

this city are reluctant to use the athletic playing fields and other recreational facilities. Only then

could the author claim unequivocally that they could focus mainly on funding the city facilities

and programs solely used by adults through convincible evidence demonstrating an increase of

the number of adults in this city

In the second place, even if the birthrate does be decreasing and the number of students enrolled will soon decline, it is unreasonable to make a assertion that they can safely reduce the funds for education and athletic playing fields and other recreational facilities. On one hand, to some extent the number of students have nothing to do with the funds for education since it is possible that people may pay more attention to the education because they recognize that education is of significance in the competitive society in the future, thus it may call for a large funds for education and those recreational facilities. On the other hand, adults also needs education, for example, someone whose major was e-commerce finally works as a salesman and he has to learn something about marketing. Therefore, without the strong evidences concerning that they can really safely reduce funds for education and those recreational facilities, the recommendation seems invalid.

Finally, even if they can safely reduce funds for education and those recreational facilities, the author unfairly assumes that the measure will be effective to gain the goal to have sufficient money to fund city facilities and programs used primarily by adults. However, it may not be the case. There is a possibility that the funds budgeted for education and athletic playing fields and other recreational facilities were originally a small part that even if reducing funds on those fields there has little effect on getting enough fund for city facilities

Page 58: ARG-FINAL

and programs used primarily by adults. Therefore, without the information concerning the weight of budgets for each field, the recommendation seems unconvincing. 

In conclusion, in order to better evaluate this recommendation, the author needs to offer more information concerning the recent years' particular birthrate, the strong evidences to prove that they can reduce funds for education and those recreational facilities safely and the weight of budgets for education and athletic playing fields and other recreational facilities.

ARG-22

In the given passage a scenario of Groveton College is presented for the noticeable reduction of

cheating cases. Groveton College has implemented an honor code and it is assumed by the author

that due to this there is a significant decrease in cheating cases. The policy under the honor code

is explained and further its consequences are presented. Groveton students opinion for the honor

code is mentioned following by author's conclusion that is to implement similar code in all

colleges and universities.

According to honor code student is tempted to agree not to cheat in their academic exams and if

they find any student to do so, it is their responsibility to inform this to faculty member. But here

this policy seems to be not so strict and details of the codes are inadequate to understand it

properly. If a student has caught in cheating case what punishment will be there ?, how much

strict the rules and regulations are, these type of questionable premises should be answered

properly to strengthen the argument.

In an honor code implemented system, teachers are assigned to closely monitor the students and

their activities. But it is difficult to monitor the class with an enormous number of students by a

singe teacher. So technologies like CCD cameras can be implemented for this purpose. Whether

Groveton College is using such type of policies or not, is not described.

The faculties of Groveton College has reported that cases of cheating have dropped from twenty

one to fourteen in recent five years. Here author is assuming that this honor code adaption is

responsible for this. This assumption seems to be unreasonable and weaken. There can be several

causes which would have been responsible for this like fear of hidden cameras, castigation for

cheating cases and many more. It is reported that majority of students said that due to honor code

Page 59: ARG-FINAL

they are less prone to cheating, which can be said by students under pressure of teachers and

seems to be unjustifiable.

At the end author has concluded with the suggestion for the other colleges to implement the

honor code. More explaination and technical details of the code should be mentioned in order to

make the conclusion strong and justifiable.

1. Most institutions tell their students not to cheat, but still have problems with cheating. Even if the statistics were accurate, there could be other factors that reduced the cheating. More lenient professors or more interesting subject material, more opportunities for student-teacher contact or perhaps even smaller class size could be the primary factor which can account for less reported cheating under the new honor code system.2. Author's primary fallacy however is that he or she confuses reported cases of cheating with total instances of cheating by ignoring the fact that cheating may have even increased overall, but may just be harder to detect. Less teacher supervision certainly accounts for this fact.3. Many students may not report cheating even if they see it. Many students feel that witnessing other students cheating does not really affect them. Furthermore, students may not feel that they have support even if they were to report the cheating. If a student reports cheating, the cheater could simply deny that he or she was cheating. Now the burden of proof is on the student who is reporting the cheating. It seems unlikely that most students would go to the extreme measures of acquiring sufficient evidence necessary to convince scholastic officials that cheating has taken place. 4. Another reason that students may not report cheating is that it can be seen as socially disfavorable. Many students care about how they are perceived by others. No one wants to be seen as the tattle-tale or teacher's pet.5. Furthermore, even if we were to assume that the cases reported represent entirely all cases of cheating, it remains to be proven that the rate of cheating has decreased. If the school's population is cut in half due, then it follows that the rate of cheating may actually decrease as well given that there are fewer students to engage in cheating. Likewise, the author does not indicate how the cases of cheating were reported. Realistically, all cases of cheating may still be reported by teachers alone, having no students report cases of cheating that they observe. 6. Finally, the authors last statement of support needs to be substantiated. Many students may claim that they would be less likely to cheat with the honor code in place than without, but the students may simply be lying. What guarantees that the students are being honest? Perhaps the surveyor was a recognizable school official and the students were just telling the official what he or she wanted to hear. The survey does not even indicate that the students surveyed went to Groveton College, many may not attend the school. Also, the survey only gave two alternatives, a more reliable survey would

Page 60: ARG-FINAL

question only Groveton College and give students the option of choosing between the new honor code system vs. the traditional system.

To begin with, by citing this comparison, it is obviously that the author(or the author hopes the reader) takes for granted the following assumption: The decrease in number of cheating cases reported is due to the honor codes. But the author fails to account a number of alternative explanations. The college might underwent a lot of changes over the course of five years. For instance, the admission standard might be enhanced and the new students, due to their relatively good record before, would volunteerly reject to cheat academically. Moreover, fewer cases reported does not necessarily lead to fewer cases occured. The reporting procedure might become ineffective during these years. Without considering and excluding these possible scenarios, the author cannot convince me of his reasoning.

Another problem with this argument is the cited survey. The argument provides no assurances to the reliability of the survey. In other words, can the respondents represented all the students in Groveton College? How many students participated in this survey? Without providing detailed information concerning the above questions, it might well be that the students who never cheated in exam were more inclined to response to the survey, or perhaps only 1 percent of the students in GC participated in this survey. Any of the events, if true, would serve to undermine the validity of this survey.

Even assuming that the author can substantiate all the foregoing assumptions, he cannot conclude the same approach would work for other institutions. In order to substantiate this, he should provide clues to show that all these institutions are comparable or similar in every aspects. Otherwise, he just cannot make his recommendation convincible.

In sum, I have no doubt that the author is in his/her best intention to help reduce the cheating rate in those institutions by proposing this "honor codes" approach, but unfortuantely, I have to say, his evidence has failed this intention.

ARG-24

The author predicts that the number of headaches are sure to decline by basing his research on a twenty year old study which found a correlation between consumption of salicylates and decline in headache reports.

The author however fails to mention the possible effects of salicylates that results in decline of headaches. The decline of headaches could have been due to various factors. It could be possible that people under study experienced frequent headaches in the nascent stages of study due to work pressure which declined over a period of twenty years as they steadied into their jobs. The decline in headaches could also be due to increased use of other medicines

Page 61: ARG-FINAL

such as aspirin.

Similarly, the increase in salicylates could be due to a number of other reasons such as the quality of the product which was being consumed. Another reason for rise in consumption could be an increased dosage of salicylates by the food processing industries in order to preserve their products for a longer period of time. The author does not mention that everyone under study consumed the same quality and quantity of salicylates over the entire period of study.

To justify the findings, author must first deduce the effect of salicylates on human body when consumed in mild quantities, large quantities and excessive quantities. The author should also carry out a survey whereby random people are selected and provided with identical diet and fixed quantity of salicylates and physical regime. The author must also provide valid proof that increasing the consumption of salicylates does help in reducing the headaches. Unless the author backs his/her prediction with some logical reasoning and facts, the prediction cannot be held true

In this argument, the author predicts that the residents of Mentia will suffer fewer headaches, for

more salicylates will be used as additives for food. To illustrate this, the author argues that

salicylates is of the same kind as aspirin and cites a recent study showing the correlation between

commercial use of salicylates and a steady decline in the average number of headaches reported

by study participants. However, the author's prediction is not as grounded as it seems, he or she

fails to provide enough information for the argument.

To begin with, the evidence provided that salicylates belongs to the same chemical family as

aspirin doesn't lend scant support to the effectiveness in treating headaches, particularly the

residents of Mentia. Although salicylates may be similar to aspirin in some chemical properties,

these two medicines can still differ in medical use, just as graphite and diamond, which

subordinate to the carbon family but have different physical properties and are applied to

distinctive use. Moreover, there are many reasons accounting for the headaches Mentia residents

suffer, like the polluted air or unhealthy way of living, for example always staying up late at

night. Without ruling all these possibilities, the author cannot convince me that the salicylates are

the right treatment for headaches.

Additionally, there are some vital shortcomings in the study mentioned by author, which

seriously compromise its validity. Firstly, since many food have already contained salicylates,

the researcher should conduct the experiment on two groups of people, one group have

Page 62: ARG-FINAL

salicylate-contained food while the other have food containing no salicylates at all. In this way,

one can clearly observe whether salicylates really treat headaches or not. Besides, the author

didn't provide sufficient information about the decline in reported number of headaches. It is

quite possible that not all the participants report their headaches. In order to reach reliable

conclusion, the researchers should take measures to refine the experiment.

Finally, even if salicylates really can cure headache, it doesn't necessarily require more

salicylates added into food. For one thing, many foods are naturally rich in salicylates as the

author mentioned; for the other thing, it is quite possible that a slight amount of salicylates may

work properly on headaches, while the overdose of salicylates may create undesirable side-effect

on human body. Without showing enough evidence that how much precisely salicylates can

work on headache, one cannot make hasty prediction that the more salicylates, the fewer

headaches.

From the reasons discussed above, the author may not see the results as predicted, unless more

information about salicylate’s chemical properties, effectiveness as a medicine and improvement

in the experiment can be made. In order to evaluate the predicted results, all these questions

should be answered.

ARG-25

The facts given by the developers seem to support the recommendation that a local jazz club be built in Monroe. But by strictly speculation can we find some defects about the ratiocination given by the developers. In following we will present these specific evidences to determine whether the club behoove to be invested.

First, mentioned in the application, the fact that over 100,000 people have attended the annual jazz club hold in Monroe is not enough to support that Monroe residents are fond of jazz music, for the following reasons: not all 100,000 people are Monroe residents, and there may be a majority of them accumulating from different places to attend the jazz club, in which case we cannot claim that many people living in Monroe enjoy jazz music. Besides, it was last summer that 100,000 people, whatever it is a large number, went to the festival, which suggests it is not always like last summer that over 100,000 people attended the festival, or a large group in last summer is a special example. However, whether 100,000 is a large number is not for sure, given that the tend of the population attending the annual festival and the total population in Monroe remain unknown to us. In these case, we suggest that the developers present more specific data addressing the tend and the population of Monroe and thus make a more deep analyse. 

Page 63: ARG-FINAL

Second, the application includes a fact that several well-known jazz musicians live in Monroe. The writer assume them being the evidence of the love of jazz from residents in Monroe. It may make sense because those musicians might choose a place where fan group are in large population as their home. However, that they live there for the other reason, for example, the beautiful view, or little pollution, is always reasonable. It follows that jazz musician's living there should not become the evidence to argue that most residents of Monroe like jazz music. The write should investigate the relevance between these two events before regarding it as a support of building a jazz club in Monroe.

Still, the radio on weekday, mentioned in the application, might not be the evidence of the affection of jazz music among residents of Monroe. People who like this channel may not be in a large number, and 7 P.M on weekends is a period when most people finish their work back to home. If, coincidentally, there is a lack of channels airing at that time, large amount of residents may have to pick this one airing jazz music. In addition, the nationwide study indicating a worth of $1,000 per year spending on jazz entertainment, which is listed at the last part of the application, does not reflect on a specific place like Monroe, where people may not pay that much for their jazz entertainment. Moreover, that does not lead to a profit for the considered jazz club in that we are still not clear whether the payment worth the cost of building a club. In a nutshell, we recommend that developers take every situation into account, considerably; to make a rational decision is based on an exhaustive analyse regarding all kinds of issues.

All in all, as recommended above, the evidence on the cost of building a club, whether it will call local residents' attention, and whether a majority of people living in Monroe love jazz should be given, to determine whether the plan to open a jazz club is worthy

The argument states that the city of Monroe requires a jazz club because jazz is very popular in

the city and that the proposed club, the C-Note, would be a tremendously profitable enterprise

and would have the local market all to itself. Moreover, several renowned jazz musicians live in

Monroe and over 100,000 people attended Monroe's annual jazz festival last summer. Hence, the

developers have decided to establish the club for which they require a small-business loan. The

agrument, however, relies on assumptions for which there is no clear evidence. It is infested with

poor reasoning and thus, it is rather specious.

Firstly, the argument states that jazz is popular in Monroe and that the nearest jazz club is 65

miles away. Had jazz been really popular in Monroe, there would be at least a couple of

renowned jazz clubs within the vicinity of each other established in the city already. However,

this is not the case and thus, the statement seems unconvincing. Secondly, it makes a really broad

Page 64: ARG-FINAL

assumption that it would be a tremendously profitable enterprise and would have the local

market all to itself without providing the necessary substantive details - How many people, on an

average, attend the nearest jazz club that is 65 miles away? How many people among them hail

from Monroe? How many musicians perform on a daily basis? What is the annual turnover?

Without these details, the assumption is seriously weakened.

Secondly, it states that over 100,000 people attended Monroe's annual jazz festival last summer.

Well, there is no proper evidence to support the statement. It is highly possible that out of these

100,000 people, only a fraction of them were from Monroe and a majority of attendees were

from the other parts of the nation or from the various parts of the world, for that matter. It also

states that several established musicians live in Monroe and that the highest-rated radio program

in Monroe is 'Jazz Nightly'. How many of these musicians actually perform on the show? It is

very much likely that the majority of performers are from the other parts of the country.

Thirdly, it states that a nationwide study indicates that the typical jazz fan spends close to $1,000

per year on jazz entertainment. The statement makes an unconvincing generalization. Just

because a typical jazz fan in the country spends $1,000 on jazz entertainment does not mean that

they belong to Monroe. Additionally, there is no mention of the number of "typical" jazz fans in

the country.

Thus, the arguments fails to consider the above mentioned facts and therefore, it is rendered

unconvincing to the reader. Had these details been mentioned along with a supporting evidence,

it would have appeared much more compelling.

A national study shows that typical jazz fan spends almost 1000 on jazz entertainment, which again, is a piece of weak evidence. First, a national study does not precisely reflect the situation of Monroe, where people might virtually spend none on jazz. Plus, we do not know where this average of 1000 goes. It might be spent on jazz CDs or other instruments, but according to the argument, it is more likely to be used for an annual trip to Monroe for that jazz festival, say, money for the road trip, food, accommodation and tickets to the festival, or maybe souvenir for friends and family.

The developers try to substantiate the argument that jazz is considerably popular in Monroe, however, with groundless evidence, unfortunately. They argue that over 100,000 people attended the annual jazz festival held in Monroe last year which gives readers an illusion that Monroe has a huge jazz fan base. However, it is not necessarily the case. These people might have come from other cities in the country; or that last year’s jazz festival was sponsored by a certain charity group which aimed for raising fund for AIDS research, which attracted

Page 65: ARG-FINAL

people who were committed to charity and humanity work, not necessarily jazz. We just do not know more concrete details of the festival. 

Several well known jazz musicians living in Monroe is not sufficient enough to demonstrate jazz is popular here. Some think it might be the “jazz atmosphere” that attracts them over; however, this phenomenon can also be interpreted in another way: for pronounced stars, they may want to avoid being chased by fans, and that they want an ordinary life as anyone else; if Monroe is not a place where fans congregate, it might be a perfect city for famous jazz musicians to live in, which in a way counteracts the developer’s argument.

ARG-27

The argument put forward by the editorial of the newspaper plans to introduce a bicycle lane in

order to ease the traffic flow and reduce the rush-hour traffic. But the argument is not supported

by valid evidence and the information presented in the argument leads to lots of questions that

must be answered in order to validate the sound reasoning of the argument. The recommendation

to add a bicycle lane can be implemented only if it's efforts are proven to yield the beneficial

results without an iota of doubt.

First of all, the editorial quotes that the commuters complain of the increased rush-hour traffic

between suburbs and the City center, solely attributing them to the increase in commuting time.

What about other conditions necessary to increase in commuting time? Are the roads in proper

condition so as to ease the traffic flow? Do the motorists travel in the required speed to reach the

destination in time and do they start early in time so as to make up their trips successful? If the

motorists start late and if they happen to put the blame on traffic, then the reason for introducing

a new lane becomes invalid. Hence a thorough analysis for the increase in commuting time must

be made at the first instant.

Secondly, the author assumes that the widening of the highway can be facilitated by an

additional lane of traffic? But, one has to consider if the widening of the highway will ever be

economically feasible? Even if the highway is widened, will it serve the purpose of reducing the

rush-hour traffic between the suburbs and the city center, which is the main reason for which the

expansion work is performed? These questions if unanswered will severely mar the efficacy of

the argument. Additionally, the argument seems to indulge in a faulty analogy wherein the author

Page 66: ARG-FINAL

compares the effects on Blue highway with that of the Green highway. Do both highways

ecnounter the same number of motorists? Were the type of vehicles that passed through both of

the highways of same nature comparable to each other? Furthermore, it must be considered to

what extent did the addition of lane to Green highway worsen the traffic? If the worsening factor

is negligible, then it is least likely that it will lead to severe consequences in the current

implementation with Blue highway.

The opponents of the motive of widening the highway are quoting an alternative proposal to add

a bicycle lane. But is adding of bicycle lane, the only possible alternative? If there exists several

alternatives, they ought to be weighed against each other before reaching a conclusion. Also, the

author seems to commit the mistake of "Post hoc Ergo Propter Hoc" fallacy wherein he comes to

a conclusion that bicycle lane is beneficial because many area residents are keen bicyclists. The

interests of the people to bicycle has less to do with the traffic encountered in the highway.

Because, some may prefer to use bicycles only in the evening hours, the case is that the newly

constructed lane will be vacant most of the time. Hence, how interested are the bicyclists to use

tha lane during peak hours must be given due attention before planning to implement the

suggestion.

Above all, the important factor to be considered is the rate of growth of population in the cities

using Blue highway. If the population grows at a tremendous rate, then it is least likely that the

addition of a bicycle lane will serve to relieve the problems encountered during rush-hour traffic.

The age-groups of the residents using the Blue highway must also be taken into consideration.

This information will help the evaluators of the new policy to determine how many people will

exactly be using the bicycle lane and the Blue Highway. Hence a variety of other factors must be

listed out to decide whether adding a bicycle land is necessary to ease the traffic flow or if any

other alternatives can be explored.

Thus, the proposal of constructing a bicycle lane has to answer a variety of questions like: the

rate of increase in rush-hour traffic, opinion of the citizens concerning the expansion of highway,

feasibility of constructing an additional highway, comparison between Blue highwat and Green

highway, other alternatives for constructing the highway, the extent to which the bicycle lane

will be put to use. Only if these questions are answered, can we come to a conclusion regarding

the efficiency of the newly proposed recommendation.

Page 67: ARG-FINAL

First of all, the author implies that addition of a motor lane to the current highway is not a good solution, because this proposal resulted in a worsening of traffic jams on Green Highway. However, it is fundamentally true that the result of one case does not always predict the result of another case. In this case, we need to know what caused the worsening of traffic jams after widening Green highway, and whether Blue Highway has the same problems as Green Highway does or not. If the worsening of traffic on Green highway is because drivers tend to drive faster after addition of a lane, then, governments could still consider adding a motor lane with a more stringent speed limit. If the worsening of traffic jams on Green Highway is coincidental, or completely irrelevant to adding a new lane, for instance most of these accidents are due to drunk driving, then, Blue Highway can still consider adding a motor lane, because the worsening of traffic has nothing to do with adding a new motor lane.

ARG-30

The author contends plausibly in this argument that they will have sufficient money to fund city

facilities and programs used mainly by adults, due to a prediction of increasing population of the

adults in the city. However, this argument remains flawed based on a lack of evidence of the

author's omission of validity of the reasons he quotes after a close scrutiny.

First off, the threshold problem is that the fact that last year's birth-rate was only one-half that of

five years ago could expound on nothing related with the trend of birth-rate. For example, it's

possible that the birth-rate last year was higher than those of two years ago to four years ago.

That is to say, perhaps the birth-rate of five years ago was highest during the five years. Thus the

author should pay a close heed to the evidence of the all five years' birth rate indicating a gradual

decrease in the city, otherwise it's not possible to predict the decrease of the birth rate of next

year, let alone that of the next decade.

Moreover, even if the birth rate could keep sliding through ensuing evidence, this argument still

stays suspicious owing to the casually concluded relationship between the number of students

enrolled in their public schools and the birth rate in the next decade. As is universally known, a

school may never confine itself into enrolling merely local students. In this case, it's hard to

determine the number of students enrolled in their public schools could be predicted based on the

statistic of the birth rate in this city. Therefore, reduction of budget for education could result in

serious problems pertaining to all students attending schools, which ought to be a basic right for

every citizen in this city as well as in any other location. Unless we are provided with evidence

Page 68: ARG-FINAL

concerning reassurance of controls of students outside the city, this argument scarcely convinces

anyone that the number of students could decrease along with the birth rate.

Besides, even though the foregoing problems may be solved within short time, the author falsely

assumes that the athletic playing fields as well as other recreational facilities are solely devised

for students. As a matter of fact, a great many adults all over the world are sports fans, and it

might prove to be necessary for them to exercise within the schools 'borders in addition to the

facilities and programs used primarily by adults, due to the athletic playing fields and

recreational facilities within the schools' grounds. This assumption could only be true in a

situation where the adults in this city are so different from ones outside the city that either they

almost don't value the athletic playing fields for daily use, or they are too lazy to take exercise.

The problem presented above bear no relative evidence in this argument, thereby casting doubts

about the soundness of the final prediction.

In retrospect, the author omits to present a series of evidence directly related with the ability of

convincing of this argument. A more sound argument should include evidence as the statistics of

every year's birth rate, the control over the number of students outside the city, and the adults in

this city are reluctant to use the athletic playing fields and other recreational facilities. Only then

could the author claim unequivocally that they could focus mainly on funding the city facilities

and programs solely used by adults through convincible evidence demonstrating an increase of

the number of adults in this city

In the second place, even if the birthrate does be decreasing and the number of students enrolled will soon decline, it is unreasonable to make a assertion that they can safely reduce the funds for education and athletic playing fields and other recreational facilities. On one hand, to some extent the number of students have nothing to do with the funds for education since it is possible that people may pay more attention to the education because they recognize that education is of significance in the competitive society in the future, thus it may call for a large funds for education and those recreational facilities. On the other hand, adults also needs education, for example, someone whose major was e-commerce finally works as a salesman and he has to learn something about marketing. Therefore, without the strong evidences concerning that they can really safely reduce funds for education and those recreational facilities, the recommendation seems invalid.

Finally, even if they can safely reduce funds for education and those recreational facilities, the author unfairly assumes that the measure will be effective to gain the goal to have sufficient money to fund city facilities and programs used primarily by adults. However, it

Page 69: ARG-FINAL

may not be the case. There is a possibility that the funds budgeted for education and athletic playing fields and other recreational facilities were originally a small part that even if reducing funds on those fields there has little effect on getting enough fund for city facilities and programs used primarily by adults. Therefore, without the information concerning the weight of budgets for each field, the recommendation seems unconvincing. 

In conclusion, in order to better evaluate this recommendation, the author needs to offer more information concerning the recent years' particular birthrate, the strong evidences to prove that they can reduce funds for education and those recreational facilities safely and the weight of budgets for education and athletic playing fields and other recreational facilities.

ARG-31

This argument is well presented but far-fetched. It lays a claim that Parson City residents place a

higher value on providing a good education in public schools than Blue City residents do. The

argument is in effect definitely impractical due to several flaws after a close scrutiny, albeit it

may appear plausible at a cursory glance.

First off, a threshold problem comes into being in this argument that the author clearly assumes

larger proportion of revenues on education with same number of people means bigger

concentration on education. However, this contention is open to a number of interpretations. We

would never know if the same amount of residents would not necessarily result in proportional

number of students. In this light, Blue city may have spent more on average since the number of

students in Parson City might be, say, more than twice than that in Blue City. Thus, without

accounting for and ruling out other likely scenarios, by no means could the author conclude that

the students in Parson City receive more revenues in on average.

Moreover, even though the author might be able to provide evidence for us to deduce a solution

to the problem presented above afterwards, the argument still maintains ill-conceived owing to

another problem. It's totally possible that the gross revenue of Parson City is more than 2 times

as those of Blue City. In this case, the amount of money in Blue City certainly occupies larger

proportion. To corroborate his point, the author should pay a close heed to as well as cope with

the representative alternatives, such as the amount of revenue both cities earn every year, at least.

Only then could he bolster the conclusion.

Page 70: ARG-FINAL

Finally, even if the foregoing problems might turn out to be solved by ensuing evidence, a

crucial problem remains that is the more money invested in education meaning the higher value

on providing education? It's reasonable to cast doubts upon the author's presumption which I

reject as inadequate. For instance, the author omits to inform us about the attitude of the residents

in both cities. Perhaps they regard money as a means of policy but not a really effective way to

improve the quality of education. Pursuing this line of reasoning, it proves to be the author's

responsibility to mull over his provisos so as to pave the way for a more tenable argument.

In retrospect, the author seems precipitous to jump to the conclusion based on a series of

problematic assumptions pertaining to the residents' attitude, the amount of revenues in both

cities, as well as the proportion of students among residents.

To dismiss the spectre of implausibility in this argument, the author ought to come to grips with

the problems mentioned above: (1) the proportion of students are the same in both cities; (2) the

amounts of revenues are congruent in both cities; (3) and the residents 'attitudes are like the

governments, which regard the amount of money as a criterion of the emphasis on education.

Only by grasping the gist of sound assumptions could the author draw a convincible conclusion.

After all, feckless attempts with a fallible method could be nothing but a fool's errand.In the first place, the author argues that Parson City residents place a higher value on providing a good education in public schools than Blue City residents do, because Parson City has recently spend almost twice as much per year as Blue City, even though both cities have about the same number of residents. However, same number of residents does not mean the same number of students. Thus, we need evidence about if these two cities actually have the same number of children who need to attend schools. In other words, if majority of residents in Blue city are senior citizens and most of their children do not need to attend local public schools, then Blue City do not a high budget for its public schools, because there are not many students. If this is the case, it is not reasonable to claim that residents in Blue city do not place high value on education. Furthermore, we also need evidence about if government budget is the only funding source that supports public schools in Blue city. If public schools in Blue city receive funds from various sources, for instance many public schools in Blue city receive a large amount of donation from their alumni, or there are some successful local businesses in Blue City that donate a lot of money to public schools, then, government budget alone does not reflect total fundings of public schools in these two cities. This can weaken author’s argument, because public schools is Blue

city may overall receive more funds through various sources.

ARG-32

Page 71: ARG-FINAL

Discussion of the reducing on-the-job accidents among companies as a whole has come into

vogue during the last decade. In the preceding statement, the author claims that to reduce

accidents on job they should follow the example of Panoply industries by shortening work shifts

by one hour. Though his claim may well have merit, the author presents a poorly reasoned

argument, based on several questionable premises and assumptions, and based solely on the

evidence the author offers, we cannot accept his argument as valid.

Primary issue with the author`s reasoning lies in his unsubstantiated premises. The arguer

provides no assurances that the data 30 percent on which the argument depends is statistically

reliable. Lacking information about the number of people surveyed and the number of accidents,

it is impossible to assess the validity of the result. For example, if only limited accidents were

surveyed, the conclusion that 30 percent decrease would be highly suspected. in the second

place, the argument is based on a false analogy. The arguer simply assumes that reducing work

hours as done at Panoply industries will result in declining job accidents for Quiot

Manufacturing and other company bears some relation to Quiot Manufacturing. However, the

author provides no evidence to support that this is indeed comparable. Maybe one company is

doing metal works with high accident risk but the other one is only dealing with producing

software for computers. Therefore, even though shortening working hours proved effective in

reducing accidents there is no guarantee that it will work just as well for other company. As a

result, we cannot safely assume that it will work also for Quiot Manufacturing. The author`s

premises, the basis for his argument, lack any legitimate evidentiary support and render his

conclusion unacceptable.

In addition, the author makes several assumptions that remain unproven. The arguer assumes that

just because decline of accidents follows shortening work hours, then decline has been caused by

that reason. However, no evidence is provided to support that this is the case. The mere fact that

shortening occurs before decline does not necessarily establish a casual relationship between

decline and shortening of accidents. It is highly possible that other factors might also bring about

the same results. For example, new security rules were applied at Quiot Manufacturing by

security section such as rules forcing to wear helmet at work. Without ruling out these and other

possible factors that give rise to reduction in accidents, the author cannot confidently conclude

that shortening resulted in decline. In the second place the author assumes that fatigue and sleep

deprivation are main contributing factors to accidents but he does not provide any evidence to

Page 72: ARG-FINAL

support this is the case. Even if fatigue and sleep deprivation result in accidents it does not

follow that they are the only reasons for accidents.

While the author does have several key issues in his argument`s premises and assumptions, that

is not to say that the entire argument is without base. With search and clarification, he could

improve his argument significantly.

In sum, the author`s illogical argument is based on unsupported premises and assumptions that

render his conclusion invalid. If the author hopes to change his reader`s minds on the issue, he

would have to largely restructure his argument, fix the flaws in his logic, clearly explicate his

assumptions, and provide evidentiary support. Without these things, his poorly reasoned

argument will likely convince few people

In the first place, the writer seems to assume that it is the longer work shifts of Quiot Manufacturing that made on-the-job accidents there were more than Panoply Industries plant. However, there may be other factors that contribute the result. It is possible that rather than working in the nearby Panoply Industries plant which may more safer, working in Quiot Manufacturing is more dangerous that always have on-the-job accidents in itself. In addition, it is very possible that rather than Quiot Manufacturing, the Panoply Industries plant was good at taking measures to protect workers which contributed to fewer on-the-job accidents than Quiot. All these lead the assumption that it is the longer work shifts in Quiot Manufacturing that made on-the-job accidents are more than Panoply Industries plant to an assertive one.   In the second place, the writer makes an assumption that it was the longer work shifts of Quiot that made workers there are fatigue and sleep deprivation which always resulted in on-the-job accidents, thus shorten work shifts would provide workers more time to have a good rest, and on-the-job accidents would naturally fewer. However, there is no information that workers are fatigue and sleep deprivation. Even if workers are fatigue and sleep deprivation, there is no evidence that the reason is the longer work shifts. Since it is very possible that some workers who needn't to work in the night, playing overnight which is easily making them tired, and it has nothing to do with the longer work shifts. Therefore, it is unfair to say that the longer work shifts contributes to the workers' seemly fatigues and sleep deprivation.Finally, the writer positively assumes that shorten each of their three work shifts by one hour would reduce the number of on-the-job accidents at Quiot and thereby increase productivity. However, the writer did not consider the consequence of shorten work shifts. As the writer's assumption that workers were fatigue and lack of sleep, if shorten work shifts, it is very possible that some workers would have more time to play outside the work which may easily make them be more tired, thus there may be more on-the-job accidents. Therefore, shorten work shifts doesn't necessarily reduce on-the-job accidents thus neither would it increase productivity.

Page 73: ARG-FINAL

ARG-33

The writer of the memorandum, who claims that construction of new generating plants will not

be necessary, seems to offer a cogent argument. However, the writer makes a lot of tenable

stated and unstated assumptions, even the superficial correlations are seemingly logical. And

there is a list of the results of those assumptions above examined as follows, of which results

reveals the fallacies in his/her reasoning.

First of all, the writer cites several recent surveys to reinforce his/her argument in that these

surveys indicate that home owners are increasingly eager to conserve energy, with an obvious

assumption that these surveys could also apply to their area, and also give the credit to these

surveys. However, it is entirely possible that the home owner in their area is not eager to

conserve energy, perhaps instead profusion on energy. And, perhaps these surveys are conducted

by the manufacturers which produce home appliances with excellent energy efficiencies and

have the purpose of inducing wide usage of energy efficient appliances, while the fact may be

opposite. Thus, with the lack of evidence to support the assumptions, the writer cannot convince

me that the total energy is going to decline.

Secondly, the writer assumes pontifically that the more energy efficient, the less energy

consumption there will be. However, it is not always true. For example, it is also possible that

the total energy consumption is rising with excellent efficiencies due to the augment in need of

increasing in numbers of home appliances. Even if energy efficient ones do consume less energy

than the older one, the writer also assumes unjustly that the most appliances in our area are made

up of the energy-efficient appliances which the manufacturers are marketing. Nevertheless, it is

possible that most appliances are still the older ones, which is not that efficient and cost less

energy. Thus, it is unfair to conclude such a claim that the total demand for electricity in their

area will not increase-and may decline, without checking the reliability of the assumptions

above.

What's more, the writer also assumes that for the past twenty years have always met their needs

doesn't necessarily means next year ,then the needs would also be able met as well. On one hand,

considering the development of society, it is entirely possible that the energy would increase as

more and more appliances come into everyone home. Anyway, it is natural that things would

change with the time flies. On the other hand, what if the tendency of the demand of electricity in

their area is increasing, and in the last year the demand just met the supply? Then it is natural to

Page 74: ARG-FINAL

predict that in the next year the demand would likely to be over the supply.

Without careful evaluation of these dubious assumptions, the writer cannot convince me that the

construction of new generating plants is not necessary.

All in all, without justification of the reliability of the survey he/she discussed, credibility of the

assumption of proportional relationship between the efficiency and the consumption and the

assumption of the tendency that the demand will not increase, the author cannot convince me that

construction of new generating plant is unnecessary.

In addition, the author also assumes that there is no other product that can potentially increase the demand of electricity. However, it is possible that people start to used electric cars or other appliances that used to powered by other resources such as gasoline, but now using electricity instead. This can increase the demand of electricity, and invalidate the author’s claim.

Finally, even if the demand of electricity indeed decreases in this area, which may not be true, construction of new generating plants may still be necessary. The author assume that the electricity generated by new plants only supplies the demand of this specific area, but if this assumption is unwarranted, for example, it is possible that the additional electricity generated by new plants can supply other surrounding areas. In this case, if the assumption is unwarranted, then it would weaken the author’s claim.

ARG-34

In the letter, the author argues that to prevent employees from wasting time, they should

implement electronic monitoring of their internet use from their workstation. And he cites

various reasons to bolster it. However, none of these are convincing because the claim relies on a

series of unsubstantiated assumptions which render them unconvincing on their stand.

first, the argument unfairly rests on the assumption that all the things empolyees do in the

computer is shopping or playing games. thus,the author argues that they waste time.however,

there is absents evidece to support it. so, it is as likely that they communication or chatting with

their client to develop their relationship in order to deepen their understand of their company

which is benefit to their business. however, if the company conduct the practice, i imagine that

these empolyees will aviod do it in order to decrease the possiblity of misunderstanding by their

leader and the punishment.

second, the author unfairly assumes that even if their stuffs spend time on playing games, it is

necessary that they will waste time? the author argues that only the employees work all the time,

Page 75: ARG-FINAL

the profits would be improve, or decrease. however, it is know to us that high efficiency in not

the result of the whole time working, but the best distribute of the time. if they are too tired to

work any more, the suppression from the leaders may make them less power to do other things.

in contrast,offering the stuffs the chance to enjoy themselves when they have accomplished an

item of the thing, they will be in gratitude to the company and will be work hard for it.

third, the author also unfairly assumes that the only way that the stuffs wasting time is to play

computer. thus ,the author argues that only if we prevent employees from surfing internet,there

be no chance for them to waste time. however, the thinking is navie. in my view, if the stuff

dissatisfy of the management of the leader, they will try to protest in secret, instead of surfing the

internet, they can be chat with colleagues, which is difficlut for leaders to oversee.

in sum , the argument is unpersuasive and in order to bolster it , the author need to offer more

evidences to substantiate these assumetions mentioned

The vice president for human resources claimed that to improve productivity, the employees' use of Internet use should be monitored. He/she further claimed that the employee who used the Internet should be identified and punished. At first glance, his/her suggestion is somewhat convincing. However, further inspections will reveals the flawed aspects of the assertion.

Firstly, there is no efficient evidence showing that most of the internet uses in work hours were spent on personal or recreational activities, such as shopping or playing games. Nowadays, many companies equipped internet to encourage employees to do more research on their researching subject, absorb more material useful, or conduct more thorough survey. Internet really had positive effects in job. However, still some employees used internet on personal or recreational activities. But what proportion of such use in total use of internet should be raised to determine the restriction of the internet.

Secondly, the identification and punishment will affect the moral of the company even reduce the productivity. Many websites have commercial links. If an employee pressed the link carelessly but received a severe punishment, he/she will be discouraged a lot, even had the mind to resign. Considering that it is very difficult to distinguish those careless press from surfing the website, the identification and punishment should be considered more seriouly. Sometimes moral is more important than discipline.

Finally, the monitor will increase the gap between the employer and employees. It is somewhat means I did not trust you. It will arose the complaints among employees and affect the productivity indirectly.

Page 76: ARG-FINAL

In sum, the vice president offered little, if any, support for his/her suggestion. The software and the monitors are not a little investment. Unless a more thorough investigation was conducted, the company will ill-advised to act on it

To beginning with, it lies in the presupposition that everyone use Internet to something besides their work. It is a kind of Internet-era; we use Internet for many kinds of business. So do employees. This assumption is invalid due to its assertive. If someone use Internet to check mails from their clients or look up some vital information for their work, and be punished in the end. Imagine this-people dedicate themselves to their employer, the consequence is doomed to penalty; how the so-called “better work ethic at Climpson” does could appear in this placeSecondly, another assumption is questionable. The vice president of human resources assumes that once they install the monitoring software, hours that be wasted on the Internet would be reduced. One possibility is omitted that there might be someone who is keen for Internet recreations at a certain time; they will probably surf the Internet it even if they are to be punished. To some degree, this software is not going to work as it is expected. It would cause more rebellions, because people usually say that “repression leads to revolution”.Additionally, a untenable assumption is made by the author that employees in this company have merely one way accessing to the Internet. However, it is fallacious hypothesis. Since we are in a high developed technical society, mobile phone, ipad and other electronic facilities can also facilitate people to access the Internet. Granted that the software is working, employees would have other way to make them accessible to the Internet. In this case, installing the electronic monitoring is not a so effective method that reduces the time-wasting.The vice president of the human resources believe that if they install the software to restrain employees’ recreation hours can lead to the higher productivity as well as a better work ethic. Another possibility he didn’t concern is that, which might exist rationally, the restraint of Internet use would make the employees feel that they are in a kind of no-Internet prison, it’s harmful rather than helpful to motive their passion at work. So, the decreasing productivity is imaginable.In a nutshell, the author makes this recommendation based on a trial of void assumption and conjecture. There are manifold could-be situations he’s not consider. To promote the productivity of employees, set some software to impede them from recreation is not an effective way. The crux of some breakings in their productivity and work ethic is to find out the root of hour-wasting or the low productivity, so that a more convincible and tenable recommendation could be available.

ARG-35

The letter described the situation in which Sunnyside Corporation should decide whether a restriction of the maximum water flow scheme should be applied from the current three buildings to all their twelve buildings. The writer listed evidences about the complaints from the residents and savings they gained, making a confirmation that the restriction should be performed to all the

Page 77: ARG-FINAL

buildings. However, some questions should be posed about the ratiocination process, and thus we may get a more rational conclusion. First, the owner have received few complaints about the restriction of the water over the first month. Given the conclusion they make, we know that they have assumed a long-lasting few complaints and a low proportion of the complaints. Actually, we have little information about the latter two factors that the complaint will keep at a low level and a low ratio. Furthermore, the report of few complaints from the residents is not equivalent to that they do make few complaints, in that some of the residents who are not happy about the water restriction have not reported to the owners, making the number of cases of complaints unreal. Besides, if the low complaint level in the first month does indicate that the residents do not mind the regulation, they are not likely to neglect it for always. For example, the first month for which the corporation restrict the water threshold may be in winter when people seem to need lower water supply than in summer, so as the time passes, complaints are likely to emerge in a larger amount that the corporation have to regulate the maximum of water supply again. In conclusion, the writer should consider the complaint report to a more strict extent and include more possibilities into consideration, hence he or she might know how to get the the attitude of the residents more precisely.Second, it appears on the letter that the owners want to save money by means of restricting maximum water flow. It may not be achieved for that people, when having shower with a low water pressure, might need to take a longer time than when the water pressure is normal, thus the water consumption is not likely to lower and the money could not be saved. What's more, were the maximum water modified to a low value that residents cannot stand, they are moving out despite other advantages of the Sunnyside that could offer them. In this case, not able to make a profit, the Sunnyside is even facing a situation that they cannot compensate the money they invest to the water restriction program. We recommend that they survey their residents about the average showering time and make a calculation concerning whether the corporation will save money through the restriction of maximum water flow, so that they could gingerly make a decision on this stuff.

All in all, the Sunnyside Corporation should acquire more facts about the residents' attitude on the modification and make some calculation on the possibility that they will save money. Only after that can they make a more rational decision

The argument contains several unaddressed details that can potentially affect the validity of the author’s argument. First of all, there is no evidence available to show that the water consumption indeed has decreased after implementing modified showerheads. Then, there is no solid evidence to show that predicted outcome is warranted. The author predicts that the consumption of water will drop by restricting the maximum water flow of all the showerheads. However, the owner ignores an important fact that it is possible when the maximum water flow is reduced to one-third of what is used to be, it takes longer for residents to shower. Consequently, the longer shower time would compensate for the reduced flow rate. After all, this would not contribute to

Page 78: ARG-FINAL

reducing water consumption. If this is the case, the owner’s predication is invalid. Therefore, due to the other possibility, it is questionable if the owner’s suggestion can help reduce water consumption, and further save money on the water bill.

On a perspective of a company, profit plays the essential role in its enterprises, and some managers would always consider about how to enhance their profits.  However, when a company is serving to public, it has to preserve a balance between the benefits of people and the income of the company.  The author of the letter makes a kind of evidence that seems as a shelter of the flaws in the author’s conclusion.  It would be listed in the following paragraphs.  

To cite the one-third of the water flow, the owner of the Sunnyside Towers apartment (S) refers to the evidence of restriction, which could be helpful for saving the spending of water of the corporation on each month.  The author ignores many different possibility.  First, the decrease of water flow might make the users to use water longer each time than before, and the sum of the water utilization would be without any change.  Secondly, the declined water flow might give the users a reason to choose another company for water accommodating. Losing the users, the profits might be influenced.  Without any justifiable evidence, it would be difficult for the author to predict the future profit whether it is going to be boosted or not. As an evidence, the author indicates that because of a few complaints, the change makes no problems with the showers.  The report should be disputed that, maybe, there are many problems of the adjustment but all the users and the one who modified the machine of controlling water flow do not discover, or even more likely, the problems are still hided in a shadow which might be occurred in future. Besides, even there is not so much trouble of the modification, but the complaints of the users are intense outside the report. Some one might complain a lot without expressing to the researcher.  It is unjustifiable that the owner has only concentrated on the report literally. More analysis must be needed for the manager to make the determination.

According to the evidence which the owner indicates above, he concludes that the adjustment could be expended to all twelve buildings in S.  It means that the author gives an assumption of the fact that all users in twelve buildings would be the same as the people in the first three buildings after a month.  The threshold problem of the evidence is that the survey is only participated by a part of the user before one month.  This part of people might be change their mind after the month.  Differences must be occurred around all the people in the twelve buildings, which makes the reflection of the change would be far away from equivalence.  It would gives rise to more complaints of different aspects, and the problems would be revealed. The writer should give more details about the other users of the buildings, and focus more on their benefits not only the profits of the company. In short, more analysis would be needed for the manager to make the determination  of whether the change should be expended.  The owner should give more details about the other users of the

Page 79: ARG-FINAL

buildings, and focus more on their benefits not only the profits of the company.  Losing the users, the profits might be influenced a lot.

ARG-36

The argument concludes that the daily usage of Ichthaid which is derived from fish oil can

prevent colds and lower absenteeism of West Meria. This conclusion is based on the premises

that people of East Meria eat more fish then they have few colds, that Ichthaid derives from fish

which can prevent cold equally with fish, and that colds lead to absenteeism. At the first glance,

the recommendation seems plausible. However, its logic is faulty, since some specific evidence

are overlooked by the arguer.

First, the arguer assumes that people in East Meria visit the doctor not very much often equals

with they have fewer times of getting colds. However, based on the common sense, people who

get colds may treat themselves at home instead of going to ask for the doctors' help.

Even if colds have a lower rate of appearance in East Meria, the reason of it may not only caused

by consuming fish. Although fish consumption is high there, other factors can also prevent the

appearance of colds such as environment and climate. If the air is clear and whether is moderate,

people have lower chance to catch a cold. Therefore, those hidden factors ignored by the author

can weaken the recommendation.

Secondly, the argument falsely depends on the assumption that both East and West Meria are

very similar in each environmental and physical condition. Then the arguer assumes that West

Meria will be lower its colds rate if it follows East Meria's steps. However, although eating fish

does prevent the cold in East Meria, West Meria may not receive the desirous result by simply

copying the action. Because there may different conditions between the two areas. The arguer

does not provide any specific evidence to substantiate the people, environment and any other

conditions even the causes of colds in both areas are totally same. Without them, the analogy

between two areas can not adequately serve to the points suggested by the arguer.

Finally, the author maintains that Ichthaid, an element exists in fish oil can prevent colds.

However, the author does not offer the evidence that there are no other ingredients in fish or fish

oil can also inhibit the appearance of colds. For example, the eggs in fish may also have the same

Page 80: ARG-FINAL

function.Thus,unless the argument can rule out the other possibilities, the prediction is shaky and

vulnerable.

Even though the Ichthaid has been proved that it can prevent cold effectively, however, cold may

not the very reason that leads to the absenteeism in schools of West Meria. Other factors need to

be taken into account. For example, students may not finish their homework on time and feel too

guilty to show up at school; or they may just do not feel that they need to come to school that

day. So, author should to reveal evidence which is strong enough to support the standpoint, or

the whole recommendation is only built unreliably.

In conclusion, the argument that focusing on taking Ichthaid daily to prevent colds and

absenteeism is rather weak. If the author takes into account the potential evidence behind the

event, the argument would be greatly strengthened. The action of caring development of the

West Meria should be certainly appreciated, however, without the additional evidence to support,

the recommendation can not bring the efficacious effects as predicted

In this memo, the author recommends the daily use of a nutritional supplement derived from fish oil as a good way to prevent colds and lower absenteeism. To supports this assertion the author cites a study conducted on people in East Meria, and points out that the attendance level will be proved since cold are the reason most frequently given for absences from school and work. While the advice is plausible at the first glance, a close scrutiny of this argument reveals that the argument is unconvincing in several aspects.

To begin with, as for the study the author cites, a question need to be answered is whether the study is reliable enough before I can accept any conclusion the author arrives at based on it. However, the scope and the validity of that study is vague. Perhaps the sample was not representative of people in East Meria, asking only those healthy club members who seldom get cold because of exercising often. Perhaps the study used ambiguous words such as the option of “seldom” to mean “once or twice ”, but the respondents thought people who go to doctors less ten times a year are “seldom” visit the doctors. Still perhaps the study was 15 pages long, and the respondents did the questionnaire halfheartedly. We just do not know. Only if the study is fully valid, representative,  and reliable, can it use to back the author’s argument. Additionally, the author falsely assumes that the high fish consumption contributes to less colds, simply based on a coincidence. We may ask if there are other factors leads to less colds. It is likely that high medical care and healthy eating habits keep people in East Meria stay healthy and be away from doctors. It is also likely that the consumption of fish is not for eating, but for other uses. For example, people there mostly are fond of fish-farming and often buy fish for fish-farming, or always raise cats with food of fish. Unless all these possibilities can be ruled out, the causal relationship between high fish consumption and less colds is weak, if not unfounded.

Page 81: ARG-FINAL

Finally, even if all the inference and deduction made in the argument above is true, the author still fails to perform a feasibility analysis of the true cause of absenteeism and the effectiveness of the nutritional supplement derived from fish oil. Firstly, we may ask if having a cold is the true reason given for absences from works and schools. However, it is likely just a excuse in many cases. If so, the amount of absenteeism will not decline even they use the nutritional supplement every day. Secondly, we doubt if the nutrition supplement can work. After all, natural fish are different from something derived from fish oil, and if some nutrition in fish that is helpful for curing cold may not contained in the nutritional supplement, it will be useless. Also, the author give no information to deal with whether the dose of daily use is appropriate and whether it can cause side-effect, and it is quite possible that the daily use of a nutritional supplement do harm to health. The author’s reasoning is definitely flawed unless he/she can convince me that these and other possible scenarios are unlikely.Pursuing a healthy and high quality life, people come up with many advice. However, the recommendation the author make is invalid and misleading, basing on my above analysis. They cannot expect improved situation of less colds and lower absenteeism if people in West Meria use nutritional supplement derived from fish oil. Careful analysis of all the factors I have presented is the best first step to reaching their goals of preventing colds and lower absenteeism.

ARG-37

Author states that ten years ago, as part of a comprehensive urban renewal program, the city of Transopolis adapted for industrial use a large area of severely substandard housing near the freeway. Subsequently, several factories were constructed there, crime rates in the area declined, and property tax revenues for the entire city increased. To further revitalize the city, we should now take similar action in a declining residential area on the opposite side of the city. Since some houses and apartments in existing nearby neighborhoods are currently unoccupied, alternate housing for those displaced by this action will be readily available. At first sight the argument looks plausible but the thorough analysis proves it to be a shallow conclusion.

Here author assumes that the new area, which is at the other side of city, has the same environmental and physical conditions as that of the area, which they adapted ten years ago for industrialization. For Industrialization the basic requirements like adequate land, water supply, electricity and the most important one are transportation facility should be available. The area, which Transopolis adapted ten years ago, is near freeway. That means it is on the way for the transportation. To strengthen the argument author should provide the proof that the new area, which is at the other side of city has same facilities available as that of the area, which they adapted ten years ago.

The second assumption made by author is the crime rate declined due to factory constructions

Page 82: ARG-FINAL

and industrialization. Here if author gives the proof how the industrialization really affected to crime rate declination then it will strengthen the argument. Probably author can provide the statistics of what was earlier crime rate and what is present crime rate. 

The third claim author makes is the property tax revenues increased for the entire city. The property tax revenue increase can be by other reasons also like new facilities provided to citizens like good schools, good shopping complexes, due to which may people are interested in staying in that city which resulted in the increased property tax revenue. The evidence stating that the property tax revenue increased for the entire city only because of the industrialization of the area ten years ago will strengthen the argument.

In summation the author has stated the argument without providing sufficient evidences and proofs. The evidences discussed above should be provided to strengthen the argument

Additionally, the author makes blind assumptions like the decrease in the crime rate and increase in property tax revenues are only due to the construction of several new industries. Though the industries could have made the people involved in better jobs instead of committing crimes, and increased the flow of money, that cannot be related to the reduce in crime rate. There might be some other factors which might have contributed to the increase in crime rate; the city police might have been more active in the past ten years, or some new commissioner of police may have kept the city quiet, or the people might have attended some meditation classes which might have motivated them to stay away from crimes. The crime rate reduction has made the author think the crime rate is due to the urban renewal program. Similarly the property tax revenues might have been the result of increase in the percentage of tax to be paid. So the residents might have paid more tax amount. 

Though the construction of industries might lead to improvement in the economy, they also add to the pollution in the environment. Since already one side of the city has been industrialized, sparing another side will end up in people moving out of the city in search of some pure air and water. Then the city authorities will be facing problem in relocating the industries outside the city at some point. So there will be a waste of fund and the people will be disturbed by the city authority's unplanned actions. 

To put it in a nutshell, examining the various angles of a new renewal proposal, the argument does not provide strong evidences to create industries. The authorities might take a survey of how many peopleRG- are living in the area, how many of them would like such industries, how about the need for new jobs, if opening such industries would prove a profit to them. So the city authorities must seriously consider various issues before undertaking such a proposal.

ARG-38

Page 83: ARG-FINAL

In addition, the author claims that people will pay more for alpaca overcoats because the price of most types of clothing has increased. To test the veracity of this claim, we need evidence to show that, nowadays, people are willing to spend more money on coats, or on alpaca overcoats specifically. It is possible that the increasing of clothing price is due to inflation instead of people being willing to pay more on their clothing. If this is the case, people might not want to buy alpaca overcoats at all since alpaca overcoats are relatively expensive. The alternative possibility can weaken the author’s claim.Finally, the author suggests that selling alpaca overcoats will be more profitable, because the retail price for alpaca overcoats is high. In order to make the argument more persuasive, we need evidence about how much it costs to manufacture alpaca overcoats. If the price of fabric has increased significantly, and compensates for the higher retail price, then, selling alpaca overcoats would not be very profitable as predicated.Overall, the author’s conclusion is based on several details that the author fails to give substantial evidence to validate. Thus, in order to make the recommendation more sound, the author should provide more concrete evidence

The author of this argument claims that Sartarian, a company should resume production of deluxe alpaca overcoats to increase their profits. To support this argument the author points out that a company has a new fabric supplier and their major competitor no longer makes an alpaca overcoat and so on. But for several reasons, this evidence provides scant support for the author’s recommendation. First, it doesn’t mean that discontinued production of an alpaca overcoat by their major competitor is linked directly with their profits. Because there can be many similar companies in clothing market. Although their major competitor no longer makes an alpaca overcoat, the rest companies have possibility of an alpaca overcoat production. So, if Sartarian want to produce an alpaca overcoat, it should compete against new competitors. Second, it is possible that an alpaca overcoat lost popularity from most of the people for some reasons as time goes by. For example, changing of weather, variety of preference, appearance of alternative design, etc. If the more high-quality and fashionable design were developed, there is a no cause to insist on an alpaca overcoat. In this case, Sartarian not obtains more profits by principle of the market economy for dissonance of demand between supply.Third, risen price of almost types of clothing for the past five years isn’t connected directly with higher prices of alpaca overcoat. According to Adam Smith, who is the father of economics, invisible hands settle the price. So, it isn’t that an alpaca overcoat brings Sartarian’s more profits in proportion to the increase of price in almost types of clothing. An alpaca overcoat is differ form normal clothing because it is used in winter or cold weather. So, it isn’t possible to compare with almost types of clothing without concerning about an alpaca overcoat’s character. In conclusion, the author’s assertion is not well supported. To bolster the assertion, the author must provide better evidence how many potential competitor is the clothing market and reliable data about a favorite rate of an alpaca overcoat. And to better evaluate the argument I would need to compare a unit cost of a new fabric to a unit cost of high-quality wool before five years

Page 84: ARG-FINAL

ago. First, We can know this company had trouble with the supplies of good cloth five years ago. However, we cannot know their deluxe alpaca overcoats had good quality. This is because high-quality cloth cannot guarantee the quality of clothes. Furthermore, we have no information whether those were popular or not. Therefore, it can be said that the vice president's study on the market five years ago has no significant information.

ARG-39

In this argument, the author recommends that Bay City should open a new seafood-specific

restaurant which will lead to fame and profit. For approving the conclusion, the author employs a

variety of evidence such as statistic data and nationwide study. At first glance, the

recommendation sounds reasonable, however, further examination reveals that there maybe other

evidence ignored by the author. Without considering all the possibilities, the recommendation

may not be persuasive and invulnerable.

First, a 30% increase in seafood consumption at Bay City does not necessarily illustrate that a

high demand on seafood served by restaurants is existing. Although 30% seems significant, the

author may ignore the fact that the actual level of consumption of seafood there is rather low. If

true, this data is too weak to prove that a new seafood restaurant can bring to profit.

Second, a nationwide study showing a clear tendency that two-income families toward dining out

and eating healthily does not necessarily apply to Bay City. Perhaps, families in here do not

prefer this life-style. The author needs to proceed a sound survey in two-income families in Bay

City to find out whether its result will support the initial idea.

Even if most of families in this city follow the nationwide trend mentioned above, it is unfair to

infer that these families will select seafood or even seafood restaurant as their first choice.

Because other than dining out, ordering take-out food at home is relatively convenient for people

who dislike eating in a a noisy and crowed restaurant. Although people may go out for dinner,

there are still various kinds of food available for people to pick. Facing all kinds of delicious

food, it is hard to imagine people will stick to only seafood. Furthermore, according to the

common sense, seafood can not be the only healthy diet, other nutrient food like milk, eggs and

vegetables are all can assist people to keep sound. Any of these scenarios discussed above, if

true, would cast considerable doubt on the argument's conclusion that a new seafood restaurant

in Bay City would be popular and profitable.

Page 85: ARG-FINAL

Finally, even if Bay City families surge into the new seafood restaurant, the restaurant may not

be popular and profitable. In one thing, Captain Seafood restaurant is rather unfamiliar for the

citizens, and as a new brand it may need a long time for people to accept it. Another thing is,

profit is a factor relating to not only revenue but also cost. The new restaurant may suffer a

deficit when it is establishing its positive images at the beginning. For example, obtaining high-

quality, healthy seafood, or promoting the new restaurant both will cost a large fortune. At the

same time, if there are rare customers coming to eat, the restaurant will lose a lot.

To sum up, the proposal lacks credibility to conclude that the new brand seafood restaurant will

be popular and profitable for the prediction is based on several shaky and dubious surveys. To

strengthen the argument, the author would have to set a new survey which is thorough and

comprehensive enough to substantiate the feasibility of opening a brand-new seafood restaurant

in Bay City

the arguer fails to provide some accurate statistics about the consumption of seafood dishes in Bay City restaurants. It is true that consumption has increased by 30 percent during the past five years, but maybe it just because the base number is actually a small number and therefore the whole consumption is still not so considerable. Thus, without giving compellent evidence, the arguer can not bolster the recommandation.

At first glance, it might seem logical to concur with the claim made in the argument, that the new Captain Seafood restaurant would be quite popular and profitable. But on a second thought, the conclusion seems to be based on some assumptions and there is no evidence to justify that the restaurant will be popular. Answers to certain important and significant questions are required, in order to evaluate the conclusion and the premise on which it is based. 

The first question to be answered is how significant the 30 percent increase in seafood is. According to the sales study, the consumption of seafood, at local restaurants, has increased by 30 percent. However, the study has not mentioned what is the overall increase in the consumption of all kinds of food at restaurants. This information is essential to determine if the increase in seafood consumption is only a part of an overall rise in hotel food consumption or due to a genuine orientation of people’s culinary taste towards seafood. Suppose, for example, the overall consumption of food at restaurants has increased by 300% during the past 5 years, then the increase of 30% in seafood consumption is only a part in the whole. 

Another piece of information that might help us come to a conclusion is the type of food that two-income families consume. The argument states that the amount of home-cooked meals consumed by these families has reduced significantly over the past ten years. Therefore, the number of restaurant meals consumed by them might have increased. But, there is no mention

Page 86: ARG-FINAL

about the kind of meals that they eat. Also, the study mentions that the families give importance to the nutritional value of their meals. We cannot assume that, just due to a decrease in the number of home-cooked meals, the number of seafood based meals will increase. If these families consume only salads, vegetarian food and chicken due to health reasons, then there will be no effect on the consumption of seafood. 

Also, the argument extrapolates that the new Captain Seafood restaurant will become popular just because it specializes in seafood. Evidence is needed as to what are the factors that make a restaurant popular. Price, cuisine, hygiene, waiting time, location, décor and other parameters may be used to assess the popularity of a restaurant. Suppose, the new restaurant is situated far away from the city centre and is difficult to access, then not many people would visit the restaurant owing to the distance. Also, the service provided at the restaurant plays an important role in its popularity. Bad service may not attract many customers, thereby reducing the profits. So, popularity and profitability of a restaurant cannot be attributed to the type of cuisine served by it alone.

On the whole, had the argument been supported with more facts and information from the studies conducted about increase in food consumption and the quality of food consumed by families in the area, and also comparisons of the quality of service, pricing and rating of the new restaurant with other established hotels, it facilitate the process of assessing the success of the new restaurant.

ARG-40

The group of people who have consistently consumed dairy products throughout the years may

have had fractures more than other people, but the author of the article makes number of

unsubstantiated assumptions about the risk of disease. Based on these assumptions, the author

boldly claims that the consumption of the dairy products actually increases the risk of

osteoporosis (OS).Though the argument or study is made for the betterment of the society, it may

fail to achieve the goal because it is flawed.

The author’s first mistake is to assume that many people think that the disease OS can be

prevented by consumption of dairy products and that the disease is associated with number of

environmental and genetic factors. Although people may think that the dairy products may be

good for their bones on the whole, the author has no basis to assert that people think that the

disease is prevented by consuming such products. It is not stated that how many people are of

this opinion. May be only the grandma living next door to the author thinks so or may be a very

large population is of this view.

Page 87: ARG-FINAL

Secondly, the author states that the study conducted led him to the conclusion that the people

who consume dairy products throughout their lives have a higher rate of bone fracture. The

author never states that how many people were included in the study and for how long were they

studied. Also it is not stated that what kind of people were included in the group and what were

their environmental and genetic condition. May the people that were under study were football

players or may be hockey players and there were no computer nerd included in the study group.

Obviously, the people involved in games tend to have a higher rate of injury and may be fracture.

Also it does not make sense that to consider the consumption of dairy products, the cause of this

disease when author himself states that the disease is caused by many environmental and genetic

factors.

Lastly, the author states that bone fractures are symptoms of OS, therefore the consumption of

dairy products increase the risk of OS. The author states this on the above flawed assumptions

and also wrongly assumes that any kind of fracture is an indication of OS. It may be true that OS

is accompanied with weak bones but with someone having a fracture because of a car accident is

not likely the case of OS. Also it is not stated that in which conditions these fractures occurred.

For instance , if the person injured got a fracture by only a gentle kick of his best buddy, may be

a sign of poor bone condition. Neither is the body structure or the genetic makeup is stated.

The author’s argument can be very powerful and persuasive if the author were to include certain

assumptions. For example if the author were to include the type of study that was carried and by

whom, the people that participated in it, their daily activities and genetic make-up and also the

detail of why where and how these fractures occurred , can make the authors argument very

powerful. With these kind of assertions the authors call might be more justifiable.

ARG-41

In this newsletter, due to a nationwide study, the author claims that to decrease the traffic

accidents, government should pay more attention on educating people about bicycle safety than

encouraging them to wear helmets. Supported by statistics, at the first glance, the

recommendation sounds plausible. However, unwarranted evidence and flawed assumptions

made by the author weaken the argument's reliability and feasibility.

First, a ten-year nationwide study suggests that there is an increase of 45 percent of bicyclists

who wear helmet while there is an increase of 200 percent of accidents related bicyclists. The

Page 88: ARG-FINAL

author deems that there is a relation between those two increases. However, the author fails to

consider the possibility that during ten years the population may have increased either. If true,

the increase of accidents may be still limited in a normal level or even may decrease somewhat,

because with the higher population, there are likely more bicyclists than before, so are cars and

the roads will become more crowed. For that matter, accidents tend to happen more often but

which may be not led by the increase of bicyclists who wear helmets.

Second, the author unfairly assumes that those accidents are caused by bicyclists who think

wearing a helmet will be safer then care less and risk more. However, other factors which may

cause the accidents are ignored by the author, even though bicyclists may get involved. For

example, after such a long time, is there any chance that certain sections of roads have had

problems, but government did not take actions on time. Roads like these, the rate of accidents is

usually high. Or, it maybe the car drivers' responsibilities to cause the accidents for they possess

poor driving skills and are inept to handle emergencies. In those two probable conditions, the

bicyclists can hardly be blamed and even may become victims.

Finally, even though the assumptions in the newsletter are acceptable, however, the advice made

by the author that people needn't to wear a helmet when they are riding is of potential risk. Based

on the common sense, although helmets can not fully protect people from serious car crash, they

do play a irreplaceable role in daily protection. People, especially the kids who prefer to expose

themselves into fresh by dangerous ideas, those helmet can be useful.

To sum up, after examining all the assumptions involved in the recommendation, intend to prove

the disability of helmets and convince the government to offer bike-safety lecture rather than

encourage people to wear helmets, the author fails to employ strong evidence to substantiate the

conclusion. To relieve the traffic problems, the author would have make a more well rounded

research. Imprudent recommendation maybe misleading and cause severe problem in a long run.

First of all, the author implies wearing helmets results in increasing bicycle-related accidents.

Since the number of bicycle-related accidents has increased 200 percent while more bicyclists

started to wear helmets based on the nationwide study, the author necessarily assumes that there

is a positive correlation between wearing helmets and the increase in the number of bicycle-

related accidents. However, there is no evidence suggests these increased bicycle-related

accidents are caused by bicyclists who wear helmets. It is possible that majority of these

accidents are caused by bicyclists who do not wear helmets. In addition, it is also possible that

Page 89: ARG-FINAL

the significant increase of bicycle-related accidents is due to the increase of cars on roads, which

has nothing to do with wearing helmets. If the correlation between wearing helmets and bicycle-

related accidents is proved unwarranted, and it is because of the worse traffic nowadays, then the

conclusion concerning the impact of less emphasing on wearing helmets would be false, and

there would be no grounds for predicting reducing the number of bicycle-related accidents.

Furthermore, the author suggests that wearing helmets can contribute to increasing the number of

serious injuries from bicycle accidents. This argument only make sense on the basis of an

assumption that people tend to take more risks because they feel safer when they wear helmets.

However, this assumption is not supported by any evidence. If the assumption is unwarranted,

and people do not necessarily feel safer, nor they want to take risks, then the conclusion

concerning that people feel safer with helmets on, and are more likely to take risks, consequently

more serious accidents would be invalid. Since the assumption about bicyclists take more risks

and feel safer may not be warranted, as a result, the suggestion of emphasizing educating people

about bicycle safety is not sound. 

Before the author's conclusion that government should educate people more about bicycle safety

and less on encouraging or requiring bicyclists to wear helmets is made, the evidences provided

suffered from several logical flaws. The author seems to have assumed the bicyclists take more

risks when they are wearing helmets without considering other factors may have effect on the

results of the argument.

Firstly, the author claims that there is an increase of 60 percent of bicyclists who wear helmets

during a ten-year study; meanwhile, there is an increase of 200 percent of accidents involving

bicycling. However, the author assumes without justification that the population during the ten

years has not increased. The assumption is unwarranted because it is definitely possible that the

population would increase during these ten years. So it is likely that there are more bicyclists

than before. With the increase of population there will also be more car drivers, which would

make the traffic more crowded. For that matter, the increased accidents are due to the increase of

population.

Secondly, the author's conclusion is based on an unfair assumption that all the accidents are

caused by the bicyclists. The assumption cannot convince me because there are other factors may

attribute to the increase of the accidents. For instance, as time goes by, there are more cars than

before and many car drivers are not that skillful when handle with traffic emergencies such as

when driving on the mountain road, while the bicyclists wearing helmets have been more careful

Page 90: ARG-FINAL

than before. Then it stands for the reason that the bicyclists are actually the victims instead of the

one responsible for the accidents.

Finally, even we concede all the assumptions the author made, the author's recommendation may

not reduce the number of serious injuries from bicycle accidents. When dealing with one project,

we should consider the whole conditions. Since bicycles are not the only vehicle on the roads,

the author recommends concentrating more on educating people about bicycle safety is

unilateral. Perhaps cars are the main cause on the roads, or perhaps the traffic conditions such as

roads became dangerous during these ten years. Without a comprehensive investigation about

this and other factors, the author's recommendation would not reach the effect as expected.

To sum up, this argument is not based on valid assumptions, which is indispensable for a cogent

conclusive. The author should consider more realistic factors mentioned above so as to

substantiate the argument.

ARG-42

First of all, the author suggests that the tourism bureau on the island of Tria should charge people

for using the beaches to raise money for replenishing the sand, and to protect the beaches. To test

the validity of this suggestion, we need evidence to show whether charging people for using the

beaches will significantly reduce the number of tourists. It is possible that many tourists use the

beaches because they are free. If the tourism bureau starts to charge tourists, then, it is possible

that the number of tourists would decline significantly. Further, if the number of tourists drops

significantly, the tourism bureau on the island of Tria would not be able to raise a lot of funds for

replenishing the sand. For this reason, the suggestion will be proved to be invalid, and weaken

the argument.

Secondly, the author implies that replenishing the sand can help prevent the erosion of beach

sand along the shores. To test the veracity of this claim, we need evidence about whether the

project succeed in Batia or not, in other words, whether replenishing the sand can effectively

stop the erosion or not. Since no substantial evidence is provided to show that replenishing the

sand can effectively prevent erosion. It is possible that replenishing the sand has failed in Batia,

and it cannot effectively prevent the erosion of beach, then, there would be no grounds for the

author’s suggestion, and the author’s claim is weakened.

Moveover, even granted that replenishing the sand can effectively stop the erosion, the author

also suggests that copying the project of replenishing the sand from Batia can help prevent the

erosion of beach sand in Tria. To verify the validity of this suggestion, we need evidence about

Page 91: ARG-FINAL

whether this project is suitable for Tria’s case. It is possible that beaches of two islands are

facing different situations. For instance, nearby buildings along beaches of Tria are well-

contructed, or there are less storms in Tria area, or the erosion in Tria is not as serious as that in

Batia. If these alternative possibilities are proven to be true, then, it seems that replenishing the

sand is not the appropriate solution, because, replenishing the sand does not solve the specific

problem for Tria’s case.

To sum up, in order to evaluate the argument, we need evidence about whether charging tourists

for using the beaches will result in the decline in the number of tourists, whether replenishing the

sand is an effective solution to prevent erosion, and whether it is suitable for Tria’s case or not.

In order to protect the beaches from further erosion for a better island and tourist industry, the

speaker suggests a policy of charging people for using the beaches. He considers this method

will function well due to the less usage of beaches and raising money for replenishing sand to

improve the environment of the beaches.However, because of a shortage of necessary evidence

to dismiss the possibilities behind his assumptions in this reasoning, his argument seems

completely unconvincing and fallacious.

First off, he considers the main reason for the erosion of the beaches is the over usage by people.

How can he prove it? After all no evidence shows that the erosion is a man-made result. The

erosion could be fairly likely caused by the natural issues, like the pH of the sea water or the

fierce climate changes; it can also be induced by the air pollution of the industry, or acid rains. If

one or more than one of these possible factors are the real reasons for this erosion, the charging

policy simply cannot make any sense in protecting the beaches from further erosion. 

Secondly, the writer presumes that replenishing the sand to the beaches can function well to

recover the eroded beaches. Is there any convincing evidence for that statement? The only

seeming proof is the analogue with the building protection by replenishing sand. However, are

they the same issue? Beaches are known as some kind of natural products, part of the

complicated biological system, while the buildings are constructed by humans with materials of

simple components compared to the nature. The conditions of the sand in the beaches could be

fairly complex, regarding the pH, the components, the moisture contents and even the living

organisms. I suppose this method of replenishing the sand may even damage the biological

balance of the beach systems, let alone recovering it. 

Finally, even if the main reason for the erosion is from man-made issues and the recovering

Page 92: ARG-FINAL

method works, there still no evidence showing whether the fines from charging the users can

smoothly make up the financing. There is no reference to show how many visitors would like to

pay for using this beach. Also, the erosion issue may decrease the attraction of the beaches,

making it less profitable to people, let alone the barrier of charging fine itself. In fact, there could

be very few people who would like to pay for the beaches. If so, I suppose we cannot raise

enough money for the recovering issues unless there are funding from the government or some

private sources. 

In sum, the writer thinks about this issue so partially as to dismiss all other possibilities, therefore

fail to provide necessary evidence discussed before to clarify the puzzles. This argument needs

an in-depth scrutiny with all necessary evidence. Otherwise, no conclusion can be obtained.

ARG-43

Consultants have predicted that the landfill in West Egg’s would completely fill up within 5

years. This prediction was made 2 years ago; at the same time as when residents began to recycle

twice as much as the previous years. Next month, the chairperson of the town council foresees an

increase in recycling because charges for pickup for other household garbage will double. Ninety

percents of surveyors say that would recycle in more in the future. Therefore the chairperson has

reasons to believe that the landfill should last longer than predicted. This is an admirable attempt

on the residents to protect the environment. However, we don’t have enough evidence to show

that the landfill won’t get filled up within the next 3 years.

Before we can assume that it will take longer to fill up the landfill, we need to have some of the

following key information: What is the percentage of garbage presently covering the landfill?

The charges for pickup. And the number of residents that took the survey.

Unless we know how much garbage is in the landfill at present time it would be fallacious to say

that that it will last longer than 5 years. How much garbage was disposed before the residents

started to recycle? We are 2 years in, and the number of garbage could be half filled up. With the

decrease of increase of recycling, we could say that the landfill will be 3/5 filled up for the next

year. That leaves us with 2 years left.

The charges for garbage pick up will double. How much will the charges be? The increase may

not be significantly be high enough for the residents to be more concerned about recycling. Some

may be more conscious of recycling, some may not give a hoot.

Page 93: ARG-FINAL

The 90% of surveyors who said that they would recycle more in the future; how many residents

of the 90% is that? It could be 9/10 people. We don’t know for sure, so the survey is not a good

indicator to predict the decrease of garbage disposal. A better question to have ask the residents

is: If they have been recycling for the past year.

With some weak evidences to show that the prediction for the landfill will last longer, we cannot

full assume that it will. Key informations such as: the amount of garbage that is already presently

in the landfill, the cost of the increase garbage pickup charges, and the number of residents who

took the survey need to be taking into account before making such presumption.

To begin with, one of the pieces of evidence for us to better assess this argument is information about the total amount of the garbage. Common sense tells me that it is highly possible that the total amount of garbage including the recycling materials may increase due to the increase in the population and other social or economic factors. If that is the case, though the residents might recycle much more materials than ever before, the garbage which filled the landfill would increase as well. Unless the author can provide sufficient and convincing evidence to rule out this possibility, by no means could he/she conclude that the available space in their landfill can last longer. Even if the author is able to offer evidence to prove that the total amount of the garbage will not increase, we also need to be offered another piece of evidence that can confirm the relationship between the higher charges for pickup of other household garbage and the increase in the amount of recycled material. There is a likelihood that the people know nothing about the rise of the charges and hence fail to respond as the author predicts. Even though they are well informed about this policy, perhaps they care little about the price change. In order to confirm the prediction that there will be an increase in the amount of recycled material next month, the author needs to provide firm evidence to eliminate these possible alternative explanations, otherwise the argument will be seriously weakened. Last but not least, even if the evidence mentioned above can be provided, in order to confirm his conclusion, the author also needs to offer evidence about the validity and scope of the survey mentioned in the argument. Who conducted this survey? And was the number of the respondents large enough to ensure the representativeness of all the residents in West Egg? We just don't know. Moreover, it is possible that the survey was concerned with too many aspects of city life and contained only one question about the people's willingness to recycling. Without sufficient evidence to confirm the validity and representativeness of this survey, it can't reveal people's commitment to recycling and hence the conclusion that available space in the landfill will last for a longer time may be unpersuasive.To sum up, the argument is ungrounded as it stands and fails to convince me that the available space in West Egg's landfill will last for longer time. To bolster his argument, the author needs to provide scientific, clear and sufficient information about the total amount of the garbage and substantiated evidence to strengthen the assumption that the higher charge for pickup can lead to

Page 94: ARG-FINAL

more recycled materials. Furthermore, the validity and the representativeness of the survey should be proved with more evidence.

ARG-44

The author claims that customer's boycott could forbid CCC carrying out their mining plan could

protect the African tropical rainforests on the West Fredonia. However, there are no sufficient

persuasive evidence to support the advice though it seems appealing at the first glance.

The benefit of customers would be cared firstly. Refusing CCC's copper productions maybe

could affect their works or lives, notwithstanding they can refuse CCC brand, which is forged on

the direct-productions, how can they realize those indirect productions such as the wire or

integrated devices? Once CCC provides their copper to other factory as wire corn, but customers

cannot tear the plastically surface down to copper corn to identify if it comes from CCC. So

completely refusing the CCC's productions is unpractical.Moreover, if the CCC gives up their

mining plans, other companies will invest to that land for their industrial projects too. Not for the

copper mining, but cleaning the forests for cattle grazing, logging, automobile factory or building

highways or dams, the local ecosystem would be destroyed too. So to prevent one kind of

investment is not a property step, because we cannot refuse all the daily consumptions.To protect

the tropical rainforest, not only West Fredonia but also all rainforest nations should prohibit all

kinds of extractive industry. Because the rainforest ecosystem is a delicate balanced web, the

species depend on each other, one disappears, another extinct with it, which in turns destroy the

entire web. So the sole plan of preventing the CCC’S mining plan is meaningless.

People would not believe the boycott to the CCC's products will entirely protect the West

Fredonia's rainforests unless more information should be provided, such as the nature

environmental information about the West Fredonia, the CCC's production list and the analysis

of the plan's possibility

First of all, the author suggests that boycotting the Crust copper company’s copper products can force the company abandons its mining plan, and prevent the environmental disaster in the tropical nation of West Fredonia. However, this suggestion only makes sense on the basis of the assumption that the Crust copper company will actually

Page 95: ARG-FINAL

use the land they bought in tropical nation of West Fredonia for mining copper. In the statement, the author does not provide any evidence to show that the CCC has decided to use the land for mining copper. It is possible that the CCC company purchased that land for other use instead of mining copper. If the CCC company purchased the land in the tropical nation of West Fredonia for other use such as building new research labs, then, the assumption would be proved unwarranted; thus, there would not be any mining related problems at all. After all, there would be no grounds for the suggestion. 

Secondly, if the CCC company indeed decides to use the land for mining copper, the author concludes that mining copper on that land will inevitably result in pollution, threaten local endangered animal speices, and further lead to environmental disaster. Since mining copper can results in pollution, the author necessarily assumes that mining copper is the only factor that could contribute to environmental disaster. However, it is possible other factors can potentially threaten these endangered animal species, for instance, climate change, floods or other natural environmental factors which have nothing to do with mining copper. If the assumption that mining copper leads to the environmental disaster proves unwarranted, and it is the climate change that results in the extinction of endangered species, then the conclusion concerning the impact of mining copper on environmental disaster would be false, and this can weaken the author’s argument.

Overall, in order to evaluate the validity of the author’s

Page 96: ARG-FINAL

suggestion, we need to address these assumptions first, including making sure the CCC company is going to use the land in the tropical nation of West Fredonia for mining, and mining copper is the only factor that results in the environmental disaster and threatens local endangered species.

ARG-45

The arguer advocates an immediate and active promotion of online degree programs like those at

Omni to increase enrollment and decrease the cost of maintaining buildings. This decision is

made just on the basis of a rough comparison between Humana University and Omni University

in enrollment and expenditures. However, if a all-rounded survey of the result together with the

further development of the school is taken, the situation of the promotion above may not seem so

optimistic and need a second thought.

First of all, comparative data of the two universities are not comprehensive and unconvincing.

Because the base level of the two universities has not been given yet. Perhaps Humana

University originally has a larger student population, and maintained a relatively higher level of

enrolment. Maybe the quantity remains stable just for the past three years. While in Omni

University, enrollment increased merely in last year for a variety of reasons, not only owing to

online degree program. The arguer did not make these prerequisites clear. It is unreasonable to

make such a constructive educational change after this comparison.

Second, Humana University didn’t connect the current dilemma as to decrease in freshmen

number and budget deficit with their own reality. Thus a rather misleading solution came out

being. Because there are a lot of reasons combined together causing the embarrassment of

Humana University now. It is because reduction of the number of school-age students going to

college or backwardness in online educational program.

Meanwhile Omni University has less entry requirements for online students so that many adults

choose Omni University to continue their education. Since the majority of these online members

attending class simply for a higher diploma, it is difficult for so many people to learn stuff really

useful. Therefore the quality of these graduates cannot be match for those who complete their

four-year or longer education in the formal school. Over time, this mode of rush online education

Page 97: ARG-FINAL

will be opposed by the public sooner or later. However, the regular students of the class in class,

at least one student and the school system, quality guaranteed, after their graduation, to

contribute to the society are more or less.

On the contrary thanks to the school and the teacher's supervision, regular school graduates must

have mastered the required knowledge. Later, they can eventually get anywhere in life when

entering into society.

Furthermore, if Humana University wants to carry out e-education, do avoid copying temporarily

successful model of Omni University. Instead quite a careful study into those subjects suitable or

online teaching is inevitable. Then try these patterns for several terms as to spread more subjects

online. Only in this way can Hunan University ensure the quality of teaching activities. In

addition, an especially online reporting feature should be available for teachers in order to keep

tracking trainees ‘attendance and progress.

Finally, educators should bear the aim of education in mind-- preparing prospective graduates

with the necessary competencies and offering an integrated education program for all students to

assure their fitness and developmental interests in subject’s and educational activities which will

serve them for life. If one college meets both, regardless of whether it is loss or profit, the

education is a success.Summing up, it is best for Hunan University to think about these aspects

before implementing the hasty programme to maintain its reputation as well as solve the current

troubles

First of all, in the memorandum, the president of Humana University concludes that offering online degree programs can help increase its total enrollment, because nearby Omni University showed that the number of students who enrolled in online degree programs has increased by 50 percent since last year. However, the given statistic is very vague, because if the base is small (if, say, there were a few students who enrolled in online degree programs at the beginning), then a 50 percent increase would not be very significant. Thus, the first question that needs to be addressed in order to evaluate the argument is the actual number of students who enrolled in online degree programs. If the number is not significant compared to the current enrollment at Humana University, then creating online degree programs will not help the embarrassing enrollment situation at Humana.Furthermore, the president implies that the online degree program contributes to reducing the cost of maintaining buildings and dormitories, since Omni showed a significant decrease in expenditures for dormitory and classroom space. However, the president only points out one side of the issue. What about the money students paid to live in dorms? It is true that since students who enrolled in online degree progrmas do not have to live on campus, and the cost of

Page 98: ARG-FINAL

maintaining dorms will decrease, but the school will also lose the revenue from student living, and food, too. Will the revenue compensate the cost of maintaining buildings? Therefore, the second question we need to ask in order to evaluate the conclusion is whether the revenue from dorms and foods (cafeteria) offsets the cost of maintaining buildings.Finally, the president concludes that creating the online program can solve current budget problems. However, it is not clear whether the online program is profitable or not, and how much it cost to start the program. Thus, the last question we need to ask to verify the claim is how much money the university needs to invest on this online program, and is it profitable after all? If it costs Humana Univerisity too much money to start the online program, and it makes considerably less, then, following the recommendation will not help solve budget problems. 

To sum up, in order to evaluate the president’s recommendation, we need to know the number of students who are likely to enroll in online programs, if the revenue from dorms can offset the cost of maintaining buildings, and whether the online program profitable or not.

ARG-46

In this argument, the author asserts that experts are mistaken in suggesting that spending too

much time using computers has caused a decline in fitness. The author also claim that the recent

decline in the economy is most likely the cause of fitness levels declining, and fitness levels will

improve when the economy does. This argument is unwarranted due to several critical flaws as

discussed below.

Firstly, citing the medical experts' saying that the number of Corpora's citizens who meet the

standards for physical fitness decline in comparison with that twenty years ago, the author claims

that the physical fitness of Corpora's citizens actually declines. Blind to the difference between

current standards for adequate physical fitness and the standards as defined twenty years ago,

both the author and experts assume that the current standards for adequate physical fitness are

amount to even laxer than the standards defined twenty years ago. Unfortunately, advances in

medical technology have made it possible for the tendency that current standards are stricter. In

this case, although the number of Corpora's citizens who meet the standards for physical fitness

decline, actual fitness of Corpora's citizens may not necessarily decline due to the difference of

standards. The author would benefit from offering more information about the current standards

and standards defined twenty years ago for physical fitness.

Then, on the basis of the fact that overall fitness levels are highest in regions of Corpora where

levels of computer ownership are also highest, the author claims that the experts' opinion about

Page 99: ARG-FINAL

the cause of fitness declining is wrong. In fact, the author equates levels of computer ownership

with levels of computer usage. However, it is possible that levels of computer ownership are

high in an area, while the levels of computer usage are low. Perhaps for the demand of work,

most citizen have at least two computers, one in office while the other at home. But it’s highly

skeptical that those citizens who have at least two computers would spend more time on

computers than those only owning one computer but addicted to computer games. In view of

such situation, since levels of computer ownership do not indicate levels of computer usage, the

relationship between high levels of computer usage and high levels of overall fitness is weak and

thus the author could not provide sufficient evidence to case doubt the experts’ opinion.

Finally, based on the assumption that decline in the economy lead to low expenditures on fitness-

related products and services, the author claim that the recent decline in the economy is most

likely the cause of fitness decline, and fitness levels will improve when the economy does.

However, no evidence provided to support this assumption. In fact, people’s expenditures on

fitness-related products and services could be affected by so many factors, such as people’s

attitude on the efficacy of those products and services, that only consideration on economic

environment is ill-advised. Perhaps citizens are so credulous about the efficacy of fitness-related

products and services that although the economy decline recently, they are still willing to pay for

those products and services. The author should have a more complete understanding of facts

associated with people’s expenditures on fitness-related products and services.

In summarily, the argument is unpersuasive. To bolster it, the author should first carefully

evaluate the assumption discussed above

First of all, the author claims that using computers has not made citizens less physically fit, since overall fitness levels are highest in regions of Corpora where levels of cumputer ownership are also highest. Therfore, the author necessarily assumes the high level of cumputer ownership means the level of usage. However, the author does not consider the possible that for some families, all family members have their personal computers but they do not use their computers all the time, whereas in some families, there is only one computer, but everyone use it heavily and spend a lot of time on that computer. In this case, when the assumption proved unwarranted, many people have their own personal computers but barely use their computers all the time, and the level of computer ownership does not necessarily reflect the usage, then, the author’s claims would be false, and there would be no grounds for the claim.

Moreover, the author argues that the decline in the economy is most likely the cause of the

Page 100: ARG-FINAL

decline in fitness, and fitness levels will impove when the economy does, because it is shown that this year’s expenditures on fitness-related products and services are low. However, this argument only make sense on the basis of the assumption that the drop of expenditures on fitness-related products and services contribute to the decline in the fitness. It is possible that people do not need to buy new fitness-related products in order to exercise, they can just go for a walk in their neighborhoods, or the city newly built some public facilities that citizens can work out there instead of paying to go to gyms. If these alternatives are valid, and the assumption is proved unwarranted, then, the decrease in expenditures on fitness-related products and services would have nothing to do with the decline in fitness levels or the decline in economy. Therefore, the conclusion that fitness levels will improve when the economy does is false, because the conclusion has no grounds.

To sum up, in order to evaluate the validity of the author’s argument, we need to make sure these assumptions including levels of cumputer ownership relfect levels of usage, and the decline in economy contributes to the decline in fitness.

ARG-47

The owner of the Armchair Video presents a memorandum that comprises suggestions to

increase their declining profits. Taking a clue from one of their stores in Marston, which reduced

its operating hours and the overall inventory by stocking only recent films, he says that this

should be adopted at other stores as well. Since they are known for their discounts and special

offers, they cannot consider increasing the prices to gain profits. This change has also seen very

few customer complaints because of which it can be adopted at other stores to reduce operating

expenses.

However, this could be a useless change, if not profitable, since the memorandum ignores a

number of other possibilities that can prove the opposite to be true.

As is mentioned in the memorandum, the store in Marston reduced its operating hours in the last

one month. Reducing the operating hours can reduce the expenses only if it can be introduced

successfully. However, there are a number of things attached to this decision that are ignored in

the memorandum. While reducing the operating hours reduces the expenses, it also cuts down on

the number of customers that come to the store. There are many people for whom the evening

time of 6 to 9 suits best to visit the market. They cannot come to the store before that. Moreover,

the evening time is most profitable in many places. Similarly, there are customers for whom

other times of the day are suitable. Hence, if the operating hours are cut down, these customers

Page 101: ARG-FINAL

are lost. Therefore, the owner of the video rental stores should consider this before reducing the

working hours of all the stores.

The owner also considers reducing the overall inventory of films by stocking only those films

released in the past two years and giving away the films older than those. With this he probably

expects to reduce the costs of maintenance of these films.

However, it is possible that there are a number of customers whose first choice is older films.

The owner has to thus make a careful decision of the choice between retaining the business by

keeping these customers and earn profits or by reducing the costs related to keeping these films

to increase profits.

There could have been very few customer complaints of these policies in Marston, but it cannot

be accepted for all places. The choice of films by customers depends upon the composition of

customers of the place. While at one place people could prefer the recent films, at other places

there could be a liking for the older films. Hence if there are no complaints from customers of

Marston store, there could be reports of customer complaints at other places. Moreover, it should

not be forgotten that such a policy has been adopted in Marston only for a period of one month.

This period is a very small period to observe the reaction of customers as well as predict future

profits. People take time to notice such changes. There could be many customers who are not

aware of these policies and thus they did not complain about them. If there have been very few

customer complaints in the past one month there could be an increase in the number of

complaints in the coming times as the customers get to know about the changes.

These points discussed above disagree with the owner's suggestions of following the policies of

Marston to gain profits. Instead of such policies, there should be other ways of increasing profits.

A change in the marketing strategies should be considered to attract more customers. Since

Armchair Video is known for its special bargains, the owner should introduce more of such

offers along with some subscription or membership for customers and ensure profits. Thus, it can

be said that the suggestions given in the argument should not be followed in all the stores

because of their impracticality. The owner should instead consider other options that can be more

profitable for its video rental stores.

Secondly, the author claims that in order to reduce the overall inventory, Movies Galore can stop stocking any DVD released more than five years; thus, this can improve profits. In order to test the validity of this suggestion, we need evidence about if there is a significant amount of audiences who like old movies. If there are many audiences who come to Moives Galore because

Page 102: ARG-FINAL

of these old DVDs, then, stop stocking movies which are more than five years ago could potentially drive these audiences who like old movies away. The loss of audiences can result in the decline in the revenue, which may also offset the reduction in reduced inventory costs, and this would not help improve profits, thus, the suggestion is weakened.

Finally, the author mentions that Movies Galore only received a few complaints about these new policies; thus, these new policies are applicable. To test the validity of this claim, we need evidence about if the number of audiences who visit Movies Galore has dropped. It is possible that audiences think it is too troublesome complain. Nor does this necessarily mean that they like these new policies. If audiences do not like these new policies, they would simply stop going to Movies Galore instead of complaining, then, the loss of audiences could significantly reduce the revenue, and this would not contribute to improve profits. For this reason, the author’s claim is weakened.

To sum up, in order to evaluate the author’s argument, we need additional evidence including whether these new policies can result in the significant loss of income due to the loss of audiences.

  Firstly, the author assumes that raising prices is not a good option to increase profits because they are famous for their low price. It seems available at the surface. However, the author fails to provide any evidence to demonstrate that raising prices indeed cannot bring us profits. With the development of economy, we all know that prices' going up happens very often nowadays. What occurs at the same time is the increasing level of people's consumption. If the store can take a proper raising which meets with people's consumption level. The customers can understand it very well and it can prevent the prices from declining. Moreover, we cannot eliminate the possibility that our consumers are more interested in our good serving attitude or rich inventory. Thus the price is not a problem for them at all. Unless the author provides me a entire and reliable survey which can show the actual reason leading to the profit declining, I cannot agree on the author's views that raising prices is not a good option to increase profit   Last but not the least, confessed that these polices work efficiently in downtown Marston, the author's inference that these polices can be extended to all the other Movies Galore stores is still unpersuasive and open to doubt. Every store's condition is different from each other's and determined by various factors such as, location, main consumers, people’s interests etc. Without reliable and strict investigation about every store's exclusive conditions, we cannot hastily get this conclusion  Last but not the least, confessed that these polices work efficiently in downtown Marston, the author's inference that these polices can be extended to all the other Movies Galore stores is still unpersuasive and open to doubt. Every store's condition is different from each other's and determined by various factors such as, location, main consumers, people’s interests etc. Without reliable and strict investigation about every store's exclusive conditions, we cannot hastily get this conclusion.

Page 103: ARG-FINAL

ARG-48

First of all, the author suggests that Clearview is a good place to retire since housing costs in Clearview have fallen significantly during the past year. In order to evaluate this claim, we need evidence that shows whether current housing prices in Clearview are lower than the average price in that area. It is possible that housing costs in Clearview were initially very high, and even after housing costs have fallen significantly, housing costs still remain higher than the average housing costs in that area; then, the implied decline in housing costs would not effectively help attract more retirees; thus, the author’s argument would be weakened.

Moreover, the author also mentions that taxes in Clearview remain lower than those in neighboring towns, thus Clearview is a more suitable place for retirees. In order to evaluate this claim, we need to know whether taxes in this area are generally higher than other parts of city. If it is shown that taxes in this part of city tend to be much higher than other parts of city, then, taxes in Clearview are still high even though they are lower than its neighboring towns. For this reason, the author’s claim is ilogical.

In addition, the author implies that Clearview is a good choice for retirees, since the mayor promises many new programs to improve schools and streets. To test the validity of this argument, we need evidence that the local government will put these words into practice. The argument would be weakened if the local government does not do what they have promised, but even if they really work on these projects, improving schools is not likely to benefit the majority of retirees, because it is possible that majority of their children are old enough to work or go to college somewhere else, then, improving schools might not be favorable.

Furthermore, the author suggests that Clearview can provide good health care to retirees, because there are far more physicians in this area. In order to verify this argument, we need evidence about whether these physician can provide quality service or not. The argument would be weakened if there is evidence shows that these physicians in this area do not provide good services, for instance, they are not approachable, unfriendly, or impatient when they talk to senior citizens, then, people might not like to visit them even though they are close and available, since the quality of the health care is more important than the quantity.

To sum up, in order to evaluate the author’s argument, we need evidence about whether housing costs in Clearview were initially very high or not, if taxes are generally higher in this area compare to the rest of the city, and whether the quality of heath care is high or not.

The article presents a suggestion for retirees to move to Clearview. This suggestion sounds good -- Clearview seems like a paradise for retired people for its beautiful scenery and social environment. However, one cannot make the decision to moving to Clearview only by this

Page 104: ARG-FINAL

article. Some specifics presented in this article make the argument illogical.

In the first place, the article asserts that housing cost fell significantly last year, which can be a good reason for people to move. But the falling prices make no sense of an inexpensive house. For instance, although the expense of houses in Clearview has fallen, there might be another city named Wonderfulview whose housing cost is always much lower than Clearview's. Other evidences may weaken the process of deduction, for example, people living there prefer renting a house rather than buying one when they retire. If the above evidence presented may be true, the author's seemingly logical suggesting is definitely unconvincing. To convince retired people to choose Clearview, the author has to provide much more evidence such as a statistical survey report about people's preference of buying or renting a house, and prove the fact that after housing costs' declining, there is a great advantage in housing in Clearview.

The argument also claim that the mayor of Clearview promises that some infrastructures will be built and some benefits will be given to the people there. But we have noticed that it's just a promise from the current mayor. In other words, the assertion is just a word, which is not restrained by a law or a signed protocol, and there is a probability that it will not be realized in a near future or even never be achieved. If either of these happens, a retired person who chooses Clearview will not enjoy the newly-built equipment in the city, which also makes the argument tenuous.

In addition, the number of physicians in the area is above the national average level. From my perspective, the number of physicians cannot embody an excellent health care service of a city and there will be no embrace from retirees. To weaken the argument, the author provides no evidence indicating whether the physicians are skillful or not. If the physicians are all inexperienced interns just graduated from medical schools, no one will believe that retirees in Clearview can expect excellent health care. As a consequence, only if the author can guarantee that most of the doctors in Clearview are prestigious big shots in medical field can we believe in this argument's assertion.

To sum up, the author of the argument provides some evident reasons for us to persuade retirees to move to Clearview, but it isn't reasonable enough. To strengthen the article, other solid evidences have to be presented. In this way, the argument can be persuasive and logical, which will contribute to a moving-boom to Clearview.

ARG-50

First of all, the author claims that lavender cures insomnia within a short period of time, since the quality of sleep of tested subjects seems to be improved overtime. In order to test the veracity of the claim, we need evidence about the experimental result of a control group, in which group members do not receive any treatment. It is possible that it is some psychological effects that

Page 105: ARG-FINAL

make tested subjects sleep better, or insomnia cures by itself overtime even without any treatment. If the quality of sleep in control groups is also improved within three weeks, then, the improvement in sleeping quality might not be mainly due to the effectiveness of the scent of lavender, and the claim would be false and weakened.

Secondly, the author implies the link between the effectiveness of lavender and the improvement in sleeping quality. To test the validity of the implied link, we need evidence that the observed alleviation in insomnia is solely due to the scent of lavender flowers instead of the medication during the first week. It is possible that the drowsiness observed during the first week is due to the side effect of the sleeping medicine, or it is due to the synergistic effect of both lavender and medicine, and can worsen insomnia. If the evidence shows that the observed drowsiness is indeed the side-effect of the medication, and the sleep medicine typical shows its effects during the second or third week, then, the observed relief in insomnia might not be solely due to the effectiveness of the lavender, the implied link might not be valid, and this can weaken the author’s claims.

Finally, the author suggests the improvement in sleeping quality is due to the effectiveness of lavender. To validate this conclusion, we also need evidence about whether the sleeping environment in the controlled room is the same as those at homes or not. It is possible that it is quieter in controlled rooms compare to people’s home. If the alternative possibility is valid, then, the claim that lavender cures insomnia would be false, and the conclusion would have no grounds.

To sum up, in order to evaluate the author’s conclusion, we need evidence about whether a control group shows improvements in sleeping quality, and whether the alleviation of insomnia is solely due to lavender rather than medicine or a better sleeping environment.

First, it is unfair to assume that 30 volunteers represent the whole patients having insomnia. It is entirely possible that 30 volunteers have the specific vocation or live in the specific area. In either case, the results of experiment could merely represent the people having the job or living in the area. Without credible evidence their representing whole insomnia patients, it is unpersuasive to me.

ARG-53

First of all, the author suggests that the citizens of Forsythe have adopted more healthful lifestyles since the survey shows that their eating habits are more nutritious. To evaluate this claim, we need evidence about if the survey is representative. If the sampling population of that survey is not large enough, or the participants they chose were biased, then, the result of the survey would not reflect eating habits of the majority citizens, and the result would not be

Page 106: ARG-FINAL

conclusive. For this reason, the author’s argument would be weakened because the given supporting evidence is problematic.

Morever, the author also implies the link between the increase in sales of food products containing kiran and citizens having adopted healthful lifestyles. To test the veracity of that implied link, we need evidence to show whether citizens consume more food products containing kiran because they want to be healthy, and thus they purchase more food products containing kiran on purpose, or whether food manufacturers added kiran into various food products while consumers are not even aware of that. If the increase in sales of food products containing kiran is a coincidence, and people buy more food products containing kiran for other reasons instead of realizing kiran can help reducing cholesterol, then, the author’s claim would be weakened.

Furthermore, the author also mentions the link between sales of sulia and citizens having adopted healthy lifestyles. In order to evaluate this claim, we need evidence about the nutritional content of sulia, and whether people choose not to buy products containing sulia solely due to their health concerns. The argument would be weakened if the evidence show that sulia is actually a healthful substance; but even if sulia is not good for the health, there can be a plethora of reasons (such as bad taste, expense) other than health concerns that prevent people from purchasing food products containing sulia.

To sum up, in order to evaluate the author’s conclusion, we need evidence to show whether the survey is reliable, and whether people choose to buy or not buy some food products due to their health concerns. First of all, the author suggests that the citizens of Forsythe have adopted more healthful lifestyles since the survey shows that their eating habits are more nutritious. To evaluate this claim, we need evidence about if the survey is representative. If the sampling population of that survey is not large enough, or the participants they chose were biased, then, the result of the survey would not reflect eating habits of the majority citizens, and the result would not be conclusive. For this reason, the author’s argument would be weakened because the given supporting evidence is problematic.

Morever, the author also implies the link between the increase in sales of food products containing kiran and citizens having adopted healthful lifestyles. To test the veracity of that implied link, we need evidence to show whether citizens consume more food products containing kiran because they want to be healthy, and thus they purchase more food products containing kiran on purpose, or whether food manufacturers added kiran into various food products while consumers are not even aware of that. If the increase in sales of food products containing kiran is a coincidence, and people buy more food products containing kiran for other reasons instead of realizing kiran can help reducing cholesterol, then, the author’s claim would be weakened.

Furthermore, the author also mentions the link between sales of sulia and citizens having adopted healthy lifestyles. In order to evaluate this claim, we need evidence about the nutritional content

Page 107: ARG-FINAL

of sulia, and whether people choose not to buy products containing sulia solely due to their health concerns. The argument would be weakened if the evidence show that sulia is actually a healthful substance; but even if sulia is not good for the health, there can be a plethora of reasons (such as bad taste, expense) other than health concerns that prevent people from purchasing food products containing sulia.

To sum up, in order to evaluate the author’s conclusion, we need evidence to show whether the survey is reliable, and whether people choose to buy or not buy some food products due to their health concerns.

ARG-54

The given argument proclaims a hypothesis for the extinction of large mammals in the Kaliko Islands. The author cites several inferences that he makes, disregarding the possibility of humans as the cause for the extinction. However, the author fails to explicate certain crucial details of the argument and hence makes the argument specious.

The author identifies that human and the large mammals lived together for about 4000 years on Kaliko Islands, after which the mammals dwindled in number. The author fails to blame the humans for the act, arguing that, had the human kind hunted the larger species, the bones of the animals would have been found in the Island. However, the human could actually be responsible, not through hunting, but through the encroachment of the lands inhabited by the species, resorting to deforestation and utilizing the food of the mammals themselves. Since bones of fishes were found in the Islands, it could as well be possible that these mammals relied primarily on fishes for food and consequently lost their source to the humans. These could have contributed to the departure of the larger mammals from the Kaliko Islands. 

The fact stated by the author in his story, regarding the absence of bones from the Islands is intriguing. Presuming that the species were destroyed by some environmental factor or a climatic change, as propose by the author, the bones of the animals must still be present in the forests of the Islands. This undealt theory raises speculation whether the animals are actually extinct. It could be possible that these mammals have evolved or migrated to other places, leaving the Kaliko Islands. Thus the author's views are nebulous and incomplete. Evidence that the large mammals actually died on the Islands and an account for the bones of the dead mammals would make the argument convincing.

The author fails to divulge any information about the other forms of life in the Kaliko Islands. Though the author's view that humans are not to be blamed for the destruction could be acceptable, the extinction could have been the consequence of other larger and feral animals in the Islands hunting these mammals for food. Considering another case, these mammals could be

Page 108: ARG-FINAL

internecine in their lifestyle, mutually destroying each other. Thus the author in shallow in his approach to the incident at Kaliko Islands.

The author states his theory for the extinction of large mammals in the Kaliko Islands and holds man ignorant for the deed. However he cites very few proofs for his views and fails to account for the absence of bones in the Islands, thus creating a major flaw in the story. Had he paid heed to the aforesaid arguments, would his hypothesis be cogent.

ARG-55

The writer claims that by introducing several games relating to life like graphics the decling trend of the sales of whirwind games will be reversed. To support the argument , the author mentions that the players prefer lifelike graphics games as indicated by the survey.Also the ad campaign introduced towards a specific age group might give an impetus towards its sales. The argument though plausible at first suffers from major inconsistencies.

The author asserts that the recent survey conducted on the players indicated they were more inclined towards life-like graphics. However the reliability of the survey undermines the authenticity of this assumption/.May be the players who participated in the survey were fanatics of life-like graphics concerned games.However the survey nowhere indicates that it included the players related to all fields of video games.Unless the author gives more information about the survey this assumption cannot be substantiated.

Even if we assume that the survey is indeed reliable, the extensive ad campaign introduced by whirlwind will positively increase their sales is not convincing. It is possible that the players who had preferred life like graphics games were not 10-25 years old people.The survey does not show any evidence to prove this .If the people 10 to 25 years old are not fascinated by these games then this ad campaign might itself backfire.Thus the sales trend might fall even further than anticipated.

The author assumes unconvincingly that the players 10 to 25 years old will be possessing upto date computers. If hardly few players have the latest computers the introduction of such games would be infeasible The survey must provide more information whether majority of the population possess up to date computers. f they do not,then are they willing to spent money on updating to latest ones to play the games they are planning to introduce.

To sum up,the argument mentioned suffers from many inconsistencies. To bolster the argument , the author must confirm whether the survey conducted was reliable one and whether it included players from all fields and age groups.The survey must also mention whether the ad campaign will indeed be fruitful and the 10 to 25 year old people are really looking forward towards their newly introduced games.

Page 109: ARG-FINAL

The sales of the video games might increase in the next few months, but the argument provided is insufficient in various fronts to make such a prediction. A thorough analysis of reasons for decrease in sales is essential to understand what improvements need to be made to increase the sales. The author says that the company has conducted surveys to gain customer feedback which I think is a step in the right direction, but only a specific out come of the survey is presented (video games player prefer games that provide life like graphics etc). I think this information in itself is not sufficient to claim that the sales are bound to increase. 

The business of video games is tricky one, one needs to understand that there are two major components to this. One is the customer who plays the game and the other is the payer who more often than not just pays for it. In most cases (age group 10 to 18) customer is the person who plays the video game the payer is the parent who might not necessarily play the game. I think survey of the parents is essential before we can make such claims. Whirlwind is ought to take into account the preference of parents and children to make better and affordable products. This information is not provided. 

The author claims that the new products will meet one particular customer preference but he does not perform any analysis of the competition, affordability, ease of use, availability, scalability and other essential factors to draw his conclusions. In fact from the argument one might infer that this new product will require latest technology, which might have an adverse affect on the sales. There are various factors which should be taken into consideration while designing a product. The author does a poor job in explaining what the various factors are that have gone into designing the new products of Whirlwind. The author’s claims are weakened due to lack of this information. 

End of the day, the sales of Whirlwind might actually go up, but the argument put forth by the author has left out lot of pivotal information to make this claim. Due to this, the proposed argument is weak in various fronts and fails to convince me that the sales of Whirlwind company is going to increase in the next couple of months.

if the sample population of the survey is very small and biased, or most participants are beyond the ten to twenty-five years old age range, then, the survey could not accurately predict the preference of the targeted players, and the argument would not be convincing.

Moreover, even if the result of the survey is reliable, the author mentions that the sales of Whirlwind video games will increase dramatically, since Whirlwind has introduced several games that satisfy players’ preferences. Thus, the author necessarily assumes that the decline in the sales in the past two years is solely due to that fact that Whirlwind failed to provide the specific feature which most video-game players like. However, it is possible that players do not like Whirlwind’s games because their games are too expensive, or it is also possible that their games are too easy even for ten-year old players. If the assumption is proved unwarranted, and

Page 110: ARG-FINAL

players do not like Whirlwind video games for some other reasons, then, even if Whirlwind video games have features players like, players still might not choose Whirlwind video games, and the claims would be weakened.

In addition, the author claims that the sales of video games will increase dramatically in the next few months, since Whirlwind has invested extensive advertising campaigns. Thus, the author necessarily assumes that new games and advertising campaigns can help achieve their desired sales. However, if the assumption is unwarranted, and there are many similar games available on the market with lower prices, then, players might not choose Whirlwind’s games, the sales would not increase dramatically, and the argument would be weakened.

ARG-56

In this argument, the arguer claims that the company should stop the Endure manufacturing

process to seek more profit. This conclusion is based on the premises that customers prefer the

appearance of product rather than the endurance. The recommendation's logic is faulty, however,

since it relies on several shaky assumptions to bridge the gap between it premises and

conclusion.

According to the resent survey, the author assumes that they should care more about the socks'

look rather than their endurance which is based on the purchasing frequency and preference of

purchasers. There are potential flaws. First, the author fails to provide the information of the

respondents whether they can be the representatives of the whole buyers or even actual wearers,

and whether they are selected randomly in a large scope. Besides, the author mentions that the

customers in the survey are from northeast. Can those people represent buyers from the whole

country?

Secondly,several other things may also determine their choices. For example, distinct career

needs divers socks to match the working suite. Usually, people work in an office will choose

plain color to fit their suites while people in fashion fields will consider style and color of socks

more. Gender and age, other factors may influence the choices. Females and children may prefer

light colors and fancy styles while males and the older may prefer mono-color and simple

patterns. Simply put, different needs call for different choices.

Thirdly, even though the average customers will buy similar kinds of socks, the arguer still can

not assume customers care less about socks quality based on their purchasing frequency. As the

author mentions in the survey, their customers know clearly that the socks are strong enough to

Page 111: ARG-FINAL

last quite a long time and this may be the requisite when customers buy socks. Because,due to

the common sense, people will consider the quality of product first before buying.

Moreover, the author assumes that the profit will increase followed by cutting the Endure

manufacturing process. However, "Dura-Socks"-the brand is famous for its feature of endurance.

If people buy Dura-Socks according to this, the move may cause the company losing its

customers. Another disadvantage of this advice may be that the appearance of the product may

be easily copied by their rivals. Then the company may lose their competitive superiority in the

market. At that time, the company may be trapped into the financial problem let alone the

possibility of making profits.

To sum up, the argument that advises to cut the endure manufacturing process is rather weak. If

the author would have considered other hidden factors behind purchasing preference and

proceeded a more well-rounded survey, the recommendation may seem more reasonable and

practical.

In this argument, the arguer claims that the company should stop the Endure manufacturing

process to seek more profit. This conclusion is based on the premises that customers prefer the

appearance of product rather than the endurance. The recommendation's logic is faulty, however,

since it relies on several shaky assumptions to bridge the gap between it premises and

conclusion.

To begin with, the validity of the survey is doubtful. Lacking information about the number of

customers or wears surveyed and the number of actual respondents, and about how the survey

was conducted, it is impossible to access the validity of the results. For example, if 200

customers were surveyed but only 2 of them responded and they are not even the actual wears,

they will not offer exact and valuable information. Or if only leading questions were asked, such

as how often will you buy a pair of sock or which kind of patterns will you choose, etc. Those

are questions that have little relationship with the theme and they may lead customers to echo

with limited answers. Without ruling out these interpretations, the results would be highly

unconvincing.

Secondly, the author falsely assumes the company should devote their money and energy into

designing new appearance of the socks rather than keeping the solid quality of the socks based

on consuming preference and frequency of customers. However, the author may ignore the facts

Page 112: ARG-FINAL

that people may select various patterns or colors of socks to match their formal suits and casual

dresses, or that they are too busy to do laundry then more socks are required. None of them hints

that customers will throw old pairs away after getting new ones. Therefore, good quality of the

socks is still needed, if not,customers may lose their trust on them and turn to other brands. How

can the company gain any money without customers buying it productions. Also, based on the

common sense, the quality of productions is always the prerequisite when people buy things. So,

unless the author rules out such possibilities, this argument is unwarranted.

Moreover, the author also confidently asserts that the profit will increase followed by cutting the

Endure manufacturing process. "Dura-Socks"-the brand is famous for its feature of endurance

and which may the very reason people tend to buy the its product. If the move causes the quality

of socks being slippery, people may turn their back on them which may cost them more than

mere money but also well-established reputation. Another disadvantage of this advice maybe that

the appearance of the product can be easily copied by their rivals. If so, they will possibly lose

their market superiority and fall in trouble of losing rather than gaining profit. Here, the author

neglects the fact of competition.

To sum up, the argument that advises to cut the endure manufacturing process is rather weak. If

the author would have considered other hidden factors behind purchasing preference and

proceeded a more well-rounded survey, the recommendation may seem more reasonable and

practical.

ARG-59

The president of Bower Builders tries to find ways to increase the profits of his company by

presenting the results of a recent survey to his partners. He states that if his company includes in

new homes whirlpool tubs and large kitchens the sales of the houses will be increased. In my

opinion, this article lacks of well-reasoning and the ideas that are provided in order to increase

the profits are over simplistic.

First of all, the competitor Domus Construction that builds new houses at a nearby development

may provide better quality to its customers in overall, except from building large kitchens and

whirlpool tubs. In addition, the competitor may have used better marketing strategies in order to

sell the houses faster and more expensive. Another possible fact is that the high price is a result

of a much better quality that the competitor may have as standard. So, we cannot be absolutely

Page 113: ARG-FINAL

sure that the only factor that causes the high sales for the competitor is the two popular

characteristics of a home.

Furthermore, the area that the Domus Construction builds the new houses may has much more

advantages than the area which Bower Builders build houses. For example, if one of the areas

has better view and more green around then the customers will be very interesting about these

two characteristics and will not care much if a house has a large kitchen or not. In addition,

although the nationwide survey states that people would love to have a large kitchen or a

whirlpool tub in their homes it does not states clearly that they would also pay a bigger amount

of money in order to buy this kind of homes. So, maybe the buyers of houses that Domus

Construction builds are rich people and in general are not easy to find many of them or to

increase the sales only from rich customers.

Moreover, the cost of constructing larger kitchens and whirlpool tubs may decrease the total

profit at the end because the rest of the house should have also high quality in order to be

attractive to a new customer. In addition, the point that by reducing the size of the yards the

companies will increase its profits is completely wrong. This is because we do not know if the

buyers that the article says that do not have complained bought a house inside a city or in the

countryside. For example, reducing the yard in a countryside house may disappoint a possible

customer and though the company will have to decrease its price in order to find a buyer and in

that way the company will lose profits.

Finally, we should emphasize that the president of the company is very narrow-minded in

providing the right solutions to increase the profits of his company. There could be also some

other ways to increase the sales such as better marketing, advertising or to improve the quality of

the houses

To begin with, the president cites a nationwide survey to claim that a large family room and a

large, well-appointed kitchen are the two most-desired home features. However, the result of this

survey should not be considered seriously unless the assumption that this survey is statistically

reliable is verified. If the number and representativeness of those respondents enrolled in this

survey are not statistically significant, all conclusions based on this survey become unsound.

Even if the statistical reliability of the survey is validated, the president assumes further that the

strategy employed by Domus Construction can be effectively applied to Bower Builders.

Page 114: ARG-FINAL

However, this assumption cannot be readily verified. On the one hand, other factors rather than

the proposed two features could account for the higher prices and larger market of homes built

by Domus Construction. For example, it is entirely possible that Domus Construction has an

above-average group of managers who can make better marketing strategies. On the other hand,

homes built by Domus Construction might possess better qualities than the national average and

probably better than that of Bower Builders. In the aforementioned cases, Bower Builders would

probably fail to increase sales and profits by simply mimicking Domus Construction’s situation.

Finally, the president cites their recent buyer’s opinion to support his proposition that their larger

family rooms and kitchens can come at the expense of the dining room. Here, he just simply

assumes that those recent buyers are representative of all their buyers in general. However, this

assumption is rather fallacious. It is possible that a significant proportion of their buyers need a

separate dining room and they value a great deal of the size and design of the dining room.

In the end, the argument turns out to be unconvincing after careful analysis. To strengthen it the

president must provide reliable data verify those assumptions on which the argument is based

ARG-60

In this argument the author recommends that the clients invest in Consodidated Industries whose major business operations is retail sale of home heating oil.To bolster his recommendation tha author point out that the winters of the northeastern United States are very cold and is predicted to continue for the following few years and he also indicates that new aprtments have been built in that district.After a thorough consideration I found this recommendation seriously flawed in several respects.First of all the author groundlessly makes the assumption that the prediction of scholars at Waymarsh University is coming into reality.Yet the climate will probably become warmer because of more settlers along with more factories,emitting greenhouse gas such as carbon dioxide.Without enough evidence the trend of the climate's continuing remaining cold is only prediction.Even if the prediction comes true it is also doubtful that the residents will continue to use the heating oil as their major fuel for heating.As technologies developing fast nowadays it is likely that new resources will come into use in the near future.Or perhaps they will begin to use a new kind of coal or other resource.It is hasty to make a decision without taking other possible factors into consideration.Secondly as the author says many new homes have just been built in that region but not sold out.It remains to be seen whether people will like these houses and if not there will be no profit

Page 115: ARG-FINAL

in investing in the heating oil.Even assuming that the houses have been sold out already . It is possible that buying these houses is just a investment and few people will really live there then will anybody use heating oil?I can not be fully convinced before validate evidence is shown proving that enough number of people will move to these houses in recent years.Finally little statistic is available on the advantages of investing in Consolidated Industries.It is equally possible that other competitive companies may be better choices.I will need more evidence before make a decision.In sum,this argument is unconvincing as it stands.To further strengthen the investment advice the author needs to provide more evidence showing that this kind of bad weather will continue and under such circumstance people will still move into houses built there.Before I make an investment decision the author shoud also proves that heating oil will remain the major fuel and not likely to be replaced in recent years

First of all, the author implies that the demand for heating oil will increase in the future, because most homes in the northeastern United States have traditionally used oil as their major fuel for heating. Thus, the author necessarily assumes that most homes will still use oil as their major fuel for heating in the future. If the assumption is proved unwarranted, most homes switch to using electricity as their major fuel for heating instead of using oil, or people start to use more high fuel-efficiency appliance, then, the demand of oil would not increase significantly and the author’s argument would be weakened.Moreover, the author claims that the demand for heating will increase since last heating season this region experienced 90 days with below-normal temperatures, and climate forecasters predict that the weather pattern will continue for several more years. This claim only make sense on the basis of the fact that the weather in the future will continue to be cold or become colder. If the author’s assumption is proved unwarranted, climate forecasters fail to accurately predict the weather pattern in the future, and the weather becomes warmer due to global warming or other unpredictable reasons, then, people would not need to heat their houses as often, and the demand for heating oil would not increase significantly, thus, the author’s argument would be undermined.

Furthermore, the author claims that it is necessary to invest in Consolidated Industries, since the demand of heating oil will increase. This claim only make sense based on the assumption that new homes will choose to use oil as their major fuel for heating, and Consolidated Industries is the major heating oil supplier for homes in northeastern United States. On the one hand, it is possible that people switch to other heating method instead of using heating oil, and if this alternative possibility is proved valid, then the author’s claim would be weakened because people would use less heating oil, and the demand for heating oil would not increase significantly. On the other hand, if the author’s assumption that Consolidated Industries is the major supplier for heating oil in northeastern United States is proved unwarranted, in fact, it is another company supplying heating oil to northeastern United States, then, investing more money in Consolidated Industries would not help ensure sufficient heating oil for homes in this

Page 116: ARG-FINAL

region, and the author’s claim would be weakened.

To sum up, in order to evaluate this argument, we need to ensure several assumptions: most homes will continue to use heatng oil in the future, and the weather will be cold or colder in the future, and Consolidate Industries is indeed the major supplier for heating oil in northeastern United States.

ARG-61

First of all, some city commissioners suggest that the city should eliminate fundings for the Grandview Symphony, because last year, private contributions to the symphony increased by 200 percent and attendance at the symphony’s concerts has doubled. However, it is possible that these increasing trends will not last in the future, and even if private contributions and the revenue have increased, this money may still not be enough for the Grandview symphony to run independently. Therefore, the first question that needs to be answered is whether the increases in private contributions and the revenue due the doubling in attendance at the symphony’s concerts-in-the–part series will stay high or continue growing in the future. On the one hand, if these increases are unlikely to last in the future, and private contributions are not always reliable, then, eliminating funding for the Grandview Symphony may result in an economic crisis when other funding sources are unavailable in the future, and the recommendation would be invalidated. On the other hand, if these private contributions are proved to be reliable in the future, then, a second question that would need be answered is whether the sum of private contributions and the revenue due to increasing in attendance are enough to run the Grandview Symphony without annual funding from the city or not. If the total funds are sufficient for the Grandview Symphony to run successfully even without the annual funding from the city, then, the author’s recommendation would be strengthened.

Furthermore, the author implies that the Grandview Symphony would be self-sustainable since the Grandview Symphony will increase prices for their tickets, and the revenue should increase correspondingly. However, it is possible that customers may stop going to concerts once the Grandview Symphony increases prices for their tickets. Therefore, the third question need to be answered is that whether the number of customers will drop after the Grandview Symphony increases ticket prices. If the majority of people think increased prices are too expensive, and they prefer to stay at home or go to cheaper places for concerts, then, the number of customers would drop and the revenue might also drop; further, the Grandview Symphony might not have enough money to run without the support from city. For this reason, the recommendation would be undermined.

To sum up, in order to evaluate the recommendation, several questions need to be answered including, whether private contributions and the revenue of high attendance will increase or stay high in the future or not, if private funds and its revenue are enough for the Grandview

Page 117: ARG-FINAL

Symphony to run independently, and whether the number of customers will drop or not after the Grandview Symphony raises ticket prices.

In this article, the writer recommends to eliminate funding for the Grandview Symphony from next year's budget. Based on the evidences concerning the development of the symphony that private contributions to the symphony increased by 200 percent and attendance at the symphony's concerts-in-the-park series doubled, in addition, the symphony has also announced an increase in ticket prices for next year. All these seem to show that the symphony could be fully self-supporting, however, some following questions would need to be answered.

In the first place, the writer seems to assume that with the significant increase of private contributions to the symphony and more attendants at the symphony's concent-in-the-park series last year, the symphony would have sufficient income so that continuing to offer funding for the symphony seems unnecessary. However, can the evidences including percent increase of private contributions and doubled attendance at the symphony's concerts-in-the-park series really substantiate that the symphony could be self-supporting? I'm afraid not. Since there is no information about the fore data of the private contributions to the symphony and the attendance situation. It is possible that after the private contributions increased 200 percent, it was still only a small part to support the symphony. The same thing can be said of the attendance situation. Doubled the attendance to some extent can show the symphony's concerts-in-the-park series was more popular, however, we don't know specific number of the people who attended the symphony before last year, if the number was 2, then doubled it was 4, thus could it mean that the symphony's concerts-in-park-series is popular enough to bring sufficient income to be self-supporting?

In the second place, the writer assumes that the symphony's being more popular leads to an increase in ticket prices, then it would bring the symphony more income, thus it is not necessary to continue to offer budget for the symphony. However, can the ticket prices really mean a lot for the development of the symphony? It is doubtful. First, there is no information about the fore price, if the price was very low that even making an increase in the price, the price is also low compared to many other common symphony, thus even if it may bring more income for the symphony, it is not sufficient for support itself. Second, although to some extent, the higher ticket price may show that the symphony is more popular, however, just as the higher ticket prices, it is possible that some people cannot afford it, thus may on the country make the symphony be less popular, which would have a negative effect on the future development of the symphony.Finally, even if the current situation of the symphony is optimistic, however, who can guarantee that the symphony would continue to be going well? Who can ensure there would be no negative effect of eliminating funding for the symphony? It is very possible that people don't like the symphony overnight and if there is no market, there would be no income source, thus even if the symphony can be self-supporting today, it maybe even cannot make any money tomorrow.

Page 118: ARG-FINAL

Furthermore, if eliminating budget of funding for the symphony, people may be suspicious. On one hand, they may suspect that the city commission has some financial problems. On the other hand, they may guess that the Grandview Symphony has some problems even it would be dissolved. All these have a negative effect on the development of the symphony. In conclusion, can the evidences that 200 percent increase of private contributions to the symphony and double attendance situation and would-be higher ticket prices really substantiate that the symphony is popular enough to make a lot of money and the money would be sufficient to be self-supporting? Who can guarantee there would be no negative effects after eliminating the budget of funding for the symphony? The answers to these questions above lead this recommendation to an unconvincingly reasonable one.

ARG-62

In this argument, the author asserts that UltraClean should be supplied at all the hand-washingstations throughout the hospital system to prevent serious patient infection. To support it, hecites that a laboratory study shows UltraClean produced a 40 percent great reduction in thebacterial population than did other liquid hand soaps. In addtion, a subsequent test of UltraClean at the hospital in Workby which reported fewer cases of patient infection than anyof the other hospitals is provided. After careful scrutiny, I find that the argument suffers severallogical flaws.To begin with, the UltraClean in the laboratory study is a concentrated solution. So thereduction in bacteria population may due to the extreme high concentration. As a commonsense, with the increasing of the concentration of the liquid hand soap, it will correspondinglymore active and effective in killing or decreasing the number of bacteria. The author assumesthat the UltraClean used in the hand-washing station will be the same condition. What is more,other kinds of liquid hand soaps may also increase the capability of reducing the bacteriapopulation when they are more concentrated. So without controlling the concentration of theliquid soap, the assumption that UltraClean is better in reducing the number of the bacteriapopulation is unconvincing.Secondly, in the subsequent test of UltraClean in the hospital in Workby may be in the absenceof significance of statistic and representation of all the hospitals. No evidence is supported thatthe situation and condition is the hospital in Workby is similar to other hospitals. Perhaps intheir hospitals, the number of patients are small and before using the UltraClean the populationof bacteria in this hospital is less than other hospitals. More it is probably the nursing standardis higher and better than others, or patients in their hospital are trained with the protectionagainst the bacterial infection and pay more attention to the sanitary problems. As a result,there are fewer cases of patient infection. So without ruling out other factors influencing thepatient infections, the test in one hospital is groundless in supporting the final conclusion.

Finally, the author assumes that UltraClean is significant in reducing the serious infection isunwarranted. In the argument, there is no information provided that this hand soap is effectivein preventing the serious infections. Maybe the decreasing in patient infection in the test

Page 119: ARG-FINAL

islargely from the slight patient infection. And even it can reduce the serious infection, noevidence is offered to ensure that there will be no side effects with UltraClean. Usually when asoap strongly fight with the bacteria, it may also do harm to the human skin, which may lead tothe irritation, allergy and corrosion. So if it is used in all the hand-washing station, it bringsmore healthy problems instead of reducing the patient infection.In sum. The argument is unjustified with several unwarranted assumptions at the best. Tobolster it, the author should take the concentration which influences the property of reducingthe population of bacteria into consideration. And more information and dates about thehospital condition is given to prove that UltraClean is suitable and effective in the seriousinfection in all the hospital. Finally, the potential side effects of the hand soap needed furtherstudy before widely used in all the hand-washing station. The following appeared in a memofrom the director of a large group of hospitals.

The speaker argues that UltraClean should be supplied at all hand washing stations throughout

the hospital system to prevent serious patient infections. To bolster the conclusion, the speaker

suggests a laboratory study of liquid antibacterial hand soaps which has a solution of UltraClean.

The speaker also shows a subsequent test of UltraClean at one hospital in Workby, where gave

rise to the effect of the solution. It seems so plausible at first glance; nevertheless, there are

several weak and porous analyses which lead the argument to the flawed one.

In the first place, the speaker overlooks other variable which could affect the laboratory study.

What if, for example, people assigned to the experimental group used the antibacterial hand

soaps more frequently that other people who used just normal soaps? In addition, depending on

how well each group of people was controlled or treated toward the infection, the result could be

different. In order for the argument to be accepted, the speaker should show that the experiment

was well controlled enough to rule out the effects of other variables.

In the second place, the argument assumes that a test of UltraClean in a hospital in Workby could

represent the whole conditions of other hospitals. However, there is no comparison of other

manipulations or controlled conditions between the hospitals in Workby with other areas. It is

quite possible that other medical treatment to protect infection was better performed in Workby

or the environmental conditions such as temperature or moisture was better in Workby during the

test period. Without nullifying such a myriad of possibilities which could affect the infection

results, the argument cannot be justified.

In the third place, the argument assumes that the whole patients could have the same conditions

and the same immune systems toward any infection. However, such assumption is flawed

Page 120: ARG-FINAL

because each individual has different immune system and the treatment for each of them should

be differently designed. In addition, the argument overlooks the possibility that other

antibacterial soaps or procedure could better work for patients. Since there is no additional

analysis about other procedure or chemical products, the argument cannot be accepted.

In conclusion, the argument is biased and not reasonable. To strengthen the argument, the

speaker should show the lab study was well conducted enough to reject other variables. In

addition, there should be a critical data that the condition of Workby quite well represents that of

other hospital in different areas. Moreover, in order for me to accept the argument, the speaker

should present more reasonable evidence that the soaps of UltraClean can work for every patient,

and there is no better way than using the soaps.

ARG-63

First of all, the author suggests that Parkville should discontinue organized athletic competition

for children under nine since, last year, over 40,000 of these young players suffered injuries.

Thus, the author necessarily assumes there is a correlation between children starting to

participate in youth-league sports at the age of nine and the fact that over 40000 of these young

players suffered injuries. In other words, the author assumes that the large number of injuries is

due to organized athletic competition for children under nine. If the author’s assumption is

unwarranted, and the majority of injured players actually did not partcipate in youth-league

sports under the age of nine, or it is due to there being an increasing trend of youth players who

start to join sports teams in middle school, or it is due to the fact that sports with intense physical

contacts are increasingly popular in recent years, which is not associated with organized athletic

competition for children under nine, then, the author’s claim would be weakened.

Moreover, the author also mentions that organizing athletic competition of children under nine

contributes to the reported psychological pressure from youth-league soccer players. This only

make sense based on the assumption that these players actually participated in sports teams under

the age of nine. If they join their teams above the age of nine, then, the author’s suggestion that

discontinue organized athletic competition for children under nine would not help solve the

problem, and the claim would be weakened. Even granted that these players participated in

competition below the age of nine, the author also assumes that the reported pressure due to

sports activities is significant enough to cause negative psychological problems. Even academic

activities can possibly cause pressures on students, besides, pressures are not always negative for

Page 121: ARG-FINAL

students, they sometimes can motivate students, too. If the assumption is unwarranted, and these

pressures are shown to be positive pressures that could encourage students’ performances, and

help students develop their abilities of dealing with pressures, then, this could weaken the

author’s claim.

In addition, the author suggested that sports activities are taking the time that students can use for

academic activities. Thus, the author necessarily assumes that if children stop spending time on

sports, they will use that time for academic activities. If this assumption is unwarranted, and

when students do not spend time on sports anymore, they use their time on playing games or

other recreational activities instead of do their academic work, then, the author assumption is

weakened.

To sum up, in order to evaluate the argument, we need to ensure the following assumptions:

organizing athletic competition for children under nine attributes to the majority of sport injuries,

reported pressures among players are negative pressures, and students will invest more time on

academic work when they stop practising in sports teams.

In this letter ,the author makes the suggestion that Parkville should discontinue organized athletic

competition for children under nine. The author points out that over40000 of young players

participated in athletic competition suffered injured. He then notes that according to a recent

interview, many youth-league soccer players face more psychological pressure. Furthermore,

education experts’ opinion, that long practice sessions take away children’s study time, is also

cited in this letter. The author takes all these factors as evidence to show the disadvantages of

athletic competition, and to support his suggestion. As discussed below, the letter suffers from

several critical flaws and therefore unpersuasive.

First of all, the author observes a correlation between the increasing number of children

undernine participated in youth-league sports and over 40000 of these young players suffered

injuries, and then makes the former one to be reason for the latter. However ,he ruled out other

causes for these injuries without any evidences. Perhaps the injured children are mostly 9 plus,

and they get injured since their hot temperas teenagers. It is also can be true that the number of

youth who get injuredare equal, even less, compared with the number when few children under

9participated.

Secondly, evenif more youth get injured with more children under 9 participated, the

Page 122: ARG-FINAL

authorcannot assume that the factor runs well in Parkville, since the factor is concluded by the

nationwide survey. Perhaps factor is just the opposite in PARKVILLE as youth players were

better trained than any other places.

Thirdly: the author assumes that young players are stressful based on a recent interviews. But the

interview is far from enough to be the evidence. At least, the author should prove that the

interviewers can represent all young players. Perhaps the interviewers were coincide with youth

who had received more expect from coaches and parents, or the pressure problem is unbearable

only for soccer players. What’ smore, the interview was undertaken in some major cities. Young

players’ pressure may result from fierce competition in these cities.

Fourthly: the claim that long practice sessions take away study time relies on the assumption that

the education experts referred in the letter own enough authority to convince people that what

they said is reliable. Even if these experts are authorities, this needs not to be the case. Perhaps,

outweigh any advantages is logically unsound. Disadvantages or advantages cannot be

calculated, even if more disadvantages can be listed than advantages, the bad influence can be

little compared with healthy effect. Perhaps study time was taken away but children study much

more efficient than ever before.

In summary, the suggestion is unconvincing as it stands. To make a reasonable suggestion, an

all-round investigation aimed at Parkville is required.

ARG-64

The conclusion made by the author is that, the collectors should expect the value of the life-size sculptures to depreciate and the value of the miniatures to increase. The conclusion is made on several unreasonable assumptions for which there are no clear evidences to state.

First of all the author assumes that, the kalinese artists would use the molds that were found recently by the archeologists for making the ancient life-size clay statues. Additionally, Kalinese artists did not use any sculpting tools and techniques. The author has to consider several other elements to strengthen the assumption for the argument. Enough evidence has to be provided about the statues that were created before the molds were discovered by the archeologists. It could be that kalinese artists are too talented to use any molds or sculpting tools and techniques to make life-size clay statues. Additionally, the molds that were found by the archeologists may be not completely related to the art tradition maintained by the Kalinese artists. Or It could be that, archeologists found the molds on a different island. The author has to cite strong evidences about before and after the discovery of molds and enough information about the location where the molds were discovered and thus the argument would be strengthened. 

Page 123: ARG-FINAL

Secondly, the author assumes that the discovery about the molds reveals the style with which the miniature statues are made. It also explains about the discovery of ancient sculpting tools. It could be that the kalinese artists maintain different styles in creating miniature and life size sculptures. For example, it could be that, even if the ancestors used many techniques in creating statues, the kalinese artists have adapted to a different style as the generations passed by. A style, which may not use any sculpting tools, but still achieves realistic precision. The author has to probe more into the ancient tradition including the kalinese contemporary tradition to support the assumption made by the author for the argument.

Finally, the author assumes that the life size sculptures value would be decreased and the miniature statue values would be increased. It depends on many factors how the collectors would evaluate the work involved in creating the statues. It could be that the collectors would like more of the statues made by the molds than having a realistic touch. It could be that collectors might like any kind of statue that is made out of clay, which has entirely different and unique style to it. The author has to evaluate the collectors choice and preference in choosing the statues to make the assumption strong for the argument.

The argument would be strengthened if the author can evaluate the discovery made by archeologists and its relation to the work of Kalinese artists on the Kali Island, including elements like style, tradition maintained before and after the discovery. Additionally, knowing the collectors preferences would be beneficial to evaluate the argument. 

Firstof all, the collectors conclusion based on the discovery referred in thearticle is logically unsound. They provide no evidence to show that the moldsof human heads and hands were used by artists. Perhaps the molds werefunctioned as teaching tools. Even if the molds were tools of artists, thediscovery is far from enough to prove that the artists create without anysculpting tools or techniques. Perhaps tools and techniques were the mainreason for the precision since they were necessary in details.         Secondly,whether the artists created sculptures only with molds is uncertain accordingthe first discussion above, the discovery of molds may not be the reason forthe few discoveries of sculpting tools. Perhaps the sculpting tools were madeof metal which can be easily corroded.        Thirdly,even if the artists created scriptures only with molds, the collectors unfairlyrule out other reasons for miniature statues’ different styles. Perhaps theminiature statues were made for decoration, and kalinese were in favor of suchabstract and varied statues.         Finally:even if the collectors’ conclusions and explanations are coincidently same withthe truth, their confidence in the value trends of the life-size sculptures andthe other is lack of reasonable analysis. Perhaps life-size sculptures will bemore valuable since more people will cherish realistic precision articlesinstead of abstract ones in the future.         Inconclusion, to make the suggestion convinced, the collectors must provide moreevidences from authorities’ study to prove their conclusion. Furthermore, areliable survey is required to assure that articles with more techniques willobtain more favor of market.

Page 124: ARG-FINAL

ARG-65

The contention that the Stanley park can be made as popular as Carlton park, by providing more benches and thereby converting some of the unused open areas into spaces suitable for socializing, is not based on thorough and logically convincing platform as it makes certain critical assumptions. The following essay will expose these flaws and demonstrate how the essay could be made more cogent.

To begin with the argument implies that the people who come to the park use only cars to get to the park, but, there may be people who just come on foot to the park. These people may not be captured on the surveillance cameras that are located in the park's parking lot. Therefore the number "50" may not be the accurate number of people visiting the park.

Also, the fact that the location of the Stanley park might also have reduced the number of visitors is not considered by the argument. It is clearly mentioned in the argument that Carlton park is located in the heart of the business district, but, the essay never mentioned about the location of Stanley park. Since Carlton park is located in the heart of the business districts, it is more than likely that the employees of those companies would visit the park during their lunch hours, just to relax. {The words (quote from the argument)"... on a typical weekday" bolster this statement}. This might have contributed significantly to the higher traffic at the Carlton park. Also, the argument never mentioned about how accessible the Stanley park is. The higher traffic at Stanley park when it was first opened might be attributed to being the one and only park in that area. But now, since Carlton park is more accessible, even though it is smaller in size compared to the Stanley park, people might be preferring to spend time in the former more than in the latter.

Also, the argument never considers if either park allows pets. Some people might go to a park just to play with their dogs. Perhaps, a glaring omission is the fact that, the argument never considers the general atmosphere in both the parks. Few people would like to relax in a calm and serene atmosphere in a park. Also, the neatness in the park is anothe factor that might attract people and this was never addressed by the argument. Another important aspect is the availability of public restrooms and drinking water fountains, which is not taken into consideration by the argument.

So, the argument does have its inadequacies. The fact that just by increasing the seating areas in the Stanley park would attract more people to the park, is not cogent enough as it fails to address the afore mentioned assumptions.

To conclude, the argument could have been bolstered by providing a general overview of the locations of both the parks, by mentioning the accurate statistics of people visiting both using cars and on foot, an accurate estimate of the traffic at Carlton park on the weekends, an overall description of the amenities in both the parks

Page 125: ARG-FINAL

Firstly, popularity of each park is compared with another using different measurements. Popularity of Stanley Park is measured using the number of cars parking at its parking lot, whereas popularity of Carlton Park is measured using the number of people visiting the park. Comparing different entities result in confusion rather than conclusion. Also in a park, most probably not all of the visitors coming by a car, more over the proportion of visitors coming by a car is not stated in the argument, so nobody can conclude about the number of visitors using number of cars parking in the Stanley park.

Secondly, after concluding that Stanley Park is less popular than Carlton Park using an ill constructed evidence, argument finds out the reason making Carlton parking more popular. Even if it is assumed that the first claim of argument, which is Carlton Park is more popular than Stanley Park, is true, second claim of argument about what makes Carlton Park more Popular is not rational. People do not usually come a park to seat, they usually come a park to make activities such as jogging, walking with their pets etc. Also there is no evidence about the condition of seat places in the Stanley Parks. If seating places in Stanley Park is partially empty then there is no reason to think that its unpopularity due to the lack of seating places.

First of all, the author suggests that the popularity of Stanley park dropped, since the video camera reveals that an average of 50 cars parked in the parking lot per day. Thus, the author necessarily assumes that all visitors drive to the park. If the author’s assumption is proved unwarranted, and many visitors walk or bike to the park instead of driving, then, using the parking data recorded by video camera to estimate the number of visitors would not be accurate, and there might be more visitors than the data suggested, the popularity of Stanley park might not dropped significantly, and the author’s argument would be weakened. 

Further, the author mentions that Carlton Park is more popular than Stanley Park, since there are more cars in Carlton’s parking lot. This only make sense based on the assumption that more cars reflects more visitors. Even if it is granted that the majority of visitors drive, there being more cars in Carlton Parking lot does not necessarily mean that there are more visitors to Carlton Park, since it is located in the business district and, thus, it is possible that people just park there instead of visiting the park. If the author’s assumption is proved unwarranted, and many people park at Carlton Park because they cannot find a parking lot nearby, then, there might not be more visitors going to Carlton Park, and the author’s argument would be undermined. 

In addition, the author also claims that the reason why Stanley has fewer visitors is that it does not have enough seats compared to Carlton Park. Thus, the author necessarily assumes not enough seats is the only factor that results in the drop in the number of visitors if it drops at all. If the assumption is unwarranted, it is shown that visitors think it is too far to drive to Stanley park, or there are fewer people who want to go to parks now, then, providing more seats would not help Stanley park attract more visitors, and the author’s argument would be weakened.

Page 126: ARG-FINAL

To sum up, in order to evaluate the argument, we need to ensure the following assumptions: the majority of visitors drive to the park, more cars indeed reflects more visitors, and not enough

seats is the only reason the results in the drop of visitors. Finally, the author also makes a

tenuous assumption that Calton Park is comparable to Stanley Park. Perhaps the position of two parks is so different that two parks have totally different set of target customers. For example, Stanley Park is a place especially famous for its natural environment, so that people who like to get away from the noisy city and bask in a peaceful time, while Calton Park is a social place for business people. Although providing enough benches in Calton Park is a big merit for it, this approach could have insignificant effect on Stanley Park, since people don’t come to talk with others anyway. Therefore, without the detail information about the characteristics of two parks, the argument seems unconvincing.

ARG-66

The memo given by the business manager of the chain of cheese stores proffers to discontinue

stocking of the varieties of imported cheeese and concentrate solely on the sale of domestic

cheese. The manager has arrived at this decision based on the best-selling domestic cheddar

cheese noted to gain popularity in the recently introduced cheese stores. However, the argument

relies on many unstated assumptions and the argument cannot be taken for granted unless these

assumptions are validated with proper evidence. Thorough analysis of the assumptions is

mandatory to evaluate the concerned argument.

The manager quotes that the stores in his chain have stocked a wide variety of domestic and

imported cheese. It assumes that both of the varieties are equally significant in number and

distributed evenly thoroughout all its stores. However, the particular distribution of each type of

cheese in each of the stores is mandatory to arrive at a clear-cut picture of the prevailing scenario

in cheese stores. It is almost impossible to judge the level of distribution of cheeses just by the

term "wide variety". Hence it has to be quantified to get a clear understanding.

Next, the manager assumes that the five-best selling domestic cheddar cheeses in Wisconsin had

no tough competition with other types of cheese. For instance, if any other kind of imported

cheese just falls second to the sales of this cheddar cheese, then it cannot be neglected and there

is a huge possibility that it will surpass the current popularity of cheddar cheeses. Thus, a

comparison needs to be established to prove without doubt that the five-best selling domestic

cheddar cheeses deserve their position. If this fact is not proven, then it is impossible to assume

that Cheddar cheeses of this type will ensure high sales.

Page 127: ARG-FINAL

The manager quotes the results of survey conducted by the World Magazine. This survey

particularly indictates the preference of the subscribers towards domestic cheese. It does not

however guarantee that the subscribers will buy this type of cheese. Hence the price levels of the

domestic cheese must be proved to be affordable for the subscribers to opt for this type of

cheese. And the group interviewed by the survey must be established. Only if the survey

interviews a wide variety of population, can we come to a conclusion regarding the validity of

the survey.

Finally, the manager is keen on reducing expenses by bringing to control, the inventories of the

company. Hence he suggests discontinuing stocking of many varieties of imported cheese. But,

the manager has initially quoted that the imported cheese has been stocked for many years

together. Therfore, to determine whether the imported cheese can be discontinued, without

incurring huge losses is a question. Moreover, it is not established that the domestic cheese will

sell in the desired manner. Consequently, it is preposterous to come to a conclusion that profit of

the stores will be magnified to a great extent.

Above all, the popularity of some of the domestic varieties of cheese is noted only in the newly

introduced stores. This leads to the doubt whether the same level of popularity for the domestic

variety will be achieved when introduced in all of the existing cheese stores. The huge profits in

one of the stores is correlated with similar kind of profits in other stores. It assumes the same

nature of stores, the income of customers, the favorable location of the stores to be applicable for

all of the stores. These have to be established with proper evidence.

Thus the argument put forward by the manager rests on lots of stated and unstated assumptions

like: wide variety of two types of cheese to be equally distributed in all of the stores, the validity

of the survey, the affordable price levels of the domestic Cheddar cheese, no losses incurred on

discontinuing imported cheese, the same level of popularity to be achieved in all of the cheese

stores as observed in the newly introduced stores. These assumptions have to be substantiated

with proper evidence. Otherwise, the suggestions put forward by the manager will turn out to be

futile and serve no purpose. Therefore, it is of utmost importance to delve into the intricacies of

the argument and solve the contradictions before considering the manager's suggestions in the

memo for implementation.

First of all, the author mentions that the chain of cheese stores should discontinue stocking varieties of imported cheeses since the five best-selling cheeses at their newest store were domestic cheeses. Thus, the first question that needs to be answered is whether the sales in their

Page 128: ARG-FINAL

newest store reflect the sales of cheeses for the whole chain stores. It is possible that the outstanding sales of domestic cheeses is due to local popularity, and the sales of domestic cheeses might not be high in other stores; then, domestic cheeses might not be as popular as the author implies, and the author’s recommendation would be undermined. Even if domestic cheeses are the best selling cheeses, then, the second question that needs to be answered is whether the sale of domestic cheeses is the significant portion of the total sales; in other words, whether domestic cheddar cheeses make up the significant portion of the total sales or not. It is possible that the sales of all domestic cheese is less than the total sales of the imported cheeses; then, the author’s argument would be weakened. If the sale of domestic cheeses is a significant portion of the total sales, then, the author’s recommendation would be strengthened.

In addition, the author suggests that not stocking imported cheeses can improve profits. It is possible that selling one piece of imported cheese is more profitable than selling the same amount of domestic cheese. Then, even if the sales of domestic cheeses are high, selling domestic cheeses alone may not be as profitable as selling imported cheeses. Therefore, the third question that needs to be answered is if selling domestic cheeses is more profitable than selling the same amount of imported cheeses. If it is shown that the profit of unit sale of domestic cheeses is much higher than that of imported cheeses, then, the author’s recommendation would be strengthened.

Futhermore, the author mentions that not stocking imported cheeses can improve profits since it can reduce stocking expenses. It is possible that this chain store is known for its large selections, and customers like this chain stores because of their varieties. Thus the fourth question that needs to be answered is what is the major feature of this cheese chain store that attracts customers. In other words, will discontinuing selling other kinds of cheeses result in the drop of customers. If people value the variety of cheeses more, then stop stocking imported cheeses might potentially result in a loss of customers, and a loss of customer might lead to a decline in profits; thus, the author’s recommendation would be undermined. 

To sum up, in order to evaluate the recommendation, several questions need to be answered: whether the sales of domestic cheeses is a large portion of the total sales; whether selling imported cheeses is more profitable than selling domestic cheeses; and whether stocking less imported cheese can result in a loss of customers or not.

ARG-67

The author concludes that the only way to increase and attract moviegoers is to offer the same features as Apex, but this conclusion is not the real solution. According to the quotation, what we know is that many peoplo decrease the times to see movies because of the expensive ticket prices, not for the poor or old features of the theater; we don't know is that how people respond to the luxury equipment of Apex. All these details makes the conclusion unconvincing and

Page 129: ARG-FINAL

digress from the real problem and solution.

Although the new-opened movie theater, Apex, is probably the most competitive opponent of Rialto, but there is no statistics about how many people have choosed to see movies at Apex instead of Rialto; maybe the amount of audience is poor. Since there is no evidence showing that Apex attracts more people, there is no reason that Rialto should adopt all features of Apex.

Moreover, the locations of two theaters may be the more important factor than equipment. Rialto is in the downtown, where is a excellent place to run entertainment business because there are many customers. On the contrary, Apex is in the mall outside of town, the location brings it not as many people as downtown does. In this respect, Rialto has the advantage to attract more audience.

Furthermore, according to the survey, most people said that they didn't go to movie theaters so often because the price is too high. If the result of survey is correct, which means that what makes people stay away from movie theaters is not the equipment but the ticket price. Therefore, there is nothing to do with how good the features of theaters are. If Rialto tries to attract more audience, the most effective way may be lowering the ticket price, but not investing money on expensive equipment which may force Rialto to reflect the cost to the customers - raising the ticket price further.

To sum up, the author jumps to conclusion with insufficient investigation; what Rialto should do to bring people back to movie theaters and create profit for itself is not blindly following Apex. Before Rialto takes any action, it really should dig deeper to find out the real reason of its bad profits, and strong evidence is needed to support that to be like Apex can make Rialto earn profits.

Merely based on the unfounded assumption and dubious evidence, the author recommends that Rialto Movie Theater, a local institution for five decades should follow the example of the new Apex Theater. To substantiate the conclusion, the author points out evidence that when opened last year, Apex featured a video arcade, plush carpeting and seats, and a state-of-the-art sound system. In addition, a result of a recent survey is cited to support the analysis. At first glance, the argument appears to be somewhat convincing, however, a further reflection reveals that it omit several crucial concerns which should be addressed to substantiate the conclusion. In my point of view, the author suffers from cause and effect, insufficient investigation and analysis, and false analogy logical flaws. Firstly, the author fails to establish a causal relationship between the feature of Apex and how it holds on to its share of the pool of moviegoers. What’s more, no evidence in the argument suggests that, it is the features of a video arcade, plush carpeting and seats, and a state-of-the-art sound system that bring about success and actual profit. Thus, this argument is unacceptable unless there is compelling evidence to support the connection between the luxurious equipment

Page 130: ARG-FINAL

and the movie theater’s success. Perhaps, for example, the success and profit of Apex, if there is any, result from the advantage of location and its inner management. Secondly, even assuming these sumptuous features bring actual profit and success to Apex, the author’s recommendation relies on what might be a poor analogy between Apex and M. The analogy falsely depends on the assumption that all scenarios between the two are similar, such as, the consumption level of the residents, citizens’ consciousness, and even the policy of local governments. However, it is entirely possible that situations of the two share no common ground. In short, without accounting for important possible differences between Apex and M, the arguer cannot reasonably prove the proposed method will help M to hold on to its share of a decreasing pool of moviegoers.Finally, unless the surveyor had sampled a sufficient number of the residents and did so randomly across the entire spectrum, the result of the survey is not reliable to gauge whether the pool of moviegoers is decreasing. The proportion of the respondents in itself does not ensure the representativeness. For example, if the sample only included the poor and the students, it would not doubt that they want to save money for other purpose. Or if the samples account for only a little percentage of the citizens, the result will also be meaningless.To sum up, the author fails to demonstrate his claim that offerring the same features as Apex will help M to hold on to its share of a decreasing pool of moviegoers, because the evidences cited in the analysis do not lend strong support to substantiate that it is the features of a video arcade, plush carpeting and seats, and a state-of-the-art sound system that brings about success and actual profit. Additionally, the author would have to what the author maintains. To make the argument more convincing, the author would have to provide more information with regard to the situation of the two places. Also would be useful is the information about the carriage of the investigation. Therefore, if the argument had included the given factors discussed above, it would have been more thorough and logically acceptable.

ARG-68

The argument that pet owners have longer, healthier lives than people who own no pets ensured by dog owners tend to have a lower incidence of heart disease will substaintally alleviate the current problem of adopt pets from shelter, which will reduce the risk of heart disease in the general population. however the conclusion relies upon the assumptions for which their is no clear evidence.

Firstly the writer assumes that a recent study reported that pet owners have longer, healthier lives on average than do people who own no pets. Infact the writer ignored the starting data regarding the recent study. who did that study ? How many people involved in this study ? is they make two different group of people, one those have pets and other one group don't have pets ? those people who don't have dogs are they doing exercise or have the same daily routine as the people

Page 131: ARG-FINAL

have dogs? how many days that study going on ? their is no clear evidence of this assumption so it seems irrational.

furthermore, the writer assume that dog owners tend to have a lower incidence of heart disease. Infact he doesn't show any analytical result or there is no credible evidence to show the lower incidence of heart disease. He doesn't tell us the eating habit of those people who lived under observation, Heart disease concerns with cholestrol, eating more iron as well as calcium food. Heart deases is related with stress and high blood pressure and less exercise. It seems irrelvent to the assumption.Finally the writer conclude that adopt - a- dog programe from sherwood animal shelter will encourage more people to adopt pets, which will reduce the risk of heart disease in the general population. he doesn't tell us why we adopt a dog only sherwood animal shelter ? is these dogs trends under special training. He doesn't have clear evidence what is the corelation between dogs and reduce the risk of heart disease.so the writer have no statisatical evidence to show his own conclusion.The writer would not be wrong to conclude that adopt - a- dog programe from sherwood animal shelter will encourage more people to adopt pets, which will reduce the risk of heart disease in the general population. In order to support the current conclsion the writter must first define the scope of the problem more clearly and demonstrate a more complete understanding of the need for and benefits of the new requirement

If this assumption is proved to be unwarranted, and some patients with heart disease feel very upset when dogs bark, or only dogs with special training can indeed help patients recovering from heart disease, and the majority of dogs in the shelter are not trained to be used for treatment, then, in these cases, encouraging patients who cannot tolerate barking or noise to adopt these untrained dogs might make their heart disease even worse, and the author’s argument would be weakened.

ARG-69

First of all, the author recommends Zeta over Alpha since the building built by Zeta years ago seemed to have higher quality compared to the one built by Alpha. However, this can only show that Zeta constructed a better building than Alpha ten years ago; it is not convincing enough to predict that Zeta will still do a better job today. It is possible that during this ten years, due to the poor management, the quality of Zeta’s building has dropped, whereas Alpha now can construct buildings with better quality. Thus, the first question that needs to be answered is whether Zeta consistently has a better reputation for their buildings than Alpha does. If Zeta consistently has a good reputation for their buildings, then, it would strengthen the author’s recommendation. However, if the evidence shows that the quality of Zeta’s buildings has dropped, and their qualities are not as good as Alpha’s buildings, then, the author’s recommendation would not be reasonable.

Page 132: ARG-FINAL

In addition, the author implies that the building built by Zeta is better than Alpha’s building since the building’s expenses or maintenance last year were half those of Alpha’s, and the energy consumption of the Zeta building has been lower than that of the Alpha building every year since its construction. However, it is possible that the maintenance cost for the Alpha building was only high last year due to internal renovation, while in other years, the maintenance costs for the Alpha building were low. Considering the energy consumption, it is also possible that people did not heavily use Zeta building, and there are less people work in Zeta building. Both alternative possibilities can contribute to lower energy consumption. Therefore, the second question needs to be answered is do people heavily use both buildings, and were the maintenance costs for Zeta building consistently lower than those of Alpha. If it is shown that the maintenance costs for Zeta building were consistently low, and people use Zeta building as heavily as they use Alpha, then, the recommendation would be reasonable. Otherwise, the author’s recommendation would be undermined.

Finally, the author recommends Zeta over Alpha, because Zeta has a stable workforce with little employee turnover. However, a stable workforce with little employee turnover does not necessarily guarantee Zeta can build better buildings. Thus, the third question needs to be answered is whether employees are more well-trained or professional in Zeta compared to those in Alpha. Although Zeta has a stable work force, if Zeta workers are not well-trained or more professional, they might not do a better job than Alpha. For this reason, the recommendation would be inappropriate.

To sum up, in order to evaluate the recommendation, several questions need to be answered, does Zeta consistently have a better reputation for their buildings, do people use Zeta building as heavily as Alpha building, and are workers in Zeta more well-trained and professional than those of Alpha.

ARG-71

In this letter, the author recommends that Waymarsh should implement a similar policy like

Garville's that rewards people who share rides to work and give them coupons for free gas. To

bolster the recommendation, the author points out that pollution levels in Garville have decreased

since the policy was implemented and people from Garville said commuting times have dropped.

However, the argument is rife with holes. In order to make the argument to be more cogent and

persuasive, several evidences should be added.

Firstly, the author provides scant evidence that the commuter took 20 more minutes to work

necessarily indicate that traffic here in Waymarsh is becoming a problem. It is entirely possible

Page 133: ARG-FINAL

that the city government has taken a policy that decrease the speed limit during these three years,

thus the commuters would take more times to work. If the government increases the speed limit,

the commuter would go to work fast. Or perhaps the survey was just completed during the

summer, in which increasingly numbers of people come to here for tourism. Hence, this

phenomenon leads to the traffic jam. Maybe after the peak season, the traffic will recover its

order. For both matter, the author's recommendation that address the traffic problem is

unnecessary.

Even if the traffic in Waymarsh is becoming a serious problem, we are not formed that the

relevant circumstances in nearby city Garville is same with that in Waymarsh. The decreased

pollution levels in Garville may be not resulted from the policy. Since we do not know what is

the kind of pollution which has been decreased, may be it is just the level of water pollution has

dropped, other than air pollution which is nothing to do with traffic. Even if it is air pollution,

that has been decreased, maybe it is because there are more trees in this city that the air becomes

clearer. Thus, the author cannot conclude that the policy in Garville is successful.

Furthermore, we need the evidence that the actual number of people who tell the author. Maybe

it is too slight, and cannot be representative of all commuters in Garville. Perhaps much more

people think the policy make them take more times to work since their bad bicycle skill. Even if

the commuters in Garville were take less times to work, it do not demonstrate that the people in

Waymarsh have the same respondent. It is entirely possible that there are large different between

the two cities. Maybe the size of Waymarsh is bigger than of Garville, and commuters in

Waymarsh may take far more times such as 2 hours to work by bike while half an hour by car.

Thus the people still would like to work by car. Without considering and ruling out these

possibilities, I remains unconvinced that a policy like Garville should be equally well in

Waymarsh.In summary, in order to make the recommendation to be more persuasive, the writer

should provide evidence that he commuter took 20 more minutes to work necessarily indicate

that traffic here in Waymarsh is becoming a problem, and that the relevant circumstances in

nearby city Garville is same with that in Waymarsh. Otherwise, the author's assertion is

untenable

First of all, the author mentions that traffic in Waymarsh is becoming a problem, since the typical commuter took 40 minutes to get to work compared to 20 minutes three years ago. To test the veracity of this claim, we need evidence about the relative distance of a typical commuter needs to commute now, and compare the distance to that of three years ago. It is possible since

Page 134: ARG-FINAL

more and more people start to drive, the city has expanded, and people tend to live farther away from their work places. If the evidence shows that the typical distance for people to go to work is much longer than it was three years ago, then people spending more times in their cars is not because of the heavy traffic; instead it is because they need to travel longer distances to work; in that case, the author’s claim would be weakened. Whereas, if the evidence shows that people still leave close to work nowadays, then, it might because the traffic is heavier now, and the author’s claim would be strengthened. Moreover, the author suggests that Waymarsh should encourage people car pooling because the pollution levels have dropped since the policy was implemented. In order to verify this claim, we need evidence about what kinds of pollution have been improved, and whether Garville has also implemented another policy that could possibly contribute to less pollution. It is not specified in the argument what kinds of pollution have dropped; thus, if the evidence shows that the water pollution was improved, which is not related to cars at all, then the pollution levels in Garville might not have dropped due to implementing the new policy, and the author’s argument would be weakened. Even if the evidence shows that the air pollution has been improved, then, we would need evidence about whether Garville implemented other policies that can potentially contribute to improving air pollution at the same time. If it is clear the Garville has not done anything that can contribute to improving air pollution except the carpooling policy, and we can rule out any other possibilities, then it is likely that the new policy might help reduce the air pollution level, and the author’s argument would be strengthened.

Finally, the author also mentions that people reported that their commuting times have fallen due to the new policy. To test the veracity of the claim, we need evidence about whether people reporting that their commuting times have fallen is due to less heavy traffic or other reasons; in other words, do they still go to work at the same time as they did before. If the evidence shows that considering there are more people going to work at the same time, and people tend to leave to work earlier than they did before, then it would be possible that it takes less time now is because people tend to go to work earlier and avoid the rush hour, and the author’s argument would be weakened.

To sum up, in order to evaluate the argument, we need evidence about the distance people commute now and compare that to the distance three years ago, and did Garville implemented other policies that can contribute to improving pollution levels or not, and do people leave for work at the same time as they did before.

ARG-72

First of all, the author suggests that the editor’s claim is undermined since a recent report on the Elthyrian economy shows that since 1999 far more jobs have been created than have been eliminated and, thus, it might not be true that it is hard for the majority of competent workers to find suitable jobs. In order to evaluate this claim, we need evidence to show whether jobs that

Page 135: ARG-FINAL

have been created are similar kinds of jobs compared to those that have been cut. For instance, if a evidence shows that most newly created jobs are the same kind of jobs compared to those that have been cut, then it is conceivable that these competent workers should have many choices; thus, the author’s argument would be strengthened. However, if there is reliable study that shows that the jobs that have been created are different kinds of jobs compared to those that have been cut and, say, corporate downsizing results in cutting many positions such as secretaries, while new jobs that have been created are positions like technicians , or skillful workers, then, these competent secretaries would still have a hard time to find suitable jobs, because companies offer different positions. For this reason, the author’s claim would be weakened.Moreover, the author implies that the majority of competent workers should not face economic problems since two-third of these newly created jobs tend to pay above-average wages and are full-time. In order to test the veracity, we need evidence about if the wages for these new jobs are comparable to these competent workers’ previous salaries. For instance, if evidence shows that a competent worker used to make 5000 dollars a month on average, and most new positions only offer 3000 dollars on average, then even if 3000 dollars is an above-average salary, it would still be considerably less than what this worker used to be paid; this competent worker might not consider this job is suitable, and he still would keep searching more jobs that offer a better salary. In this case, the author’s argument is weakened. To sum up, in order to evaluate the author’s argument, we need evidence about if these newly created jobs are comparable to the ones that have been cut, how salaries for new jobs compared to salaries these competent workers used to be paid.

In the first place, the author provides noevidence to show a clear employment situation present, since he just point outthe lowest level in decades with a vague time point-since 1999. It is possiblethat the average unemployment rate from 1999 to present reach the lowest level,however the unemployment rate for recent two or three years keeps high. Even ifthe unemployment rate since 1999 keeps low, if the unemployment rate inElthyrian still much higher than the average level of nation, the employmentsituation is still tough.

ARG-73

The writer contends that the Mozart School of Music should obviously be the first choice of any

music student aware of its reputation. The writer then goes on to describe the attractive features

of the School which include the best professional equipment, facilities and faculty. Also

according to writer the School stress on intensive training and practice so that the students begin

their training at a very young age. However the writer is very hasty in concluding that it should

be the first choice of music students, because the argument put forth suffers from numerous

flaws as is analyzed as follows.

Page 136: ARG-FINAL

The writer harps upon the excellent facilities and equipment’s with the best music teachers all

over the world. However, given so many facilities, it is very likely that the tuition fees in this

school are much higher than in other music school. Therefore, students might in fact avoid going

to Mozart School of Music due to the exorbitant fees, unless the student comes from a very good

financial background.

Another factor that generally students consider while enrolling in a music school is its distance

from their house and the housing facilities provided to the students. Many students would prefer

an average quality school near their house than a top rated school which is very far away. Also

the writer does not mention anything about the housing facilities for the students coming from

far off places. Large amount of time wasted in traveling to and from the school can keep the

students from joining the Mozart school.

While mentioning the salient features of the School, the writer says that it stresses intensive

practice and training so that the students begin training typically at a very young age. But it is not

desirable that children be exposed to rigorous training schedules at a very young age. The

intensive training is likely to wear out the young students too soon, and so much that they will

not be able to concentrate on their general school studies properly. If the training is too intensive,

then in extreme cases, the tight and exhausting training schedules might interfere in the normal

growth of the students since they will be left with little time for other essential extra-curricular

activities like playing etc. Moreover when student enroll in music schools at a very young age, it

generally is as a hobby, so such rigorous training might not be appealing to many students.

Generally, there are very few students who join the Music school for pursuing it as a career.

Since music is not a main stream branch of study, many people join music school only as a

hobby or to take it up as an avocation. So, many might not be too keen about the top rated

facilities and equipment provided by the school. Again fees and the intensive training might act

against attracting such students to the Mozart School.

The writer mentions that many Mozart graduates have gone on to be the best known and the

most highly paid musicians in the nation. But one cannot be too sure to attribute their success

only to the music school. Maybe these successful students are born with excellent music talent,

and it is well known that a person with a genuine talent can be successful anyhow, whether he

studies in a top rated school or in an average rated school. It is quite possible that their own hard

Page 137: ARG-FINAL

work and perseverance have contributed a more to their success than the facilities provided at the

school.

Thus, as is analyzed above, the argument put forth is highly unpersuasive. While making a hasty

conclusion, the writer forgets to take into account many other factors that affect a student’s

choice of a music school

This short article suggests that choose the Mozart Scholl of Music is a right thing to do for parents who want to enroll their children for music lessons. Well-presented though, it is not that convincible. Some fallacious assumptions it makes weaken the credulity of the argument. The presentation of these so-called advantages don’t indicate it a good choice for children taking music lessons.Firstly, that the music school welcomes youngsters at all ability and age levels seems not to be a good index for school-choosing. The author assumes that it is suitable for all youngsters, which is not valid. Some children may be too young to accept such intensive practice; at the first stage of their learning course, there should be some preschool instructs to the music lessons so that they could find where their interests really rely on instead of pack them to a school suits for all level students. It could only puzzle them rather than enlighten them. So, the school welcomes youngsters from all age and ability is not an element for a great music-learning place for young children.Secondly, cited in this argument is the wide range of styles and genres as well as the categories of instruments they teach. Another invalid presupposition has been made that the Mozart Music School is good at teaching every instruments they list. There might be alternative possibility that the Mozart Music School has such options for enrolling students, but what they have got is just superficial education of all those kinds, which is not that fitting to those who appeal for more insightful music learning. A more specialized music school appears to be a better choice for those people.Thirdly, even if the assumptions presented above turn out to be tenable, another flaw remains, inconvincibly, that the author implies their faculty who are distinguished musician is a good instructor too. As we all know that some great instructors of music are not really a pronounced player, while someone may be a magnet in the music profession is not a proper one for education. Without clarifying the capability of tutoring-not playing skills-of those faculties, under no circumstances could the author evince the Mozart Music School is the first choice for children who want to learn music. Additionally, granted that all doubts cast above is solved by some ensuing evidence, this argument still remains inconvincible. One flaw leads to this invalidity is the author’s fallacious presupposition that the attainment of graduates and their tutoring accepted here are associated. While other possibility could supplant this assertion that may be it is the gifted ability of them that facilitates their success. No evidence implies the relationship between the education they offer and the achievement they’ve got. It cannot be a persuasive recommendation; they should

Page 138: ARG-FINAL

put up some specific evidence to illustrate their crucial influence on these successful graduates, foreshadow the important role they play in this course rather than make a false impression on the parents that why they are success is due to their gifts, in this case, Mozart Music Scholl, to some degree, cannot be a good choice for public.

First of all, the author mentions that the Mozart school of music should be the first choice for parents considering enrolling their children since this school does not require an audition to attend. Thus, the author necessarily assumes letting students to attend schools without any selections will encourage parents to consider enrolling their children in music lessons. However, if this assumption is proved unwarranted, parents may think since students can attend the schools without audition, then it would be possible that the abilities of students in this school are highly uneven, and students who enrolled in this school may not be very competitive, and it might not be chanllenging enough for their children, and the author’s argument would be undermined. 

Moreover, the author suggests that the Mozart is the first choice because the school offers instruction in nearly all musical instruments as well a wide range of styles and genres. Thus, the author necessarily assumes that parents may value the variety of courses offered by the school heavily. It is possible that this can be one of the most important consideration when parents choose schools for their children, and if this is the case, then the author’s argument would be strengthened. However, parents might not think the variety of the courses is important at all; they may value the quality of the courses their children are interested in over the variety of courses, since their children might only enroll in one course or two. Then, they might not care about other offers at all, and the author’s argument would be weakened.In addition, the author mentions that the faculty of Mozart includes some of the most distinguished musicians in the area, and thus that the Mozart is the first choice. However, this claim only makes sense based on the assumption that these good musicians are indeed good teachers. If the assumption proves unwarranted, these musician may be good performers, but they did not receive any training of how to teach students, they may not have any experience in teaching youngsters, and they may not be patient with children, then, these famous musician might not be good teachers to teach youngsters, and then, the author’s argument would be weakened.

To sum up, in order to evaluate the author’s argument, we need to consider the following assumptions: parents prefer to enroll their children in schools that accepts almost any students, parents values the variety of courses a lot, and these distinguished musicians are good at teaching youngsters.

ARG-74

In this argument, to the future development of the university, the president of the school and the

director of the alumnae association held opposite opinions. The president expects a revolution

Page 139: ARG-FINAL

due to the university is all-female school at present and an increasing in applications if male

students are allowed to apply too. However, the director assumes this action will stain the

school's old image and offend the majority members of alumnae association. Before hastily

following the ideas of either of the both, there are several questions needed to be answered.

By citing the examples of other all-female universities which implement the new policy of

including male students and stimulate the number of applications, the president deems that

enacting the same policy will benefit Grove College too. The question here is, however, whether

Grove College can be compared with those universities. Because the author does not provide any

information of those schools even Grove College's. Perhaps the same action should be ineffective

on Grove College since their disparate background details such as the historical rate of

applications and quality of education. For example, due to the high quality of education and

advanced facilities of Grove College, its historical rate of application is rather high and staying

stable, while other universities may confront financial problems because of their low applications

and then a reformation is quite essential to them. If true, there is no need for Grove College to

change its basic policy to follow others' trend which may does not fit itself. Therefore, before

offering more convincing information about the similarities of those schools, the president

cannot persuade his/her rival.

The president's rival-the director cites two surveys to undermine the president's ideas. According

to the first survey of incoming students who say the all-female status is the initial reason why

they choose GC, the director deems that the changing policy will impair the college's old

tradition and very identity. At first glance, the assumption seems quite reasonable, however, the

survey fails to provide the respondents' size whether they can be the representatives of whole

students. Besides, whether those respondents' reply can be trusted is also suspectable because

there are possibilities that the students did not express their very thoughts if they got their

parents' companies around at that time, or that the students tried to impress the professors of the

school to get accepted. Either scenario, if true, would serve to undermine the director's own

claim.

Furthermore, based on the latter survey of Grove alumnae who strongly support keeping the

school's old tradition. However, the survey's reliability is highly doubted due to the investigator

is the director who is the leader of the Grove alumnae association. Those respondents who are

afraid to offend their leader may not offer their truthful opinions. In addition, it is quite possible

Page 140: ARG-FINAL

that for supporting his/her own ideas, the director only select part of the results of the survey

which may favor his/her ideas. Unless, the director can provide other objective and sounded

survey, the opinion hold by him/her can not convince the president to drop the initial thoughts.

To sum up, there are questions for each of the person to answer due to neither of them provides

reliable and persuasive evidence which can thoroughly illustrate the present condition of the

college. Although their concerns about the school should be appreciated, other methods which

can enhance the applications of school should be considered such as hiring famous professors

and updating technological facilities. Only after well-rounded considering and unbias discussing,

the best policy will be appeared.

First of all,the author mentions the fact that the majority of students answering the survey wished the College not to change its academic traditions.Nevertheless,I can assume that some students might be afraid of male competition in some programs.It is a proven fact that while female prevail in some disciplines,men succeed in others.Consequently,Grove might have very strong academic programs in female disciplines,but weak courses in others.Students might not be eager to study some techical ot mathematic disciplines,where male usually are better,and decided to remain College education the same.

Secondly,the article fails to correlate the alumni who participated in the survey with the whole number of alumnies.It might be that only the minority of alumnies answered the questionnaire.Thus the author might make a big mistake thinking that most of alumnies are for remale education in Grove.The author could seriously strenghten this article by providing statistic data concerning alumni's opinion.

Thirdly,I can not agree with the fact that all-female education will improve student's morale.In my opinion,co-education only benefits students of both sexes.Student learn how to cooperate with each other,study the phsychological peculiarities of the opposite sex. Guys will be able to learn in practice, how to treat girls, and the former will have a great opportunity to respond adequately to men’s attention. These practice experience will help them not only in their academic life,but teach them how to handle relationships with people later.

To conclude,I assert that the mentioned above article has many weak points.I think these logical contradictions don't allow the college's governing committee to make an adequate conclusion concerning the future of Grove's education.

ARG-75

The author of this argument claims that the Batavia government should begin to regulate retail milk prices. To substantiate this claim the author cites the report of Batavia agriculture

Page 141: ARG-FINAL

department. The argument is questionable in several respects.

To begin with, the author assumes that 25 percent increase of the number of dairy farms contribute to the increased supply of milk. The author fails to provide the original number of the dairy farms. It is possible that the original number is very few in ten years ago, so the 25 percent increase is not enough for the increase populations' demand. 

Then, the author mentions that the price of milk at the local Excello Food Market has increased from $1.5 to over $3.0 per gallon. The author does not provide any evidence to prove that the $3.0 per gallon price is high. It is probably that the expense of running the dairy farms is increasing more than the price of milk. So, the farmers also cannot get the excessive profits. In addition, it is entirely possible that the price of the near market is much higher than $3.0 per gallon. It is unconvincing to say that the $3.0 per gallon is the high price to receive excessive profits. 

Finally, the author points out that the Batavia government should to regulate retail milk price. The author fails to consider that the price of milk is decided by the market. If the government regulates retail milk, the farmers get few profits even lose profits for the lower prices. If this is the case, the farmers probably do not sell milk at the local market, they may sell the milk to the near market to get the more profits. As the result, the supply of milk at Batavia is not adequate for consumers. The author should consider some other methods to serve for this argument.

In conclusion, the argument is unpersuasive. To strengthen this argument the author should provide more information about: the original number of dairy farms; the comparison of milk price to near market; whether the supply of milk is adequate for consumers.

ARG-76

The excerpt from the newspaper seems more like a vendetta against Old Dairy Industries, rather than being a sound advice to the investors. The argument does have a semblance of veracity at the outset, but critical evaluation of the author’s line of thinking leads us to believe, that indeed something is missing from the argument. 

The author has citied an example of a certain survey in which majority of the people preferred low fat foodstuffs. Even if whatever the author states is true, there has been no mention of the cross-section of the people at whom the survey was targeted. We may concede that all those people who are really on a low fat and low-cholesterol diet, might eschew from the products currently manufactured by Old Dairy Industries. But, without mentioning the percentage of total population who shun from these high fat products, it cannot be conclusively proved that the sales of products of Old Dairy Industries will be hit severely. 

Page 142: ARG-FINAL

An important point to note is that, in foodstuffs industry, a great deal of competition prevails. So, it can be safely assumed, that if Old Dairy Industries are hit by a reduction in their sales due to the type of products that they sell, they’d also move over to low fat products. After all, Old Dairy Industries doesn’t enjoy the luxuries of a monopolistic society. So, it might be a bit premature to say the the Dairy will suffer heavy losses, and that too, depending only upon the survey carried out. 

The author goes on to mention that most of the products available in the market are low in fat composition. While, it may be proper to think that these products would be much better and healthier than high cholesterol products, some deeper probing must be done at this juncture. Low fat milk is a commonly available product. In this case, a point to be noted is that fat can be separated from milk. But, if you take the case of ghee or oil, is it feasible to think of low-fat ghee? Is that possible? If fat is removed from ghee, what will remain? Hence, some products are such that they cannot be branded as low-fat products. The author should have presented what percent of the products manufactured by the particular dairy are products which cannot be sold devoid of any fatty ingredients. That would have given us a better idea about the whole issue.

The conclusion of the author lends us to believe that maybe the person has some vested interest in undermining the reputation Official Guide Old Dairy Industries. Overall, the argument is too tenuous to make an impact on the reader. The investors should have no reason to panic; at least not yet, till the entire picture becomes evident. The scene may not be as drab as the author makes out to be.

In the argument above, the author advises old Dairy stockholders to sell their shares and other investors not to purchase its stock. To support this conclusion, the author cites the evidence that low-fat products are more desired among public in a survey, and points out that Old Dairy's sales are likely to fall. Close inspection of the evidence and reasoning line reveals, however, this advice is poorly supported and ill-advised unless some critical questions answered.

First of all, whether the survey cited in this argument is reliable is open to doubt. Specifically, we are not confirmed whether the responses are accurate and whether the respondents in the survey are statistically significant in number and representative of the overall food consumers. Besides, information is not given about who conducted this survey and whether it has any interests related to this industry. Without specific answers to these questions, the survey cited by the author is groundless and problematic at best.

Secondly, even assuming this survey is reliable, the fact that low-fat products are more desired among public does not necessarily indicates that the Old Dairy's sales are likely to decline and hence the company profits. Firstly, granted that low-fat products are preferable in public, it does not follow that people will no longer need foods high in fat and cholesterol and will never spend one cent on it. Secondly, even if this decrease of high fat and cholesterol products are inevitable,

Page 143: ARG-FINAL

this company's sales do not necessarily to fall, for that Old dairy's food products might be composed of a wide range of styles and each style could meet the satisfaction for alternative choice. Besides, the author fails to inform us whether Old Dairy has food products low in fat or not. Without convincing evidences to rule out these possibilities, the arguer cannot justify the conclusion that the company profits will undoubtly decrease.

Finally, even assuming that Old Dairy's sales and profits of food high in fat and cholesterol will decrease, it does not necessarily indicate that this company is no longer profitable, and that investors should abandon it. First, the author fails to inform us the proportion of the high-fat food sales to its overall foods sales. If that proportion is relatively small, the company's profit might not be damaged even if the decline of high fat food. And, even the overall foods sale will undoubtly be ruined, the author overlooks the possibility that Old Dairy might involve in many other industries and maintain a lot of properties, say, real estates, which amounts to large percentage of its overall profits. In short, holding old dairy could still generate interests and investing Old Dairy could also be profitable.

In sum, this advice is not well supported and ill-advised. To better evaluate this argument the author must provide clear evidence to justify the survey and specific answers to whether Old dairy's overall profits will undoubtly decrease. Otherwise, this advice could be ill-conceived at best and mislead the investors who hold old dairy's stock or are ready to purchase its stock.

ARG-77

In this memo, the mayor of the town of Hopewell recommends that Hopewell build a golf course and resort hotel similar to those in Ocean View. To bolster the recommendation, the mayor cites the facts that tourism in Ocean View has increased, new businesses have opened there, and Ocean View's tax revenues have risen since it built a new municipal golf course and resort hotel. However, the argument is rife with holes and unreasonable assumptions.

Firstly, the mayor assumes that it is the new golf and resort hotel which are built by Ocean resulted in the increased number of tourism. However, it is unwarranted. It is entirely possible that the tourist cost such as food price and rental price in Ocean View is low, which attracted so many tourism, rather than the new golf course and resort hotel. Moreover, we do not know the actual number of  Ocean View's tax revenues before building the course. Maybe it is almost zero, thus the 30 percent increase may be insignificant. For this matter, the mayor's recommendation is ill-conceived.

Even if the new construction in Ocean View was successful two years ago, yet the underlying assumption that Ocean View and Hopewell are comparable is open to doubt. Firstly, Hopewell may have no more fund than Ocean View has, and the cost of building golf course and hotel is

Page 144: ARG-FINAL

too high to afford for Hopewell. Secondly, it is possible that the popularity of Hopewell is lower than that of Ocean View, thus it cannot attract many customers and business to here. Thus, without considering and ruling out these possibilities, the mayor cannot convince me that it is a good choice to follow with Ocean View's actions.

Even if these two towns are comparable, the author unfairly assumes that the circumstances relevant are essential same. Maybe during the two years, the golf course and resort hotel are commonplace, which thus cannot attract any tourism. And is it the only way to improve Hopewell's economy? I am afraid not. Some other ways may have more positive effects, such as pay more attention on industrial development which provides more jobs. Thus, taking the same actions as Ocean View, Hopewell's economy may unnecessarily be risen.

In summary, without providing more detailed information about Ocean View, the mayor cannot conclude that the new golf course and resort hotel caused the increased number of tourism. And if these two towns are incomparable,  the mayor's recommendation that Hopewell build a golf course and resort hotel similar to those in Ocean View is ill-conceived.

ARG-78

In this argument the author recommends to return to contract with Buzzoff Company for the pest-control service for the food-storage warehouses in Palm City in order to save money from lost caused by pest damages to foods. To substantiate his conclusion, the author provides as evidence the last month lost of over $20,000 worth of food destroyed by pest damage, while in Wintervale the lost was only $10,000 because it was served by Buzzoff Pest-Control. However, a careful examination of this argument would reveal how groundless the conclusion is.The crucial point I cast great discretion on is whether the cause of the large lost of last month in Palm City was the failure of pest-control service provided by Fly-Away. The author falsely assumes that the pest-control company did not do all necessary to properly prevent the pest damage. Perhaps, last month there was an extraordinary pest in Palm City, which was not taken into account by any pest-control company before, thus that had to be the true responsible of the damage. In this case, it is totally unfair to blame Fly-Away by this fortuitous event. Even if it was controlled by Buzzoff Company, the author cannot assure that the result would be different.  Moreover it is also important to reconsider the reasons by which the company changed its pest-control services for their fast-food warehouse in Palm City. If Buzzoff was offering a good service already, why would company replace it by Fly-Away? Was the price they charged the only reason? In case that there were other causes that led to the decision, the author should confirm that those problems had already solved before making such recommendation. Furthermore, if these problems are still present, re-contracting with Buzzoff would not only not save money to the company, if not it is probable that more food would be destroyed by pest damage.Another flow of the analysis is that the author presumes that the last month lost of $10,000 in

Page 145: ARG-FINAL

Wintervale was a good indicator of performance of Buzzoff Company. It is plausible that actually Buzzoff was not operating in the most efficient way to avoid pest damage to food. Perhaps $10,000 is already relatively a severe lost according to its conditions, perhaps the environmental condition of Wintervale is better than the average, thus pest damage is not frequent hence it is easily preventable. In this case, $10,000 can not given as proof of success of Buzzoff,on the contrary, it is certainly an evidence of its poor performance. In case that the conjecture is correct, the company might have to consider the possibility of replace Buzzoff not only in Palm City, if not also in Wintervale.In sum, the recommendation relies on certain doubtful assumptions that render it unconvincing as it stands. To bolster the conclusion, the author must show dear evidence that the pest damage to foods in Palm City was exclusively due to the bad performance of Fly-Way. He also should reconsider the reasons by which the company replaced Buzzoff company before taking the decision of re-contract it. At meantime, it is necessary to evaluate the Buzzoff company’s role in Wintervale according to local environmental conditions.

In this argument, the president concludes that the best means of saving money is to return to Buzzoff for all our pest control services. To support this assertion the president points that last month over $20,000 worth of food there had been destroyed by pest damage in Palm City with the pest control of the Fly-Away Pest Control Company (F), while only $10,000 worth of the food stored there had been destroyed by pest damage in Wintervale with the pest control of the Buzzoff Pest Control Company(B), which we have used for many years. However, I find this argument suspicious on several grounds.One, the president lacks sufficient information about the situations about the warehouses in Palm City and Wintervale to determine what the assertion, if any, can be drawn from it. It is very likely that the level of pest plague in Palm City was much more severe than that in Wintervile. If B serviced the warehouse in Palm City, it is entirely possible that the food would have larger losses. In addition, perhaps the quantity of food losses in Palm City was less but the food price was higher. It is also possible that the quantity of storage food in Palm City was much more than that in Wintervale, and both these possibilities could cause the greater loss in Palm City. Therefore, without detailed evidence about the food price and quantity and the pest plague situations, the president cannot justifiably depend on the loss of food to draw any conclusions whatsoever.Two, even if the situations between two warehouses are similar or the same, the president provides no clear evidence about the concrete charging price about the two pest control companies. Though the food loss was lower if B service the warehouse, the price charged by B might be more than $10,000 than the charging price of F, thus, obviously B is not a good choice. However, the president overlooks this possibility, the conclusion is still ill-conceived.

And finally, even in pest control aspect the gross expense of food company in B was lesser than that in F, the assertion that the best means of saving money is to return to Buzzoff  for all our pest control services is also unpersuasive. Perhaps other pest control companies would be less money-consuming and more efficient. What's more, different companies have distinct

Page 146: ARG-FINAL

advantages in the control of different kinds of pests, thus, we could choose different companies based on the pest kinds in different cities. Without addressing the evidence of data of other pest control companies, the president cannot convince me that B is the best choice in pest control.

Summed up, the argument is not grounded on sound reasoning and therefore unconvincing as it stands. To bolster it the president must provide clear and persuasive evidence that the situations about the storage warehouses were similar. That the expense in B was indeed lesser than that in F, in addition must prove that other pest control companies are worse than B.

First, we have to consider about the location of the warehouse. The weather difference and geographic difference might affect food preservation. For example, the humidity in Palm City may be higher than in Wintervale and thus the pests may more easily grow in the warehouse causing more severe pest damage. So it is important to check whether the environment of the two warehouses is similar or not; if the environment is different (as in a temperature or humidity difference) we cannot say that Fly-Away Pest Control Company is worse than Buzzoff.

Second, we need to examine the total amount of food storage in the warehouse. The absolute value cannot show the real performance of the pest control, because maybe the warehouse in Palm City originally stores $100,000 worth of food there, but the warehouse in Wintervale stores only $20,000 worth of food at first. The percentage of the pest damage for Fly-Away Pest control Company is lower than Buzzoff in that case, and thus the result is totally different from the argument expected.Besides that, the food itself is also a factor that would affect the pest control efficiency. If the food stored in the warehouse is different in Palm City and Wintervale, it is not fair to compare the pest control efficiency between them, because maybe the food stored in Palm city is more susceptible to pest damage than the food stored in Wintervale.Due to the above discussion, the argument still needs more evidence to ensure that its conclusion is correct.

ARG-79

Grounding on the supposition that the issues of Newsbeat magazine featured by political news on

their front cover were the poorest-selling issues in three years, and based on a recent survey of

readers that indicates a generally greater interest in economic issues than in political ones, the

author recommends that the magazine curtail its emphasis on politics to focus more on

economics and personal finance. This recommendation seems at first sight convincing. After all,

it is the intent to bring the magazine more profit. However, further reflection tells me that it is

Page 147: ARG-FINAL

based on several flawed evidences, and does not constitute a logical argument in favor of its

conclusion.

To begin with, the arguer assumes that by turning the emphasis on politics to economics, the

magazine will gain more profit. This assumption is questionable for several reasons. First, we

don't know if it's necessary for the magazine to take this action. It might be possible that the

magazine is profitable enough with its current policy so there's no need to change the editorial

policy. Second, the author also ignores the expense the magazine has to spend on the change.

The cost might be too high to make the gain adequate. Besides, perhaps readers are already

familiar with the style of Newsbeat and a sudden change will cause untoward effect among them.

If so, the author's assumption would be seriously weakened that changing policy will bring the

magazine more profit.

Another problem of this commendation is that the author grounds the claim on the fact that those

issues with politics covering front were the poorest-selling ones. This is unjustifiable because

there is no detail information about whether those issues were poorest among all the magazines,

or just comparing to other issues of the magazine itself? If the answer is the latter one, the author

overlooks the possibility that those issues might not sold as good as before, but they were still

best-sellers in the whole market. If so, the author cannot conclude that this magazine desperately

need a new policy to change the "poorest-selling" condition.

Moreover, even if I concede there was problem with the sales of those issues, the author unfairly

assumes that the poorest-selling issues must attribute to the front cover featured with political

news rather than other possible factors. For instance, there might be an economic crisis or

depression that caused decreased sales in every business. Or perhaps some other contents in

those issues were disliked by most readers such as useless star scandals. After all, a limited

number of issues can't tell the real reason of poor selling. For that matter, a deeper research on

this question is needed.

The last thing I want to take a glance at is the survey on which the recommendation is based. It

fails to provide sufficient evidence to justify the authority of the researchers conducted the

survey. And the number of respondents of the survey is not shown. If only a few readers take the

survey, the conclusion can't be made. Even if the survey is reliable, the conclusion that readers

prefer economic issues generally is still questionable. An economic crisis, a depression, or

federal budget deficit may intrigue reader's interest in economics in a short period of time as

Page 148: ARG-FINAL

well. Considering this, they might not be interested in the topic soon after. This makes the

recommendation of changing politics to economics untenable.

In sum, the argument doesn't provide enough information. Before the conclusion of change of

editorial policy is reached, the author should give a more complete understanding of the

magazine's sales and the readers' real interests to make this a commendation more convincing.

In the argument above, the publisher of Newsbeat recommends that the magazine should take more account of economics and personal finance and less account of politics in order to increase sale. It seems to be an obvious conclusion based on the poorest-selling issues over the past three years and the recent survey of readers. However, close scrutiny of the argument, rife with holes and questions, reveals how groundless the recommendation is .

First and foremost, the questions—how the conclusion is obtained that magazine featured political news on their front cover leads to the bad sale, and is there a casual relationship between them—should be answered. Whether the conclusion is based on the author’s observance or a statistics of one authoritative research institute, the foundation of the author’s judgment, should be substantiated reliably. Even though the judgment is verified to be right, the author also neglects other reasons which influences the sale of the magazine. If the quality of cover story report and the level of the journalist’s writing cannot satisfy the demand of the readers or if the political news on the front cover is similar to the report on the television, then the readers are less likely to buy the magazine which seems dispensable. Unless the author gives satisfactory answers to these questions, the recommendation is unconvincing.

In addition, the author relies on a recent survey of general interest among readers ,which indicates readers are prefer to economic issues . However, that how the proposal of the survey is designed, including the questionnaire , the process of the data, the selection of the respondents and so on, is unknown. For instance, the survey could have asked readers who major in portfolio investment if they prefer to read political stories or would like to find some information about economics and personal finance, which may have swayed readers toward economics. Even if the author can guarantee the validity of the survey , readers having interest in the economics does not indicate they would buy these magazines. Perhaps they would rather read the economic edition in the library than printed one.

Finally, even though the questions above can be answered, it does not follow that the recommendation of focusing more exclusively on the economics and personal finance takes effect. Newsbeat’s editor implicate their magazine does not offer extensive political coverage, so the recommendation to modulate the stress could not solve the problem radically without ruling out other factors which also have a great influence on sales, such as coverage of information, price, issuance strategy.

Page 149: ARG-FINAL

As it stands, though the recommendation to transfer the emphasis from politics and personal finance is plausible to function, it has to resolve several questions. To corroborate the argument , the author ought to justify the casual relationship between the political story on front cover and the poorest-selling issues and the validity of the survey. Moreover, without deliberating the factors with influence on sales, the credibility of the recommendation is still open to doubt.

ARG-80

To increase profit is the most important issue among many companies around the world. In the

preceding statement, the author claims that public is lack of awareness that movies of good

quality are available and Super Screen should therefore allocate more money next year to

reaching the public through advertisement. Though his claim may well have merit, the author

presents a poorly reasoned argument, based on several questionable premises and assumptions,

and based solely on the evidence the author offers, we cannot accept his argument as valid.

Primary issue with the author`s reasoning lies in his unsubstantiated premises. The arguer

provides no assurances that the information he gives about the attendance is statistically reliable.

He does not mention about any surveys or statistics. Lacking information about the number of

people on which this data depends , it is impossible to assess the validity of the results. Also he

mentions of the percentage but he doesn`t say anything about previous percentage. Maybe it is a

tiny increase or there is not any increase at all. People of negative reviews might have not given

their thoughts openly. The author`s premises, the basis for his argument, lack any legitimate any

evidentiary support and render his conclusion unacceptable.

In addition, the author makes several assumptions that remain unproven. He assumes that there is

no problem with quality but with not reaching enough of prospective viewers. There is no valid

data showing this. It may be the quality why the attendance declined. He also assumes that public

is lack of awareness but it may not be like this. People may be aware of the quality of movies at

Super Screen but find them low-quality and prefer not to go. The author weakens his argument

by making assumptions and failing to provide explication of the links between the problem and

the solution.

While the author does have several key issues in his argument`s premises and assumptions, that

is not to say that the entire argument is without base. With search and clarification, he could

improve his argument significantly.

Page 150: ARG-FINAL

In sum, the author`s illogical argument is based on unsupported premises and assumptions that

render his conclusion invalid. If the author hopes to change his reader`s minds on the issue, he

would have to largely restructure his argument, fix the flaws in his logic, clearly explicate his

assumptions, and provide evidentiary support. Without these things, his poorly reasoned

argument will likely convince few people.

ARG-81

In this editorial, the author argues that the Promfoods which result in part of consumers of tuna

feel dizziness and nausea did not pose a health risk after testing. To support the conclusion the

writer points out that the conclusion is based on tests performed on samples of the recalled cans

by chemists from Promofoods and only find small amounts of the three remaining suspected

chemicals which could cause symptoms of dizziness and nausea and they occur naturally in all

canned foods. However the argument suffers from several flaws and is therefore unconvincing as

it stands, albeit it may appear plausible at a cursory glance.

First, there are problems about the representativeness and the randomness of the survey’s

sample. The author should provide the information about the sample, we don’t know where are

the cans from and when do they product, perhaps the sample comes from nationwide or just only

a local, the two statistics does not necessarily apply to each other, so the survey results are not

reliable. Hence, without accounting for as well as ruling out other likely factors, the author could

not assumes that the canned tuna did not pose a health risk.

Secondly, even if five of the eight common food chemicals which cause symptoms of dizziness

and nausea were not found in any of the tested cans but it does not means there are only eight

food chemicals can result in the symptoms of dizziness and nausea, perhaps the device of testing

is not advanced enough to find out other potential food chemicals which may cause the same

symptoms. So the company should pay attention as well as deal with testing the canned tuna if

they want to substantiate that there is no health risk with the canned tuna. With the evidence of

ruling out other likely factors could writer bolster his assumption.

Thirdly, the author cites that the chemists did find small amounts of the three remaining

suspected chemicals but pointed out that these occur naturally in all canned foods. Small

amounts does not necessarily indicate zero and the author did not say what is the degree about

small amounts, we can regard 0.1 percent as small amounts and we also regard 0.01 percent as

Page 151: ARG-FINAL

small amounts, so the point is how much can cause people sick, so is it open to a number of

interpretations, without compelling evidence to support, the arguer’s conclusion is not worthy of

consideration. Providing more information about the results of testing the writer could

substantiate his assumption.

In retrospect, further investigation and analysis are needed, if the argument had included the

given factors discussed above, it would have more thorough and logically acceptable

The conclusion made in the business magazine that the canned tuna, which were produced by company Promofoods(PF), did not pose a health risk may seem plausible at first glance. To strengthen the conclusion, the test result of those returned canned tuna was added to the statement. However, closer scrutiny of the evidence given in the statement reveals nothing about the validity of the conclusion but its inaccurate and untenable assertion.

First, the chemists checked eight chemicals that were commonly blamed. However, no more information in the statement about the components of the canned tuna is provided. Perhaps it was other substances other than those eight chemicals that caused the dizziness and nausea. If chemists only checked those 8 chemicals, it was possible that they ignored other potential noxious substances. Therefore, to strengthen the statement, the speaker must provide detailed information about the components of the food. Otherwise, it might not be convincing enough to make a conclusion that hastily.

Second, the statement mentions that no tested cans found those 5 chemicals, therefore all canned tuna made by PF contained no such substances. Nevertheless, how many cans were tested in the experiment is unknown. There is no information about the number of tested cans. The statement mentions that eight million cans were return. Yet perhaps only several hundred cans or even less were tested. If it was the case, the conclusion is untenable because only a small portion of cans were tested. Therefore, the speaker must provide information about the ratio of tested cans to all. If there is no information in the statement, it would be confusing and misleading for many consumers.

Finally, even assuming above two problems can be mitigated, the speaker is still too brusque to assert his or her conclusion. Although those three chemicals are common in all canned food, the quantity, or the concentration is not mentioned in the statement. It is obvious that overdosed substances may be poisonous. Thus, the statement should have information about the concentration of those 3 chemicals found in PF’s canned tuna as well as the safe concentration data about them. However, nothing referring to this issue appears in the statement. Therefore, the conclusion made by the speaker is not that unimpeachable.To sum up, the speaker is too abrupt and hasty to make such a unconvincing and misleading

Page 152: ARG-FINAL

conclusion. To strengthen his or her conclusion, the speaker must provide more information such as the components of the food, the ratio of tested cans to all, and the result of three substances. Equipped with these detailed information, the speaker can make the conclusion accurate, otherwise the statement is untenable and wrong

ARG-83

In this letter, the editor concludes that the decline of the amphibians was caused by the

introduction of trout into the park's waters. To support his conclusion, the editor cites the number

of the amphibians species from 1975 to the year 2001.At first glance, the argument appears to be

somewhat reasonable; actually, further reflection tells me that the argument has a myriad of

assumptions and hole.

First and foremost, the author presumptuously assumes that the trout is the main factor which

leads to the decrease of the amphibian’s species and the numbers, but the editor fails to inform us

some other information to support his conclusion. It is highly possible that there are more kinds

of amphibians in this park, just the study group did not see them, the editor unfairly assume that

not observing equals to not exist, the editor also fails to take into the possibility that the study

only observed for few days which cannot represent the real condition, it perhaps that some other

kinds of amphibians indeed live in this park.

In addition, even the species decreased from the year 1974 to the year 2001, we cannot conclude

that it is the trout, not others, that contributing to the decline of the amphibians' number. It is

perhaps the case that in this period of time, the amphibians committed a deadly disease that gives

rise to the shrink of the numbers of amphibians, there is also likely that the climate of this area

have changed a lot during this period of time, such as the huge increase of the temperature, the

plants in this area have decreased for the modernization utilities, and there is also the case that

the water of this area has been polluted. If there are a lot of mining factories on the bank of this

river which across the park, it is perhaps that when the animal drank the polluted the water, they

died.

Moreover, the author commits a fallacy called "hastily generalization", even if the species

decreased in number, the author cannot assume that the trout is the main reason for the decease

of the amphibians of this park. It is highly possible the case that most of the amphibians have

been transmitted to other areas or other parks, but the editor does not provide any information

about this, if the amphibians has been transmitted to other places and have lived for a good life.

Page 153: ARG-FINAL

The author cannot claim that it is the truth that makes brings about the, something goes wrong of

the species.

In conclusion, this argument actually has several flaws as discussed above, to make this

argument more convincing, the author would have to take into every possible consideration

account and provide more information to evaluate this claim.

This argument is well presented yet far-fetched. It lays a claim that the decline of amphibians

was caused by the introduction of trout into the park's waters. Nevertheless, this explanation is in

effect hardly convincing due to several flaws after a close scrutiny, albeit it may appear plausible

at a cursory glance.

First off, a threshold problem arises in this argument that the author assumes that the decline in

the number of species is caused by the trout other than other factors. In this situation, the author

certainly thinks that only trout eat amphibian eggs. However, this contention is open to a number

of interpretations. For example, the trout might find it hard to locate the amphibian eggs because

the Xanadu National Park was so large. We would never know if there exists other animals that

could feed on amphibian eggs. What if amphibians were just not compatible with the

environment there since there was an arid duration? Hence, without accounting for as well as

ruling out other likely scenarios, by no means could the author claim that the decline of the

number of the amphibians were caused by the introduction of trout.

Moreover, even though the author might be able to provide evidence for us to deduce a solution

to the problem presented above afterwards, the argument still maintains ill-conceived. Another

problem could be located that the editor contends that the results of the two studies are reliable.

They may be obtained by a group of experts in a meticulous way to conduct these studies but it's

totally possible that the park is too large for men to do researches thoroughly. Perhaps there were

more than four species of amphibians in the park in 2002 but it would take a painstaking work to

conduct such a research to confirm it. As is universally known, a regular way to conduct research

in a relative large area is to select some representative places to do researches. But this method

might not work when it comes to animals, for there might be ones with anomalous characteristics

such as staying under shades. To corroborate his point, the author should pay a close heed to as

well as cope with the representative possibilities mentioned above. Only the evidence of proving

the ability of conducting studies in this park is the key to bolstering his conclusion.

Ultimately, even if the foregoing assumptions might turn out to be supported by ensuing

evidence, a crucial problem remains that the author apparently assumes that the decline was

Page 154: ARG-FINAL

gradual from 1975 to 2002.  But this doesn't make sense since there is no evidence extracted

from this argument. In this light, it's reasonable to cast doubts upon the author's presumption

which I reject as inadequate, in that the decline might be abrupt. For instance, the author omits to

inform us about the situation of 2001. Perhaps the number of the amphibians was as many as that

of 1975 but a sudden calamity occurred in 2001. This could explain the decline as well without

counting the introduction of trout. Pursuing this line of reasoning, it proves to be the author's

responsibility to mull over his assumption and then furnish it with cogent evidence so as to pave

the way for a more tenable argument.

In retrospect, it seems precipitous for the author to jump to the conclusion based on a series of

problematic premises. To dismiss the specter of implausibility of this argument, the author ought

to come to grips with all existing flaws, such as the environmental factors, the reliability of both

studies, and the possible explanation of the decline. After all, feckless attempts with a fallible

method could be nothing but a fool's errand. Thus, only by grasping the gist of this argument

could the author draw a convincing conclusion.

ARG-85

The author concludes that the first study of the reading habits of Waymarsh citizens had

represented citizens' reading preferences erroneously, as evidenced by the result of the second

study that the type of book checked out most frequently in libraries was the mystery novel. The

reasoning in the argument seems plausible at first glance; however, after scrutinizing, several

logical fallacies undermining its validity could be found.

The first flaw in this argument is that the author mistakenly relates the type of book checked out

most frequently in public libraries to the reading propensities of the citizens. The former is likely

to imply another type of book that citizens in Waymarsh prefer, however it does not necessarily

follow that citizens surveyed in the first study are not in favor of literary classics, even other

types ofbook. It is possible that these citizens deem literary classics as worthy of garnering,

therefore instead purchase this type of book and ruse them at home. It is also possible that the

traditional reading method has given way to the electronic books and people could read them on

their computers. Without ruling out these possibilities, the author is not supposed to make any

conclusion about whether the second study represents more accurately thecitizens' reading

preferences.

Page 155: ARG-FINAL

Even though the type of book preferred is related to the one checked out in libraries, proof is

lacking that the two studies the author mentioned are controlled in terms of the citizens

surveyed. Perhaps the citizens in the first study are mostly adults, who are inclined to reading the

classics, while the majority of citizens surveyed in the second study are teenagers, who are more

likely to read mystery novels. If the data is not statistically explanatory, any conclusion made on

basis of it is not convincing in essential. The author should provide detailed information about

how these two studies were conducted to vindicate the argument.

Granted that the author could present evidence to rule out the factors mentioned above, the

conclusion is also untenable that the first study misrepresented the reading preferences of

thecitizens. It might be true that both of the two studies are not valid in explaining the real tastes

of the citizens, nevertheless it is somehow extreme to veto the first study as a whole. It is also

possible that in fact the second study is wrongly representing the result. The author ought to

keep his/her conclusion unbiased by presenting evidence to demonstrate that the second study is

more accurate, otherwise it will be unjustifiable to make such a conclusion.

In sum, the argument lacks validity in terms of its insufficient information and biased conclusion.

Unless detailed information and convictive evidence, such as the methodology of the twostudies,

and the relationship between the preferences of citizens and the booksc hecked out, are

provided, the conclusion could not be valid as it stands.

ARG-86

In the memo ,the author argues that the Delany Personnel Firm is effective to offer those

emolyees assistance in creating resumes and developing interviewing skills and the firm is better

than Walsh. Besides, he cites various evidences to support it, however, none of them are

convincing because it relies on a serious of unsubstantiated assumptions which render them

unconvincing on their stand.

First, the argument unfairly exaggerate the abilities of Delany Personnel, thus, the author

contribute all the success of employees finding on it. However, there is no evidences to

substantiate the power of the firm, thus, it is entire possible that nearly all the laid- off employees

find on the primarily benefited on their owe internal qualities but rather Delany Personnel's help.

Just because they have lots of experience and a good attitude to their work, they eventually find

the job, the help of the Delany Personnel only play a auxiliary roles. And for those people who

Page 156: ARG-FINAL

did not find the job, maybe their own abilities is poor, thus, they do not want to spend more

money on extra help from Delany Personnel Firm, regarding of the practice's low effective for

them.

Second, the author unfairly assumes that the situation of the work market is never change over

last eight years. Thus , the author claim that Delany is clearly superior because the firm spend

less time helping employees find the job last year than Walsh in eight years ago. However, there

is absent evidences to show the situation of the work market between the eights years ago and

the last years, thus, we can say that in eight years, there are enough workers working in every

realm, thus, it is difficult for Walsh to help people find the job, while in the last year, there is a

big lack of workers and nearly every firms needs new employees, especially those experienced

people. thus, for Delany, it is much easier for them to help people find the jobs. So, it is unfair to

compare the firms ' business in different period of time.

Third, the author also unfairly assumes that bigger staff and larger number of branch offices are

equal to the better business and better abilities of the staff. It may be possible, even the staff in

the Walsh do the part time job in the firm, their abilities and effectiveness are much better than

staff in Delany, because for the same amount of work, staff in Walsh only need three persons to

deal with while in Delany, as much as ten persons can address it. Even though the staff in Walsh

have other work, it is benefit for Walsh's business, which to some extent appear that staff in there

contain more people- circle that is benefit for them to help people find the job.

In sum, the argument is unpersuasive and author needs to offer more evidences to substantiate

the assumptions mentioned

In this argument, the speaker considers that Delany Personnel Firm (DP) can provide best service

on dealing with the issue about job finding of the laid-off employees from the XYZ company.

Also, he believes another firm, Walsh Personnel Firm (WP), cannot rival DP. This argument

seems plausible at the first glance, but after a careful scrutiny on it, we may find the whole

argument is based on some assumptions without well substantiated, therefore leading this

argument untenable somehow. 

First off, the writer simply owes all contributions to DP service that makes those laid-off

employees find jobs quickly without considering other possible reasons. It is likely that those

who take the service of DP are fairly eager to get another job, while others do not share this

Page 157: ARG-FINAL

strong desire. Therefore, the real reason why those taking DP service get job faster than others

could be due to their activation for job hunting instead of the service itself. Also, what DP firm

provides is only the resume creating service and instruction of interviewing skills. There is no

evidence showing whether the service provided by DP is good enough for an obvious promotion

which will boost the trainees’ abilities dramatically, since we do not know the original personal

abilities of the learners. Maybe they are already in good ability of resume and interviews. In a

word, it requires evidence to further prove whether DP can provide a good service to promote

job finding for the lay-off employees.Secondly, when comparing with WP, the writer assumes

the more staff, the more offices, the better quality of service on job hunting. Therefore he

concludes that DP is better than WP. How to prove it? In fact,

the number of staff and offices simply provides nothing direct about the real quality service of a

firm. It is likely that WP is well managed and able to provide a service with a lower cost, which

is more comprehensive and of better quality than DP. Without information in detail to show the

real strength of each firm in all different angles, it is difficult to tell which one is better regarding

of the number of staff and offices only.

Finally, the writer considers the average time taking a trainee to find job can portray the quality

of the service of a firm at best. But the average value in fact cannot tell the real situation, or even

blind us. After all, the sample sizes of these two firms are unknown. Maybe DP only provided

service to 10 clients, and MP did to 100. Based on simple statistic knowledge, the result of DP

could be inaccurate due to the small sampling and large uncertainty. In addition, we do not know

what kind of jobs the clients of these two firms got. Some jobs may take longer time to match

than others. Therefore, this comparison on average time could be senseless due to its poor

comparability. 

In sum, without careful substantiation, the writer hastily concludes the service of DP benefits the

laid-off employees for sure. Also, he concludes that the DP service is better than WP when

lacking necessary evidence to prove the validity of his assumptions. Further study is required to

clarify the uncertainty generated from the assumptions discussed above. Otherwise, no

conclusion can be obtained.

ARG-91

In the argument, the author made a conclusion about the reason of the disappearance of western gazelle is the larger number of predators in the eastern preserve since 100 western gazelles were moved to the East Palean Preserve. However, several flaws has appeared in the argument which

Page 158: ARG-FINAL

weaken the convincing of it.

The first possibility of the disappearance of western gazelles could cause by the shortage of food. Since the author didn’t mention the main source of food in Western Palean Wildlife Preserve and talked about the rainfall is less, though slightly, in the East Palean Preserve, it was likely that East Palean Preserve could not produce this specific kind of food for western gazelle and the declination of this animal is due to starvation. 

Besides, because western gazelle is close related to astern gazelle, they may share same kind of food. But if the East Palean Preserve couldn’t provide sufficient food for both species, than the number of both of them will decrease. Since astern gazelle process a larger population than western gazelle, the percentage of declination would not be so obvious but it would show a great declination in the population of western gazelle.

In addition, the disappearance of western gazelles could be contributed by the increased number of predators. The author mentioned that 100 lions came to the East Palean Preserve as well and it was possible that these lions still prefer to predate western gazelles for some reasons, for example, the low mobility of western gazelles caused them were easy to catch. 

Moreover, the reproduction rate of western gazelles was not given in the argument. If the western gazelles has a very low reproduction rate, this species was easily to die out due to the consumption of predators. 

To sum up, the elimination of the population was caused by many factors apart from the less rainfall. More survey and date needed to be provided to fully discuss the real reasons of it.

The editorial argues that large number of predator is the culprit of Western gazelle’s

extinction. However, its author fails to provide sufficient evidence to permit a proper evaluation

of the argument’s reasoning. Each point of deficiencies is discussed separately as follow.

One of the argument’s deficiencies involves the claim that 100 Western gazelles

disappeared in Eastern Preserve. The author provides scant information about how to confirm his

assertion. Lacking such evidence, it is entirely possible that all of the Western gazelles are

vigorously alive in Eastern preserve, but was not distinguished because of its appearance are so

much alike that of Western peers. Or perhaps, the immigrated gazelle are vast scattered in the

new habitat, which deter researchers to find one or fewer of them easily. Without ruling out such

scenarios, the arguer cannot firmly conclude that Western gazelle have already died out, let alone

further investigation about the reason that may responsible for this extinction.

Another of the argument’s deficiencies is that the author unreasonably believes that large

number of predators must be responsible for Western gazelle’s death. There are myriad of causes

that may lead to this extinction or disappearance. Climate, for instance, may become the biggest

challenge that encountered by new immigrant. Although we are informed that there are less

distinctions in rainfall between two Preserves, the writer gives no specific information about

Page 159: ARG-FINAL

temperature, moisture. These are important factors determining what kind of grass or plant

Eastern preserve will grow, and whether Western gazelle can adapt to this new environment.

Also, competition between gazelles’ may bring new-comers to the brink of death. Eastern gazelle

and its Western peers is close relative, and they may have the same or similar habit of eating or

favoring certain kind of grass. Fierce competition arises when new-comers attempt to struggle

for a territory and get fine grassland. And unfortunately, established position is hard to be

removed and this battle would uncontrovertibly result in defeating of poor Western gazelle.

Furthermore, gazelle may have a short life span, and one single gazelle can never live longer

than 3 years or even less. Other factors, like devastating epidemic or plagues, health-related

problems and so on may also cause the death of Western gazelle.

Yet another editorial’s shortcoming has to do with living surroundings of immigrated

Western gazelle. It’s entirely possible that these imported animals are barred by artificial fence

which prevent gazelle from running away, which may starve them to death when grasses in

surrounding area diminished. Or perhaps these new-comers don’t really like the area where they

were placed, they eager to get rid of some uncomfortable places where number of west gazelles

is supervised and recorded, and consequently fewer or no survival western gazelles are

documented.

In sum, without specific evidence about how to record and trace the number of western

gazelle, without specific living conditions about this new imported animal, and without

throughout evaluation of the disappearing of western gazelle, the author cannot draw a irrational

conclusion that large numbers of predator is the culprit of western gazelle’s death or disappear.

In this little story, the writer attributes the elimination of western gazelles entirely to the larger number of predators in the eastern preserve on the unsteadily single basis that the slight variation in climate wouldn’t be serious enough to cause the extinction. This, however, is surely groundless due to the following three respects.

                                       To begin with, it is conspicuously stated in the text that eastern gazelles are not exactly the

same species as the western one, and infering from the case of lions and other animals, the number of eastern gazelles definitely outweighs that of the western gazelle. So it is reasonable to conjecture that there will be some kind of cannibalism and competition between these two tribes. Taking the example of human beings and chimpanzees as an illustration, chimpanzees and other monkeys alike are human’s close relatives as well due to the same historic origin descended from ancient times, but attending to the population of human and chimpanzees nowadays, it is not difficult to notice the increasingly bigger gap between these two figures. Therefore, there is no cogent evidence showing that the always harmonious coexistence of two close related species.

 In addition, it is erratically improper to assert that the slight reduction in rainfall cannot be

the cause of this elimination, or in other words, the premis which lay at the bottom of the writer’s argument is actually questionable. As we all know, plants and vegetations rely on the rainfall to thrive, so the precipitation and other indicators of the rainfall can amount to different vegetation

Page 160: ARG-FINAL

mantled the ground on which the specific species feed. Hence, the meer small difference in the amount of rainfall can actually cut off the foor supply of western gazelles, and further resulted in the poignant elimination.

 Thirdly, the number of gazelles being transferred to eastern preserve is limited, more

precisely, 100. This minority of gazelles not only should they show their pluck and readiness to fight for places and lands to settle down, but also must adapt themselves to the changing ecological condtions around and even variate their inclination for food consumption. All these changes take time, but due to the low number of gazelles present as a whole, it may be possible that they have run out of their time before they can fully adjust to the new environment.

 In short, the elimination could be resulted from other reasons such as cannibalism and

competition within their relatives, lack of food supply, and the small number of gazelles in the first place. There are also other factors which can be lethal as well such as their lack of immunity to certain parasites and germs flourishing only in the east and some pandemic diseases only discovered in eastern preserve. Taking all this flaws and possibilities into consideration, we can confidently state that the writer’s argument is groundless and need further inspection about the subject matter

ARG-92

The Leeville Chamber of Commerce asserts that the relaxed pace of life in Leeville leads to

the two facts that Leeville take fewer sick days, and that the diagnosis of stress-related illness is

proportionally much lower, and falsely assumes that the relaxed pace of life is the only reason

why this situation exist. However, the assertion is plausible at the first glance, close scrutiny of

the argument reveals that there are many other more persuasive explanations indeed, including

the structures of production, both natural and social environment, and the variance in attitude to

illness.

To begin with, an alternative explanation is that the structures of production in the Leeville and

Mesonton result in the differences in the aspects that this argument states, and the relaxed pace

of life is just the superficial explanation. Leeville, a small town, is more likely to rely chiefly on

a less industrial structure of production, such as some producing handwork products and tourism,

a economical state that results in a slow pace of life. In contrast, as a large city, Masonton

develop a much competitive structure of products, a city that filled with high-technology

products, financial service and transformation of goods. The nature of industry decides the

superficial phenomenon of pace of life, and results in fewer sick days and fewer stress-related

illness. 

Page 161: ARG-FINAL

Additionally, another possibility is that it is the both natural and social environment in

Leeville and Masonton that engender the differences. The distance of 50 miles is long enough to

be a different natural environment. People who lived in Leeville may share the fresh air, the

clean river, and the healthy fresh vegetables because of the less industrial, while residents in

Masonton may breathe the polluted air and eat unhealthy fast food, a life style that is tend to

cause illness. Moreover, busy in workplace, residents in Masonton cross among the tall buildings

and walk among cars. A more competitive survival environment leads to a more pressure of

reality. That make people feel ill more, even experience more relate-stress illness. So the

phenomenon can be understand very well

Moreover, it is also likely that variance in attitude to illness between Masonton and Leeville

make the results look so different. People in Leeville are prefer deal with the illness on their own

to turn to doctors or even if got a cold, they will not ask for a leave and still go to work. And the

contrary habits suit for people in Masonton. That is to say, not the number or rate of people in

two places, may be they are the same, but the attitudes to deal with illness make the results seem

to be different

Looking into one of the facts presented by the argument, which is the reported fewer sick

days taken by Leeville workers, I find that it is unwarranted in proving less sickness in Leeville.

People do not necessarily take sick leaves only when they are really sick. If the Masonton

workers have been consented to take quite a large number of paid sick leaves, which is quite

common in the company policies of many big corporations in cities, it is entirely possible that

they sometimes would malinger, or “act sick” in order to escape from work responsibilities for a

while. If there are insufficient number of qualified doctors in the small town Leeville, workers

might feel that taking sick leaves to see a doctor is a waste of time and money, and thus choose

to keep working even when they are feeling sick. In this line of analysis, the argument illogically

deduces that less workers in Leeville fall sick, based on the reported sick leaves taken. Therefore,

its support for the conclusion that less sickness in Leeville is due to its relaxed lifestyle is

unsubstantiated.

Another fact presented by the argument, which is the proportionately less stress-related illness in

Leeville, is also faulty after careful examination. Although this reasoning takes into account the

factor of proportion, it fails to consider the impact of absolute number of residents on the

reliability of statistics. Consider the scenario when there are only 100 residents in Leeville and

100 million residents in Masonton: one wrong diagnosis on the mental health of a Leeville

resident can already affect one percentage in the statistics chart. While the statistics on the

percentage of mental stress gathered from Masonton may be objective, the one gathered from

Leeville residents might be heavily influenced by some anomalous singles, due to its small

Page 162: ARG-FINAL

sampling size. If there is a big family in Leeville whose family members are all genetically

slightly more resistant to mental stress, the small town may report a much lower percentage of

stress-related illness among its residents. Such comparison with the result of Masonton is unfair,

and thus it bolsters nothing towards the conclusion that the relaxed lifestyle in Leeville causes

less mental stress.

In the final analysis, the argument is weak in several aspects in its evidence as well as its final

assertion. To strengthen it, the author needs to provide information which can truly prove

workers in Leeville are indeed less inclined to fall sick, such as results of medical examinations,

instead of relying on counting sick leaves. The author also needs to ensure sufficiently large

sample in Leeville to do fair comparison. Lastly, the author needs to rule out all other

possibilities which can benefit the health of Leeville residents more than that of Masonton

residents

ARG-96

The author recommends starting a ‘Shakespeare in the Park’ program and predicts that it will

lead to increase profits for the Classical Shakespeare Theatre. To support this recommendation,

the author states that Avon Repertory Company start a ‘Free Plays in the Park’ program two

years ago and have increased 10 percent profits since then. This recommendation is filled with

unsolved questions, which make it open to doubt.

First all of, to evaluate the recommendation, the author must provide enough information about

whether the ‘Free Plays in the Park’ program lead to Avon Repertory Company’s increased

profits. The mere fact that profits increased after the program is insignificant to support a causal

relationship between these two issues. It is possible other measurements taken by Avon

Repertory Company result in its rising profits. For example, if the company made more powerful

advertisements or they declined the ticket price to attract audience, then the profits may have no

linking to the ‘Free Plays in the Park’ program. To this extent, the author’ assumption that ‘Free

Play in the Park’ program lead to A von Repertory Company’s increased profits is unwarranted.

Even assuming that ‘Free Plays in the Park’ program is the reason of Avon Repertory

Company’s increased profits, we also need more information to answer the question whether a

Page 163: ARG-FINAL

similar program will increase Shakespeare theatre’s profits. The author’s comparison between

these two companies is incomplete.

It is possible that the repertoire of Avon Repertory Company is totally different from

Shakespeare’s. If the repertoire of Avon Repertory Company is more suitable to perform in the

Park, while Shakespeare is not suitable, then the author might not increase after the program. It is

also possible that audience in the two areas have different opinion towards in park program. If

the Bardvill’s audience don’t like park performance, while Avon Repertory Company’ audience

like it very much, then even the same program taken in Bardvill by Avon Repertory Company,

the result will be different: not additional profits can be gained. As a result, running a park

program is unreasonable for the Theatre. Lacking information about these two companies, the

author’s assumption that similar program will lead to profits for their company is unfounded.

Finally, even though the income increases by the Park Program, the author’s conclusion that the

profits will increased is doubt. The author fails to consider the cost of this program. If the

increased income is less than their investment, then the profit will decrease instead of increased

as the author predicted. To the sum, this argument is filled with doubt questions, which make it

unpersuasive as it stands.

ARG-100

The author concludes that Living in small towns might not contribute to the health of people as

reported before. To strength his conclusion,the author makes two comparisons:one about the

days of sick leave taken by individual workers and businesses and one about the number of

physician for one thousand residents,between Leeville and the nearby large city of Mason City.

So interesting evidence the author makes,which has some clear faulties as below.

Firstly,The ratio of individual workers who ask for sick leave to the total workers in

Leeville ,and that in Mason City should be considered, and then the author should take these to

make a comparison . It is entirely possile that the reason of fewer reports in Leeville is the fewer

population of the resident there, so the author must provide additional data such as the ratio of

the number of applicants for days of sick leave to total number of the workers.Furthermore, the

author obviously overlooks the fact that there are so many reasons for a individual workers to

applicate for days of sick leave. Perhaps the workers in Mason City applicate for a sick leave

only because of tiny illness,such as cold and flu,while the workers in Leville do not.  

Page 164: ARG-FINAL

What is more, the author does not mention the effect of the pace of life on the health of people

throughout the articles .Clearly,the mannar of living in Leevile and the pace of life in Mason City

are both neglected so it is irresponsible to make the judgement that the living in Leeville must be

enough cosy ,and that the intense pace in Mason City compel people to rush about the life among

the city.That is unbeilievable because if only the pace of life in Leeville is more hectic than that

of Mason City, the conclusion the author made would be a scoff.

Additionally,the evidence that the ratio of the number of the physician to one thousand residents

would also be unwarranted. The author seems to tell us that there is more physician ,there is less

health in people which sounds so ridiculous.The number of the physician tend to be determined

due to the economic power of the town, Maybe the physicians are attracted by the abound

salaries in Mason City, or perhaps traditionally people in Mason City are proud of being

physicans . The author hardly establish the causal relationship between the ratio of the number of

the physician and the health level of the residents. Let alone that the longevity does not equate

with the average age.

Finally,the author's recommendation that people seeking health should consider moving to small

communities seems to implicate there must be some negative affections on the health of people

in big cities,yet without any evidence to prove it. At the very least, the author should provide the

documents of pollution level comparison between small towns and large cities,or a survey among

the residents living on both places. None of them can we see.

In sum,the author makes unpersuasive conclusion with so many logical flaws,which is

interestingly unconvincing. To prove that the relax manner in small towns is superior to the

hectic lifestyle in big cities for the longevity and the better health,the author should describe the

lives on both sides and then make a comparsion,and provide more general and justified

evidences to support the recommendation,not merely cite some useless phenomena.

The passage argues that small communities provide better health and greater longevity. Although

the author provides research evidence and statistics to support his/her argument, his/her

assumptions of using these evidences are questionable. 

Page 165: ARG-FINAL

First of all, by using the argument of the research evidence, the author assumes that all small

towns are relaxed and all big cities are busy.  The author also assumes that the comparison of

Leeville and Masonton applies to all of the small towns-large cities comparisons. In fact, not all

of the small towns are on the same pace, and neither are the big cities. For example, Seattle is a

relatively relaxed city compared to New York, and Palo Alto can be a busier town than Salt Lake

City. One cannot use a specific case to generalize to the entire category without careful sampling

and analyses. 

Second, the author assumes that the days of sick leave could measure the rate of healthiness. This

data might be able to represent to some extend the healthy ratio of the city or town, yet the author

fails to regularize the data in order to make an unbiased comparison. For instance, we need

information about the allowable days of sick leave. If the allowable days of sick leave in Leeville

are in general less than that in Masonton, then this measurement is not valid. Also, one should

look at how people use days of sick leave. If the rules in Masonton are looser than that in

Leeville, one can expect that Masonton's data be inflated. As a result, a regularized data is

required to become a valid measurement of the healthiness of a town or a city.  

Third, it assumes that fewer days of sick leave indicate a healthier life. Despite the validation

procedure provided above, this assumption is still doubtable. After all, days of attendance and

sick leave are related to more factors than health. It could because that people in Leeville are

more diligent than those in Masonton; or there are more local holidays in Leeville than in

Masonton so Leevillians can take days off in the holidays; or the doctors in Masonton only work

on weekdays, whereas Leeville doctors work on both weekends and weekdays. Above all,

making money and getting promotion might be more important to people in Leeville than those

in Masonton, so that they work harder and take fewer days of leave. 

Fourth, the assumption that the physicians to residents’ ratio can be a measure for longevity is

also lacking of sufficiency. It could because that there is a medical school in the city that many

students decide to start their clinic or to work as a physician in the city. In this case the work load

of physicians in the city might be lower than those in the town, opposing the author's first

assumption that life pace in little towns are all slower than those in the larger cities. 

Page 166: ARG-FINAL

In general, the assumptions that the author provided cannot support his/her argument sufficiently.

More detailed analyses of the data and the research is required before the assumptions are made.

For example, the author can look at more small towns and big cities via statistical vehicle, and

then starts to make further assumptions based on that result.

ARG-107

The argument above indicates that the radio station Know should shift its programming to a continuous news format. To bolster the idea, the speaker shows that the increased number of older people in the listening area despite the decreased number of total listeners. In addition, the speaker insists that the decreased sales of recorded music represent the decreased interest in general music. To further support the argument, the example of news station in neighboring cities is shown with the recent survey result before the election. It seems quite plausible at first glance; nevertheless, there are several weak and porous grounds which lead the argument into the flawed one. 

In the first place, the argument assumes that the increased number of older people means that the news program will be more popular. It seems so reasonable in a general aspect, but there is no logical evidence that older people prefer news program to other music program. There could be a myriad other alternative assumption. What if, for example, the old citizen would prefer the TV news program rather than the radio one or what if they would never have interest in such political or other formal information? Without nullifying such assumptions, the argument cannot be well supported. Moreover, the reason why the total number of listeners has been decreased should be reconsidered in the independent perspective.

In the second place, the argument assumes that the decreased sales in music records would represent the decreased interest in listening to radio music program. However, common sense tells me that people usually like to listen to music through the radio programs conveniently rather than buying the records. In this view, the idea that people would not like the music program is invalid. Moreover, even if there has been a real decline in interest in listening to music genre, the rock-and-roll genre could be an exception as many people still love it and enjoy listening to the rock-and-roll music program.

In the third place, the argument falsely suggests that the new stations in neighboring cities could have similar conditions with the KNOW. However, as citizens in each city could have different preference and there could be other different variables which affect the effectiveness of each radio station, such generalization seems to be unreliable. To prop the argument, the speaker should show that both two cities have quite similar conditions and the preference of citizens and other factors are almost the same. Furthermore, the survey result shown seems so flawed. Common sense tells me that people generally come to be interested in the news program and other political information just before the election. Thus, the result fails to represent the normal and general view of the citizens.

In conclusion, taken as a whole, the argument is invalid. In order for me to accept the argument the speaker should show more data that older people in the city willingly like to listen to the

Page 167: ARG-FINAL

news program and the number of such people is significant enough to provide the benefit to the KNOW. In addition, I need more information which shows that the conditions of the radio station and citizens in other cities are quite similar. Moreover, other survey result for the citizens or radio listeners’ attitudes in different time periodsThe author concludes that KNOW radio station should change it programming from rock-and-roll music into a continuous news format. To support the argument, it is pointed out that the increasing percentage of older people in the listening area with the decreasing total number of people and the success of continuous news stations in neighboring cities. In addition, it is cited that a survey reveals the high concern about politics. However, the author’s argument is not convincing me in some points.

First, the author has the poor assumption that older people would like to listen to the news, compared with younger people. However, there is no direct evidence to show the tendency. In fact, older people living in the area covered by KNOW radio station would prefer to appreciate the rock-and-roll music. After retiring from their workplace, they would stay here to enjoy their pastime with rock-and-roll music. Furthermore, the news program has been already well serviced by the competitive and famous local networks, which even offers the vivid moving pictures. If so, KNOW radio station’s new policy would be in failure, thanks to the existed news station or networks most people have usually viewed. Therefore, the author should survey the older people’s tendency and the other news station’s existence before transferring the original policy.

The author assumes that the decline of sales in recorded music will provoke to reduce the interest in rock-and-roll music. However, thanks to the other outside factors such as the decreased income of buyers and the depressed local economy, the sales of recorded music would be deteriorated. Furthermore it would be entirely possible that KNOW radio station’s vivid and live rock-and-roll music would cause the sales of rock-and-roll music sales, even though the other genres of music’s sale is sharply decreased. Therefore, the author should investigate whether the popularity of rock-and-roll music is really decreased or not.

The author regards the area covered by KNOW radio station as the neighboring cities. However, this area’s characteristics would be very different. In fact, the neighboring cities have just constructed by government. As a result of the fact, most people is now focusing their interest on the administrative policies and real-estate prices or something like that, all of which make them get the concern about the local news. However this area would be built up for the older people, who tends to stay here peacefully without any concerns. Therefore they would rather want to listen to music like rock-and-roll except for news. Or it would be entirely possible that well-developed networks are well serving the local news or high-speed Internets are well serviced for the old.

Last, the author is based on the assumption that the survey demonstrated the interest of news, especially about politics. However, considering the fact that the survey is researched before the election, the result would be very biased, and it would be not useful to utilize the result of it as the people’s tendency or preference for the local news. Actually, after the election, most of the people do not be concerned about the local politics and the related news. Therefore, the author should find the other survey about the general tendency of older people for the radio station’s programming.

Page 168: ARG-FINAL

All in all, the author’s argument is not persuasive for me as it stands. To strengthen the argument, the author should suggest more information about the older people’s preference for news program. To better asses it, the relationship between the decline of recorded music and the interest of music and comparison between the area and the neighboring cities should be investigated more. Last, I would need the other result of survey researched at the time when the special events are not existed.

ARG-111

The personnel director recommends to president, the "Easy Read" course to all of the employees

in Acme Publishing company. He does so, on the premise that the course will render the same

benefits as experienced a graduate and an employee. Additionally, the director assumes that the

cheap cost, additional benefits and speed reading advantage will drive most of the employees to

join the course. However, the argument lacks a lot of evidence, which need to be established, in

order to effectively evaluate the credibility of the argument.

First of all, the argument mentions the act of Easy Read Speed-Reading course being introduced

in other companies. The nature of these companies must be established. Otherwise, we will not

be able to compare these companies to Acme Publishing company. Only if the work performed

by the employees in the companies is similar, can we think of a comparison. Hence this is the

prime evidence needed for evaluation.

Additionally, the director draws a hasty conclusion by examining two persons - a graduate of the

course and an assistant manager. He commits the sampling fallacy. Two persons will not be

enough to judge the effect of the reading course on all of the employees in other companies.

Moreover, the type of report presented to the graduate, who read this 500-page report in two

hours must be mentioned. If the report happens to consider lucid language, then the results

derived cannot be accurate, and will vary for reports with high level of vocabulary. The graduate

who rose from the post of assistant manager to vice president, might not have done so, solely

because of the reading course. Other factors that contributed to the promotion of this employee,

must be analysed thoroughly and compared with effects of speed reading course.

The president quotes that the "Easy Read" course will cost 'only' $500 per employee; however,

the financial status of the employees, and comparison of this fee with the salary they receive etc..

must be established to discern that the course is affordable for the employee. Additionally, it is

not clear whether the additional benefits like three-week seminar and subscription of Easy Read

newsletter will lure the employees. Some of the employees may not have sufficient time to spend

Page 169: ARG-FINAL

for the seminar or they may not prefer the subscription of the newsletter. And, how the seminar

relates to the "Easy Read" course must be clearly mentioned. Unless the seminar conducted in

Spruce city is primarily concerned with tactics of speed reading, it is unlikely to benefit the

employees. Hence the relevance of the benefits to the course being opted must be clearly

mentioned and the cost factor needs to be given attention.

Above all, the author plunges into a hasty conclusion, stating that, the Acme company will

immensely benefit out of coercing its employees to enroll in the "Easy Read" course. When the

benefits of the course are not clear at first hand, it is almost impossible to end the argument at

such a positive note. Further details such as: the number of employees in the Acme company,

their level of work and whether their work entails speed reading must be taken into

consideration. Only then, the readers can arrive at a conclusion as to how exactly the "Speed-

Read" course suits the employees of Acme Publishing company.

Thus, the recommendation from the personal director needs more information in the form of

evidence to his hasty conclusions and generalizations. The evidence required include: nature of

the companies, applicability of the benefits achieved by two persons to the whole lot of people;

the difficulty level of the reports being tested for efficiency; other factors that elevated the post

of the employee; the affordability of the employees to purchase a "Easy-Read" course; and, the

suitability of the course to the employees of Acme Publishing company. Unless, these evidence

are established, it will not be possible to consider the recommendation of the personnel director.

In a quick glance of the recommendation above it may appear that personnel director's words sound very persuasive, but in a more in-depth view, one can encounter many flaws in the assumtions of the author.The author's line of reasoning is based on the fact that due to studying Easy Read Speed-Reading Course the productivity of Acme Publishing Company employess will increase significantly, consequently the company will benefit from this fact. The articel is unconvincing for several reasons.

First of all, the personnel director highlights the fact that after graduating from Easy Read courses one can read 500 page report in 2 hours.The director doesn't pay attentiont to the quality of such reading. An employee can just look through the report paying no attention to details.Reading of any document requires not the high speed,but a deep understanding of main problems and facts described in the document.Thus, the fact that Easy read Courses graduate can finish a large volume report within several hours doesn't sound convincing.

Secondly,the personnel director asserts that after graduating from the Course one manager was promoted in his company,and achieved excellent results during only one year.Obviously,the main requirements for vice-president of a company include a huge amount of responsibilities. A person,occupying high position,should have much more skills than just the ability to read

Page 170: ARG-FINAL

quickly.Thus,I can conclude that there are many other reasons why an assistant manager became a vice president of the company,besides taking Easy Read Course.This example can not strengthen author's reasons a lot.

Thirdly,the personnel director doesn't provide any statistical data in his recommendation.This article fails to correlate,for instance,the number of companies whose employess graduated from Easy read Course with the number of companies whose productivity has increased significantly.Statistical information could greatly contribute to the author's reasons.

Finally,the personnel director mentiones that the Course costs 500$ per employee.There is absolutely no confidence that Acme employees will return the expenses after they graduate from the Courses.Also,the Course requires a 3-week seminar in another city,which requires more time and more expenses on transportation to another city.

In conclusion, I would like to say that Acme's personnel director recommendations consist of many illogical assumptions.Consequently,the article doesn't provide enough information and doesn't let the reader to make the right conclusion about easy Read Course.

ARG-130

In this letter, the author deems that to assuage local traffic accidents, all the students should take

the driver's education course, and due to other reasons, only the course open at Centerville High

School can be available. After examining the evidence cited by the author to support the

recommendation, however, several questions appear and need to be answered by the author, or

they will weaken the reliability of the argument.

First, the author mentions several accidents involved teenage drivers happened during last two

years and unfairly assumes that teenage drivers should take all the responsibilities. But here is

the question, is there any survey or statistics to prove this assumption. Traffic accidents can

result from many factors, like climate, road condition, or geography of certain section. Even they

may be caused by human factors, no evidence there confirms that students are the troublemakers.

Besides, they may be victims and not even drivers in the accidents. Therefore, unless the author

collects all the information and sets a well-rounded survey, the conclusion is too rash to make.

Second, even if it is the students' who should be blamed for the local traffic accidents, questions

are still remained: what did those students do to cause those accidents and whether those

mistakes are only owing to their driving skills? Without any evidence, the author jumps to the

conclusion that every students who got involved in the accidents have poor driving skills, so to

forestall the repetition of same errors, all students should take the drivers education course.

However, the author fails to take other possible reasons into account. Perhaps some of those

Page 171: ARG-FINAL

accidents happened because of carelessness. For example, some of students who may possess

proficient driving skills picked up the phone or drowned themselves into the loud music during

their driving but failed to judge the situation right on time, then the tragedies happened. If true,

all they need maybe warning and punishment, and not force them to study all over again with

things which they are already so familiar. Hence, different needs call for different assistance, and

there is no necessary for all the students to take a driving course.

Finally, even though students are the troublemakers and also having little knowledge about

driving, the author's recommendation depends on the assumption that no solution other than

accepting the course offered by Centerville High School can be the best choice to assuage the

problems. However, lack of information of details about the school limits the assumption. The

qualification and cost about the school opening a course are two critical factors the author

neglects. Can the school provide or hire experienced and qualified teachers to give the lectures?

Can the school afford those expenditures? Both of the questions can cast doubt on the author's

solution. Without ruling these and other possible negative factors, the author cannot justifiably

concludes that only by taking the driving course at the high school, students can perfect their

driving skills.

To sum up, a myriad of questions hidden behind this recommendation which weaken its

reliability and feasibility, although the argument is written by the author out of consideration

about local traffic safety. Until the author provides further evidence to exclude at least relative

factors mentioned above, hardly can any substantial improvements gain even the

recommendation put into action.

Two, even if accidents are caused by teenage drivers, the author provides no clear evidence to

justify the assumption that there are a large percentage of parents who are too busy to teach their

children to drive. It is entirely possible that only a few of parents have no time to teach their

children driving courses due to the busy work. Therefore, without detailed data of the quantity of

parents having no time to teach their teenagers, the conclusion is still unpersuasive.

Three, the author irrationally assumes that the school has enough funding to sponsor the program

and that the program makes effect. In all likelihood, the program needs a great deal of money to

run and the school have no superfluous funding to sponsor it. What is more, the program might

be of no effect, and this scenario is quite possible. However, the author overlooks these

possibilities thus the conclusion is ill-conceived.

Page 172: ARG-FINAL

And finally, even the high school could afford the program and the program is effective, the

assert that the program sponsored by the high school is the only solution to this serious

problem is also irrational. Perhaps other solutions would be more efficient and economic. For

example, some lectures and brochures could be given to propagate the driving knowledge.

Without addressing the possibility the author cannot convince me that the program is the sole

solution. May be they were drunk, novice drivers, insincere and unruly, which vices

theCenterville pupils don’t have. Perhaps that none of the students has committed any accident

although driving a good length.

The writer claims that parents have complained to be too busy to act as driving tutor. However,

he did not mentioned what is the exact percentage of the parents, and their economic and

demographic facts. May be the parents who talked to the writer are all rich, who have employees

in their home to can teach the children. Or they all have sons and daughter who are in service,

who can also teach their younger siblings. Moreover, the writer also did not mentioned what

percentage of the parents are unable to pay the fees at the driving school. We should keep in

mind teaching children at school will also cost, and if the cost is greater than the possible subsidy

to the unable parents, it will be an injustice to the taxpayers who finance the school.

ARG-133

In this argument, the author advocates that all teachers in high schools in the state of Attra should

assign homework no more than twice a week, if at all. To justify this assertion, the author cites

the fact that high school students in the district of Marlee are assigned less homework while they

got better grades than the students in the district of Sanlee. At first glance, the author's reasoning

seems appealing and convincing. Close scrutiny of each of these facts, however, has revealed

that none of them could lend credible support to the assertion.

In the first place, the threshold assumption upon which the argument relies on is that the survey

is statistically reliable. In order to establish a strong correlation between the amount of

homework and the grades of students, the sample of the survey is supposed to be representative

of the overall population of the high students in the whole state of Attra. However, the author has

conducted the survey only in two districts. It's entirely possible that the high school students in

Marlee cannot stand for the other students. Without evidence of the survey's reliability, the

author cannot draw the justifiably conclusion that all schools in the state should set the schools in

Marlee as examples.

In the second place, even if the sample of the survey is representative, there is another flaw that

Page 173: ARG-FINAL

would weaken the line of reasoning. The survey is conducted among the high school math and

science teachers, while the assertion that the author has provided in the end is about all high

school teachers. The author fails to see the differences between different subjects. There is a

great chance that it's not necessary for the math and science teachers to assign much homework

to the students. But thisexperience may not apply to teachers in other subjects, such as

arts, literature and so forth. Without taking the differences between subjects into consideration,

the author draws the conclusion so hazily that it's unpersuasive.

In the third place, better grades of students in the district of Marlee may not lead from the

smaller amount of homework, but something else. As it's known to all, the grade is a complex

factor effected not only by one thing. It's possible that what the teachers in Marlee give to the

students is much easier to understand than that in Sanlee. It's also possible that most of teachers

in Sanlee are more serious about the test and it's harder for their students to get a grade as high as

the students in Marlee could make. Therefore, since the author has not ruled out all of these

factors that make contributions to the grades, I cannot agree to the assertion based on the

incomplete comparison.

To sum up, the author's argument relies on certain doubtful assumptions and an incomplete

comparison that render it unconvincing as it stands. To bolster and better evaluate the advocate,

the author is supposed to take another new survey to make sure the sample is representative and

take other factors that have influences on the grades into consideration. Only in this way can the

author come up with an effective way to improve the grades of overall students in the state of

Attra.

First of all ,the survey must be showed to be reliable before I can accept any conclusion the

author reaches based upon it.However,the arguer fail to prove that the responses are accurate or

that the teachers are statistically significant in number.Additionally,whether the sample is

representative deserves to doubt.

In addition,the argument assumes that the math and science can reflects all subjects ,but it lacks

credibility ,because math and science have their feathers which are different from others' .Even if

the survey is credibility, it is all likelihood that the other subjects except math and science need

homework everyday.In short,lacking evidence that math and science are typical in terms of all

subjects,the author cannot convince me that teachers in Sanlee high schools should assign

homework no more than twice a week.

Finally,the argument unfairly claims that better grades is the er result of assigning homework on

Page 174: ARG-FINAL

more than two or three day per week rather than some other phenomenon.the arguer ignore a

host of other possible reasons for better grades.Perhaps students in Manlee have the better ability

to study and they study themselves without homework.Or perhaps teachers in Manlee are better

than those in Sanlee.In short,without ruling out all other possible explanations for the better

grades,the author cannot convince me that the less homework is the only reason for the better

grades,let alone reducing homework.

In sum,the conclusion relies on the invalid and misleading assumptions that render it

unconvincing as it stand.To bloster the conclusion,the author would have to substantiate that the

students from the two places have the similar ability to study and that whether the quantity of

homework in the different districts are same.Moreover,I would suspend my judgment about the

credibility of this argument until the author can provide more information about the relationship

between the grades and the homework and whether math and science and the same feather of the

other subjects.

ARG-140

Though this argument makes some good points, it does not provide enough evidence for its

central premise: protective gear will greatly reduce the amount of roller-skaters being sent to the

ER. 

The main flaw of the the argument is its presumption in that by wearing protective clothing

(helmets, knee pads, etc) or light-reflecting material (clip-on lights, glow-in-the dark wrist pads,

etc), the incidents of ER roller-skating patients will be greatly reduced. The argument never

considers other situations that can harm the roller-skater. Perhaps the streets or parking lots

where 75 percent of the people who had the accidents were clogged with cars and people.

Perhaps these factors increased the roller-skating accidents moreso than the protective gear ever

had done. It is an uncontrollable and unfortunate situation when one sees a car heading straight

for them and cannot avoid it. Protective gear will not help in this case. 

Another situation the argument never considers is the chance of dogs being let loose. These dogs

may be also attacking the roller-skaters, thus the accidents did not rise from lack of protective

gear, but from animals that should not have been set free in the first place. 

Yet another example is perhaps the roller-skaters were racing each other down the street. They

could have collided into each other or into other objects, such as a bush or a tree. This again

Page 175: ARG-FINAL

weakens the idea that protective gear will save in these types of situations. 

A way to fix this undesirable situation may be to, not invest in high-quality protective gear, urge

the roller-skaters to go to a local roller-skating rink or build a park that is for roller-skaters only.

This will be the safest way. Here, the roller-skaters can invest in their own protective gear, which

may or may not reduce the accident rate. The biggest advantages of the rink and park are that it

will get them off the streets and parking lot and allow them to have a place of their own. Thus, if

there is a problem with lack of protective gear, it will surely be determined when all other

uncontrollable factors have been precluded. 

Though this argument does have some valid points, such as implying the importance of

protective gear for roller-skaters, the readers cannot ascertain that the lack of this gear is the

cause of so many ER patients. If the author wishes to improve the quality of his argument,

perhaps he should describe other factors, that cannot be controlled, which can also increase the

accidents. Cars, people, dogs, and racing, are just a few examples. Additionally, instead of

assuming that the protective gear will solve most of the problems described inthis argument, the

author should look at other constructive ways to help the roller-skaters, such as enforcing rules to

let them skate in a rink or building a park suitable just for them. If these factors are taken into

consideration, then the argument can be more sound.

This argument has presented some statistics about injured people in a hospital who have come to

emergency room after a skate rolling accident. Based on this statistics, the writer has inferred

that by more investigation on high quality protective and reflective equipment, the risk of

accidents would be reduced. The stated assumptions are inadequate to deduce the conclusion

statement. Furthermore, there are several unstated assumptions that can’t direct us to the stated

result. 

The writer claimed that 75 percent of injured people who had accident in a street or parking lot

haven’t had any protective or alarming equipment. But the writer hasn’t noted the state of these

injured people before the accident. It may happen for a pedestrian who is not a skate roller at that

moment to go to hospital after a collision with a skate roller in the street or sidewalk.  

The severity of injuries also should be considered in drawing conclusion of more investigation

Page 176: ARG-FINAL

for equipment. Are all 75 percent of people injured badly or they just have a scratch on their

skin? On the other hand, someone who has worn protective equipment may be injured severely

in an accident. 

It would be better that writer explains the status of other 25 percent of injured people who used

protective and reflective equipment. Despite using such equipment, they have injured in

accidents. This case indicates more clearly that investing on high quality protective and reflective

equipment may reduce the risk of being injured in roller skating accidents.

At the argument it is not clear that how the person was hurt and then attended the emergency

room. Does happen an accident for him just because of not using the warning and protective

equipment or he has another health problem? Some other diseases or health conditions of the

person like being out of breath or dropping the blood pressure could take the person to

emergency room. 

How much of equipment are needed that skate rollers feel safe during their practice. The same

question can be raised about the degree of quality. It is clear that even using the most high

quality equipment may not be able to protect the skate roller from a hard clash. By using the

variety of protective equipments, but not being trained very well, someone may be damaged

easily.

Considering all above points, the statistics at the argument should be changed to more precise

and thorough to be proper for inferring stated conclusion.

1.The author gives no information about how the hospital statistics are collected, thus he cannot

base on it to reach any credible conclusions.

2.The author fails to compare the severity of accidents between roller-skaters wearing the

protective equipment with those who had not been wearing, thus we cannot judge that the

protective equipment really play roles in preventing them from injuring. 

3.The fact that skating in streets or parking lots is dangerous to begin with may be the major

reason for the high rate of accidents in these places, regardless of the protective equipment.

Page 177: ARG-FINAL

4.The statistics collected in streets and parking lots are not representative of overall situations of

roller-skaters.

5.There are many other factors influencing the incidence of accidents in streets or parking lots

besides the author’s consideration.

6.The author fails to consider that only by investing in highly-quality protective gear and

reflective equipment is insufficient to reduce the risk of being severely injured in an accident.

ARG-141

It seems reasonable to agree with the budget planners advice that the city should eliminate its

subsidy to the symphony at first glance. However the author's argument is far from well-

reasoned on a second thought for it lacks convincing evidence to support the assumptions of the

author. To fully evaluate the argument, more detailed information should be provided in order to

make a wise decision.

First of all, by stating the fact that the symphony invited an internationally famous conductor and

the ability of the conductor to attract high-end musicians to join the symphony, the author

unfairly assumes that the symphony must be winning profits. However the author fails to

recognize that many other factors aside from the performers will eventually influence the

financial condition of the symphony. The symphony will have to pay more for the famous

conductor and musicians for their fame and virtuosities thus the cost of the symphony is largely

increased which may result in a worse financial condition for the symphony. Or it is possible that

the high-profile musicians would require a higher standard for the performance sites which also

impose heavy burdens on the symphony' financing situation.

Building upon the implication that the private contributions have been tripled and the popularity

of the symphony is at its peak, the author believes that the symphony has achieved the objective

to be self-sustaining. But the author is ** a hasty decision without any grounded evidence.

Indeed a growing subsidy from individuals or more money earned from tickets will help the

Page 178: ARG-FINAL

symphony to survive on itself. The private contributions may merely cover the huge gap between

the increasing cost and the profit due to the upgrading performance sites and a low ticket price.

Moreover, since the attendance only increased at the symphony's outdoor concert, the symphony

may not be able to earn any money from the performances since the outdoor concert are always

free. A specific number in the profits from the financial report of the symphony will definitely

strengthen the argument while the lacking of such information make the argument much weaker

than it seems.

To conclude, merely based on rashly assumptions, the author fails to provide cogent evidence to

support his recommendation that the city government should stop funding the symphony. Even if

the symphony is self-sustain now, I can hardly draw such conclusion from the author's argument

without the detailed information and necessary explanations.

ARG-150

Obviously, the marketing director is enthusiastic about Bargain Brand Cereals' success in selling

low-priced breakfast cereal. The marketing director states that over one year, the very low prices

of the cereal took many customers away from the top-selling cereal companies and that despite

the top brand cereal companies reducing their prices and planning to introduce budget brands,

Bargain Brand has never had to raise its prices to continue making a profit. The marketing

director then comes to the conclusion that the company should expand the business and start

selling other low-priced foods as soon as possible. At first glance, the marketing director would

appear to have a good idea, but upon closer inspection one can see that more research should be

done before such products are launched in the marketplace.

In the first place, Bargain Brand Cereals' low-priced breakfast cereals have only been on the

market for one year - a very short time in terms of analyzing the overall success or failure of a

product. With such a short history, it is impossible to evaluate the long-term viability of the

product in the marketplace, particularly with respect to the actions and reactions of competitors.

The marketing director even mentions in his or her memo that the top brands have already tried

to compete by lowering prices. It is likely still too early to tell what effect that will have on the

Page 179: ARG-FINAL

future sales of Bargain Brands cereal. Additionally, the marketing director states that several of

the companies producing the top-selling brands plan to introduce their own budget brands,

indicating that they have not yet done so. He or she also states that Bargain Brands has never had

to raise its prices to continue making a profit. Although that may be true because the competition

has not yet fully reacted, the other companies are organizing a direct attack on the Bargain

Brands cereal - companies which likely have tremendous funds available for launching these

new bargain products, possibly even selling them at or below cost to try to drive Bargain Brands

cereal out of the market. Faced with these current and upcoming battles, the marketing director's

conclusion that they should launch other low-priced food products as quickly as possible might

be foolish rather than wise. The company may need to save its funds to try to survive in their

current market rather than extending itself out into more fields of competition.

Secondly, the marketing director assumes that low prices are what attracted consumers to

Bargain Brands cereal. It is possible that it was not price that attracted customers - rather the

package, promotion or the fact that the other cereals were not as good as Bargain Brands cereal.

The company may have some special advantage with its cereal that others do not have - and that

it cannot duplicate in any other types of foods. Success in selling low-priced cereal does not

indicate the chances for success with other low-priced foods. Indeed, there may be some

industry-specific factors in cereal marketing that have allowed Bargain Brands to succeed in the

short-term. Bargain Brands may have some special expertise with cereal that they cannot

duplicate with other types of food products. The marketing director presents no direct evidence

or market research to indicate that Bargain Brands can successfully expand its business into other

food areas. Without such information, the marketing director's argument is unconvincing.

In summary, without detailed market research showing the true reason why Bargain Brands

cereal has been successful, without knowing the likelihood of its continued success, and without

showing how it can translate that success to other areas, the marketing director's argument is

based on speculation and faulty logic. To strengthen his or her argument, market research should

be conducted to determine how the marketplace is reacting to the competition's strategies in the

cereal market, and whether there is a demand in any other particular areas for low-priced food

products. Additionally, the marketing director must show that Bargain Brands has some type of

Page 180: ARG-FINAL

competitive advantage that it can successfully apply to its strategies in the low-priced food

product market

In this argument, the arguer comes to the conclusion that Bargain Brand should expand its

business and begin marketing other low-priced food as quickly as possible. To support this

recommandation, the arguer sites some facts such as their very low prices quickly drew many

customers away from the top-selling cereal companies and they have not needed to raise their

prices to continue making a profit. Although the argument seems reasonable at first glance, it is

totally ill-convinced. The reasons are stated as follows.

First of all, the arguer claims that their relatively low price drew many customers away from the

top-selling cereal companies, yet this can only assure Bargain Brand to be a competitive

company, not a profitable company. Such advantage like low price can only appeal customers

just for a short time, and the most important advantage of a company is the quality of its

products. It is very likely that Bargain Brand wins many customers at first, but when these

customers realize that the taste or other characteristics are not as good as cereal produced by top-

selling companies, they will return to buy these products instead. I will not be convinced unless

the arguer gives evidence to show that this and other scenarios are unlikely.

In addition, the arguer also sites that although the companies producing the top brands have since

tried to compete with them by lowering their price and although several plan to introduce their

own budget brands, not once has Bargain Brand needed to raise their prices to continue making a

profit. Nevertheless, there is no garantee that it will make Bargain Brand a profitable company.

Common sense tells us top-selling companies will draw some customers back when they lower

their prices and hence makes them more competitive. However, Bargain Brand did not do

anything in this situation, and thus I am wondering whether Bargain Brand will continue to make

a profit. It is possible that top-selling companies make more advertisements for their budget

brands and people know more about their products than Bargain Brand's, and therefore people

will choose their products because their influnce. If Bargain Brand is not affairable to make such

advertisements like top-selling companies, how can it compete with them and make a further

profit. Without giving compellent evidence, the arguer can not bolster the recommandation.

Finally, even if the evidence turns out to support the argument, the arguer can not just simply

draw a conclusion that Bargain Brand should expand its business and begin marketing other low-

priced food products as quickly as possible but gives no information to confirm it. Other low-

priced food products may not be as same as cereal, so it is likely that Bargain Brand can not draw

Page 181: ARG-FINAL

customers away from other top-selling companies as many as it does in cereal and therefore

makes little profit. To reach this cited conclusion, the arguer must explain why none of these and

others is available.

To sum up, this argument's reasoning is not based on valid evidence or sound reasoning. To

make a better recommandation, the arguer should cite more convincing evidence and take every

possibility into account.

ARG-155

The memo of Top Dog Pet Stores’ marketing director did some survey and have a conclusion that placing their ads in Exotic Pets Monthly could make some help for their sales and profit. Since the evidences the memo gave are with some logical leaks, nobody should agree with the conclusion rashly.

In the first place, there are no evidence shows the increase of the sales are the result of the ads on Exotic Pets Monthly. There are a lot of other situations such as more pets are kept during these years, and the 15 percent increase is under the average level of the increase whole town’s stores. Another possibly situation is the Fish Emporium sells more goods in these years or sells them with a lower price, and they gain fewer profit with the increase of sales, and that is not the goal of the Top Dog Pet Store.In the second place, we do not know the exactly cost of the ads, and we also do not know the ads will bring how much profit to the Top Dog Pet Store. Namely, we can exact compute if the ads is worthy, or it will decrease the profit or even cause the deficit of the Top Dog Pet Store. And there is a contrast possible situation is most of the customers of the Top Dog Pet Store hate the ads, and they will not come again since they saw the ad. It should not be decided without the exact data about the cost and efficiency.

In the third place, no evidence shows that the Top Dog Pet Store is comparable with the Fish Emporium. There is nothing about what these two store sales. Ads, are effective for some kinds of goods and stores, are sometimes come out a controverse result in some other region. It should be think about times and carefully to make a decision while there are such less evidences.

While making a decision, peoples should do enough survey to get conditions and evidences as much as possible. Since there are so many questionable points with so little useful data the marketing director mentioned, as discussed, it should not make a decision to place ads inExotic Pets Monthly so rashly.

ARG-158

Page 182: ARG-FINAL

The President of the Amburg Chamber of Commerce recommends to install a high-intensity

lighting in the Amburg replacing its police patrols. It is based on the assumption that the crime

rates will dwindle rapidly once the high-intensity lighting is installed in Amburg. The conclusion

is based on the premise that the high-intensity lighting was very much effective in the City of

Belville and it will do so in the city of Amburg. However, lots of questions need to be answered

before one can comfortably arrive at the effectiveness of the recommendation. The

recommendation relies on unstated assumptions and lack of evidence is prevalent throughout the

argument. These need to be corrected before considering the recommendation.

First of all, the high-intensity lighting is quoted to have reduced the vandalism in the country.

Did other factors like absence of the criminals contribute towards reduction of vandalism? On

what basis, is it stated that the high-intensity lighting is the sole reason for dwindling of

vandalism in the city of Belleville? If other reasons like criminals moving away from the city of

Belleville are noted, then high-intensity lighting will have the least to contribute towards the

decrease in Vandalism. Moreover, the act of vandalism is said to have declined immediately.

This arises a doubt - whether vandalism really vanished away? Or was it the temporary

suspension of vandalism due to unforeseen circumstances?

The city of Amburg is quoted to have introduced the police patrols on bicycles recently.

However, we cannot judge the effectiveness of police patrols, unless they are instituted over a

long period of time. After immediately introducing police patrols, we cannot exactly determine

the applicability of this measure. Hence, on what basis was the rate of vandalism assumed to be

constant? The data has to be quantified in order to get a clear view of the police patrols.

Moreover, can the city of Amburg and Belleville ever be compared to each other? They must

have the same stretch of land, similar kind of vandalism level existing in each city. Only then,

can we arrive at an exact conclusion regarding the effectiveness of vandalism-control measures.

Based on the given information, we cannot arrive at the applicability of high-intensity lighting

measures to both of the areas. And the crime rate reduction in Belleville must be supported with

relevant statistics, in order to arrive at a proper conclusion.

The President suggests that the money invested in police patrols can be seamlessly transferred to

that of high-intensity lighting. However, will the amount invested in police patrols be enough to

support high-intensity lighting? Even if the money is enough, are the surroundings suitable for

the installation of high-intensity lighting? - Is there enough space available in the city? Will the

Page 183: ARG-FINAL

people agree to the installation of high-intensity lighting? These questions need to be answered

before diverting the money towards the installation of high-intensity lighting.

Above all, the police patrols are quoted to be an ineffective measure of controlling vandalism.

But, was the act of patrolling followed adhering to rules and regulations? Were the patrols active

throughout the night each day? Were their attendance monitored strictly? The monitoring of

patrols and the resulting information from observing the working of patrols must be established

to get a clear cut idea of the effectiveness of the police patrols. If the monitoring is not done

properly, then the information given in this argument is erroneous. Unless the work of police

patrols is made strict, we cannot gauge the effectiveness of this measure.

Thus, lots of questions like: criminals staying in the city of Belleville after introduction of high-

intensity lighting, statistics concerning the reduction in vandalism, comparison between the city

of Amburg and Belleville, guarantee of sufficient funds to introduce high-intensity lighting in the

city of Amburg, effectiveness of Police patrols - need to be answered. If they are supported with

proper evidence, then we can take into the consideration, the recommendations of the President.

Otherwise, it is highly unlikely that the recommendation of the President will achieve the desired

results.

In the argument above,the author concludes that the city of Amburg shoulf install the high-intensity lighting as what the city of Belleville did at last October so that the crime rates in Amburg can be reduce in a obvious degree.To support the conclusion,the author compares the rates of police patrols on bicycle recently and before which shows no improvement. Besides,the author indicates that, Amburg should give financial support to the installment of high-intensity lighting instead of supporting bicycle patrols.However,the high-intensity lighting give great help to the city of Belleville , the argument is lacking of a number of evidences that can be accepted.First of all, the passage told us that the city of Bellevilleinstalled the lighting in its central business district on last October,however, the author fails to indicate that what is the condition of the vandalism rates outside the central of business in Belleville. We can assume that if Belleville has a condition of crime more terrible than Amburg,the lighting in the business center can help to decline the rate there, but the whole city still in an extremely vandalism situation.The author fails to prove the lighting in Belleville is able to decrease the rate of vandalism of the whole city.Meanwhile,even the rate can represent the rate of the whole city, it also give a puzzle that the author collected the rate of Belleville belongs to last October but we don't know the rate of it right now.If the crime rate of Belleville is increasing in high speed but the author forget to collect,we will cast doubt on the effection of the high-tensity lighting.Secondly,the author indicate that the rate of Amburg vandalism remain constant after the police patrols business district on the bicycle.In this evidence,it is lack of interpretation about the detail

Page 184: ARG-FINAL

of the rate.We can guess that the crime rate of Amburg is not as high as Belleville. If the number of the whole city vandolism is 5 but the business district has covered 4,the rate of this district will be too large to stun the author.Although that number is an assumption,the author should clarify the rate of it in detail.In addition,the author demonstrates that the installment of the high-intensity lighting have to throughout the whole city,which give rise to the next threshold problem.Granted that the rate of the business district of Amburg is higher than you can image,the city meybe in a harmony atmosphere and without so many vandalism except the business areas.Such a safety city that won't have a great number of criminal stuffs . The evidence is absent to prove the author's idea that the whole city should be installed the high-intensity lighting.Furthor more, even the whole city sincerely need to install that kind of lighting which help for their city security,how much money should be paid? The author told to use the money which used to be spent on the patrols on bicycle,however,whether does the author know that the money is enough to become the financial support? The writer should have given out a budget about the installment plan and compared with the patrols cost which is based on the budget had been accomplished before the patrols began. Meanwhile,the effection of every public service maybe unpredictible, the budget should be including enough money to figure out the puzzles.From the logical perspective,the arguer fails to make an equivalent between Belleville and Amburg.Even the high-intensity lighting in Belleville is able to work out and the rate of vandalism inthat city is declined,there is nothing the city Amburg can be the same as Belleville,either..Such a theshold problem the author fails to solute.To sum up,the idea of installing the whole city Amburg with the high-intensity lighting still need to be prove with more evidences which can be strongly support what the author maintains

ARG-166

In this argument, the arguer attempts to convince us that we should preclude the non-residents to

be the committee of the city Oak City. To justify the claim, the arguer pointed out that, recent

meeting failed to come to an agreement on important decisions because of the non-residents

committee. In addition, the arguer assumes that only the residents live in the city know best of

the city, as well as to make the right decision. However, a careful examination will reveal how

groundless the conclusion is.

Firstly, the arguer wrongly points out that the disagreement is due to the objection of the non-

residents committee, for no statistical evidence states that most non-residents oppose the

agreement and the residents agree with the proposal. Feather more, the arguer just emotionally

claims that the objection raised by non-residents is of little use. Perhaps, the non-residents raised

a more useful advisement that did not be understood by so-called resident committee. Without

enough evidence, I will not receive the assumption.

Secondly, the fact that some residents live in the city does not ensure that they fully understand

the city in the field of business and politics. As we all know that, business and politics have wide

effects that not only influence the people living in the city, but also the populous outside the city.

Page 185: ARG-FINAL

Although a people may fully understand the living style of a city, without the various knowledge,

one cannot claim that he know the business and politics that interplay with each other.

Thirdly, I fundamentally disagree with the claim that because one pays for the city taxes, he

knows how to use the money. How to use money need to think, communicate and even calculate,

much more the action of pay taxes. The argument gives no evidence that non-residents cannot

make a right decision without paying the taxes. Without fully examination of reason of the non-

residents' objection, one cannot come to the rasp conclusion.

In sum, the conclusion is invalid. To make logically acceptable, arguer should provide statistical

evidence to justify non-residents is the main reason that lead to the disagreement. Moreover,

more convinced reason should be raised to claim that residents committees are more likely to

make the right decision.

ARG-167

The author of this argument states that the Seaside Vista tourism has increased by 20% since the opening of new golf course and resort hotel two years ago. But there is no information on whether any other big event like opening an amusement park happened that year which increased the tourism. If there wasn't any such event that year and there was 20% increase in Seaside Vista tourism only due to new golf course and resort hotel, then this would have strengthened the argument. 

Also, the rise in business loan applications has increased this year, not before two years, which means that the increase may be due to some other new business plans enforced by the government. If a report of why there is a sudden increase in business loan applications two years after opening new golf course was provided, then the result of the report would have helped in strengthening or weakening the argument. If this increase is due to the opening of golf course and resort hotel, the report would strengthen the argument. 

The author also assumes that since Seaside Vista showed increase in tax money collected after opening municipal golf course and resort hotel, Brindleburg will also have the same effects on opening a golf course and resort hotel. The author has nowhere compared the climate, transport, geography and population of the two towns. If this information was provided by the author, it would have helped very much in determining if opening a golf course and resort hotel in Brindleburg will have same effects as in the town of Seaside Vista.

Finally, the author has mentioned that there are much-needed public improvements in Brindleburg. There is no information regarding what kind of public improvements has to be done. There are certain things like good transportation and good roads etc., for tourists to select a place as thier tourist spot. If the public improvements are laying a proper road or arranging

Page 186: ARG-FINAL

proper transport facilities for the residents of the town, then there would be not be more use in opening a golf course and resort hotel without satisfying those public needs. Author could have provided some information regarding which would have helped in strengthening or weakening the argument.

Firstly, this memo mentioned that the town of Seaside Vista had built a golf course and made some gains, there is no evidence shows that that town is comparable to Brindleburg. Climate, transportation, popularity, geography, and so many else condition could influence the customers’ choice of the golf course. Successful result in the town of Seaside Vista will not succeed in the Brundlebury in a huge probability. 

Secondly, there are not enough evidences shows the huge increase in the town of Seaside Vista is because of the new golf course. The memo only mentioned these increase of tourists, loan and tax are appeared after the golf course and resort hotel, but it is no evidence can indicate the increase is the result of building the golf course and resort hotel. There were maybe some new investment of government or other company into the town of Seaside Vista to build some new amusement or museum or some else place which is attractive to tourists, and then someone cough the chance and built the golf course and resort hotel in order to gain more money from the possibly increase of tourists, and the increase of tax and loan are the result of the new investment instead of the new golf course and resort hotel. Building a new golf course is useless to the Brindlebury unless some investment will come to here either.

Thirdly, without the recently economy situation of the local government of the Brindlebuey, no enough money to build a new golf course is possibly. Government have much more important thing to do than build a golf course. The memo mentioned the government will spend the gain of the golf course to fund much-needed public improvements, and government should do this as soon as possible if the budget is enough. 

With such logical leaks of the memo from the mayor of Brindleburg, the council should not make a rashly decision to build a golf course. What the council should do is do more survey and analyze the results to reach a more availably conclusion to make the town better

ARG-168

The Vice President of the company responsible for building shopping malls, proposes to hire

Appian Roadways replacing the currently existing Good Intentions Roadways, on the basis that it

will increase the durability of the roads. He arrives at his conclusion on the assumption that, the

same effects observed in the other part of the state, will be observed in his part too. However, the

argument is rife with unstated assumptions and a number of questions need to be answered

before arriving at a conclusion regarding the proposal of the vice president.

Page 187: ARG-FINAL

First of all, was Good Intentions Roadways solely responsible for paving the surface of a section

of Route101? If some of the other companies too colloborated with Good Intentions Roadways,

then it cannot be blamed for the undesirable result. Also, did the travellers through the roadways,

adhere to the speed limits and load limits to be carried by the vehicles? If the loads have

exceeded the prescribed limits by the Good Intentions Roadways, then the Roadways is not to

blame for the potholes created.

Secondly, the president happens to compare the section of Route40 with Route101. Are these

two sections comparable? Do they have the same size? The sections must be similar in a number

of aspects, for them to be taken for comparison. Only if these factors are alike, we can gauge the

effectiveness of roadways. If the traffic in Appian Roadways has been very much less, during the

past four years, then we cannot arrive at the strength of the Roadways. What if Good Intentions

Roadways stretches for a longer distance than Appian Roadways? Then the comparison will be

futile because it will not be comparing 'apples with apples'.

Purchasing of a state-of-the-art paving machinery and hiring of a new manager, does not

guarantee the effectiveness of Appian Roadways. Does the replacing new manager, have the

same skill sets to drive employees and get the work done, as that of the previous employees?

Will the newly purchased machinery be able to give the same positive results, as that of the old

machinery? When the machinery has not been still put to use and as the manager is newly take

charge, we cannot foresee the consequences and future of Appian Roadways. Hence these

questions must be answered by the Vice President in order to predict the future of Appian

Roadways.

Additionally, will Appian Roadways be able to construct access roads for all of the new

shopping malls? Does it have that sort of man power to carry out such an onerous task? Because

the roads constructed by Appian Roadways were safe elsewhere, how guaranteed is that the

roads constructed newly will serve for four years or more? Proper extrapolation has to be done

after comparing the previously carried out construction work, and the work that is about to begin.

The Vice President commits the flaw of analogy and it has to be given due attention. The

durability of the roads depend on a number of other factors like climatic conditions, the nature of

surrounding soil, the traffic load and the number of vehicles encountered per day and their type

etc... The contribution of these factors must be quantified, and weighed against the constructed

Appian Roadways, to measure its durability.

Page 188: ARG-FINAL

Thus, the argument put forward by the Vice President, needs to address a number of questions

including: the road conditions prevalent in Good Intentions and Appian Roadways, the

similarities between Route 40 and Route101, effect of the new machinery and manage on the

Appian Roadways, employee count of Appian Roadways. Answers to these questions will serve

to strengthen the Vice President's argument, and make it a suitable candidate for implementation.

END